Está en la página 1de 140

PART-1

Directions (1-15): In each of the following sentences, parts of the sentence


are left blank. Beneath each sentence, five different ways of completing the
sentence are indicated. Choose the best alternative from among the five
options.

Q1. If the more articulate members of a community formed a coherent and


__________ class with a common interest, democracy would probably be
__________ into the rule of that intelligent, educated __________; even as it is,
the democracies of the modern world are much closer to this fate than they are to
the much-canvassed __________ of mob rule.
(a) united, overthrown, minority, benefits
(b) separate, overthrown, majority, problems
(c) united, replaced, minority, dangers
(d) separate, replaced, minority, benefits
(e) united, replaced, majority, dangers

Q2. I think it is the duty of science – I do not say of every individual man of
science – to study the means by which we can __________ ourselves to the new
world. There are certain things that the world quite obviously needs: __________,
as opposed to dogmatism in our beliefs; an expectation of co-operation, rather
than __________, in social relations; a lessening of envy and collective
__________. These are things which __________ could produce without much
difficulty.
(a) adapt, fanaticism, competition, malice, philosophy
(b) adopt, fanaticism, affection, malice, science
(c) reorient, tentativeness, war, consciousness, science
(d) adapt, tentativeness, competition, hatred, education
(e) orient, diffidence, competition, malice, education

Q3. As man’s tool of survival, reason has two basic functions: cognition and
evaluation. The process of cognition consists of discovering--- A---, of
identifying their nature, their ----B-----. The process of evaluation consists
of man discovering the relationship of things to himself, of identifying ---C-- to
him and ---D----, what should be sought and what should be avoided.
(a) A – what is beneficial, B – what is harmful, C – attributes and properties, D –
what things are
(b) A – what things are, B – what is beneficial, C – attributes and properties, D –
what is harmful
(c) A – what is beneficial, B – what things are, C – what is harmful, D – attributes
and properties
(d) A – attributes and properties, B – what is beneficial, C – what things are, D –
what is harmful
(e) A – what things are, B – attributes and properties, C – what is beneficial, D –
what is harmful

Q4. When you first arrive in a new culture, there is a period of __________ that
comes from the new situation and from a lack of information. It leaves you quite
__________ and in of help in the form of information and more. The second stage
begins as you start to __________ with the new culture. It is called the stage of
small victories. Each new __________ with the culture is fraught with peril. It is
__________ by anxiety and information collection and rehearsal. Then the event
occurs and you return home either __________ or defeated.
(a) confusion, vulnerable, interact, observation, preceded, jubilant
(b) excitement, vulnerable, observe, observation, followed, victorious
(c) confusion, dependent, interact, encounter, preceded, triumphant
(d) excitement, dependent, interact, encounter, followed, victorious
(e) confusion, dependent, observe, observation, preceded, triumphant

Q5. The core of modern doctoring is diagnosis, treatment and prognosis. Western
doctors have been __________ the wheezes and pains of their patients since the
17th century to identify the __________ disease of the cause of complaints. They
did it well and good __________ became the hallmark of a good physician. They
were less strong on treatment. But when sulphonamides were __________ in
1935 to treat certain bacterial infections, doctors found themselves with powerful
new tools. The area of modern medicine was born. Today there is a __________
array of complex diagnostic tests, and of pharmaceutical and surgical methods of
treatment.
(a) analyzing, deep rooted, diagnosis, conceived, burgeoning
(b) curing, deep-rooted, medicines, invented, dwindling
(c) diagnosing, widespread, prescriptions, conceived, escalating
(d) analyzing, underlying, diagnosis, discovered, burgeoning
(e) curing, widespread, prescriptions, discovered, dwindling

Q6. Today, Adam Smith is widely seen as the intellectual champion of self-
interest. This is a __________. Smith saw no moral virtue in selfishness; on
the contrary he saw its __________. Still less was he a __________ of capital
over labour, of the rising __________ over the common folk. His __________ of
self-interest and his regard for the people as a whole come through clearly in his
writings.
(a) delusion, benefits, critic, middle class, mistrust
(b) misconception, dangers, defender, bourgeoisie, suspicion
(c) fallacy, risks, detractor, bourgeoisie, distrust
(d) misconception, risks, critic, bourgeoisie, mistrust
(e) delusion, benefits, defender, middle class, suspicion

Q7. A conservation problem as important as that of soil erosion is the


__________ of soil fertility. Most agriculture was originally supported by the
__________ fertility of the soil; and, in areas in which soils were deep and rich
in minerals, __________ could be carried on for many years without the return
of any __________ to the soil other than those supplied through the natural
__________ of plant and animal wastes.
(a) loss, normal, cultivation, fertilizers, compost
(b) lack, usual, agriculture, nutriments, downfall
(c) loss, natural, farming, nutrient, breakdown
(d) lack, natural, farming, fertilizers, decay
(e) loss, usual, cultivation, crops, cultivation

Q8. Charles Darwin may have been __________ when he argued that competition
was the major driving force of evolution. He imagined a world in which
__________ battled for __________ and only the fittest __________. But new
research identifies the availability of “living space,” rather than competition, as
being of key __________ for evolution. Findings question the old adage of
“nature red in tooth and claw”.
(a) right, animals, survival, lived, reason
(b) erroneous, organisms, survival, lived, consequence
(c) mistaken, creatures, control, survived, importance
(d) wrong, creatures, power, survived, significance
(e) wrong, organisms, supremacy, survived, importance

Q9. Approximately 17% of all global greenhouse gas emissions come from the
__________ of tropical forests. So __________ and restoring these forests must
form part of a __________ climate change deal; reducing __________ from the
developed world is __________, but is not enough.
(a) burning, conservation, complete, forests, sine qua non
(b) elimination, preserving, broad, temperature, essential,
(c) destruction, conserving, comprehensive, emissions, essential
(d) trees, conserving, comprehensive, emissions, vital
(e) destruction, management, wide ranging, emissions, critical

Q10. A die-hard loyalist of Jayalalithaa, and by extension of his one-time -----


Ms. Sasikala, Mr. Panneerselvam, who was perceived as the archetypal ------
supplicant, chose his moment to strike back at the ------- in the party.
(a) benefactor, feudal, patricians
(b) beneficial, few, patrimony
(c) beneficent, refusal, patriarchal
(d) bonhomie, feudal, patricians
(e) feudal, patricians, benefactor,

Q11. The __________ nature of the food inflation over the last year has brought
the acuteness of food __________ in India into political focus.
(a) truculent, scarcity
(b) persistent, insecurity
(c) hostile, shortage
(d) unrelenting, surplus
(e) repugnant, conference

Q12. The food procurement policy of the government had two objectives:
__________ regional distribution of food grains at __________ prices and the
provision of a fair price to farmers.
(a) restricted, low
(b) limited, fair
(c) just, low
(d) equal, reasonable
(e) equitable, reasonable

Q13. The situation in Kashmir has always been __________, but policymaking
in New Delhi, which has remained “one track”, has only helped to __________
it further.
(a) sensitive, alleviate
(b) confounded, exacerbate
(c) fragile, aggravate
(d) violent, assuage
(e) peaceful, spoil

Q14. Being unsure of itself, reform is pursued __________ by a coalition


government constantly __________ of voters’ reactions.
(a) vigorously, hopeful
(b) slowly, chary
(c) hesitantly, wary
(d) nervously, circumspect
(e) reluctantly, eager

Q15. Palestinian officials say that there is no use holding talks with Israeli leaders
that are nothing more than a photo opportunity intended to create the __________
of a peace process while avoiding any substantive __________.
(a) impression, conflicts
(b) blueprint, problems
(c) proposal, arrangements
(d) impression, commitments
(e) conception, implementation

Solutions

S1. Ans.(c)
Sol. “common interest” and united will go together. Mob rule eliminates benefits
vs. dangers.

S2. Ans.(d)
Sol. Adopt is incorrect. Dogmatism as opposed to fanaticism does not make
sense; we need its opposite, hence tentativeness in the second blank. War and
consciousness will eliminate option C.

S3. Ans.(e)
Sol. Immediately after blank A, it is said identifying their nature which means A
and B are related to the nature of things – hence option (E). After C and D we
have “what should be sought and what should be avoided,” hence beneficial and
harmful combination in option(E).

S4. Ans.(c)
Sol. From a “new situation and lack of information,” there cannot be excitement.
Observe eliminates option (E). Observation eliminates option (A). Hence (C).

S5. Ans.(d)
Sol. If the doctors were doing this to “to identify…” something it has to be
“analyzing.” Diseases are not deep rooted but underlying, hence option (A) can
be eliminated in favor of option (D).

S6. Ans.(b)
Sol. The first blank is a contradiction of the first sentence. In comparison
misconception and fallacy score over delusion, besides benefits eliminates both
the options with delusion. Suspicion and defender fits in the context correctly.
hence option B is correct choice.

S7. Ans.(c)
Sol. It is easier to choose between lack and loss because of the paragraph is talking
about conservation. Usual can be easily eliminated in favor of natural and normal.
Option (E) is eliminated for the last two words and option (A) is also eliminated
for the last two words.
S8. Ans.(e)
Sol. “Animals” eliminates option (A) – this would exclude plants and other
organisms. Consequence eliminates option (B). Creatures, control eliminate
option (C). Creatures, power eliminate option (D). Option (E) is the best choice.

S9. Ans.(c)
Sol. Options (A) and (D) can be easily eliminated as the cause of the emissions
from these sources can be ruled out – burning is too specific. Conserving vs.
preserving vs. management – it is easy to see that conserving is the best. Hence
option (C).

S10. Ans.(a)
Sol. benefactor, feudal, patricians fits in the context of the sentence correctly.
benefactor-a person who gives money or other help to a person or cause.

feudal-according to, resembling, or denoting the system of feudalism. "the feudal


system"
patrician-a member of a long-established wealthy family.

S11. Ans.(b)
Sol. persistent, insecurity fits in the context of the sentence correctly. the
continuous increase in the food price has brought the scarcity and political
attention.

S12. Ans.(e)
Sol. equitable, reasonable fits in the context of the sentence correctly. the main
objective of the food procurement policy of the government is to distribute
equally and have reasonable price.

S13. Ans.(c)
Sol. fragile- (of a person) not strong or sturdy; delicate and vulnerable. the
situation in Kashmir has been weak for long and the idleness of policy makers in
New Delhi is only deteriorating the situation.

S14. Ans.(c)
Sol. The coalition government is not willing to apply the reform because govt. is
cautious of voters. Hence, hesitantly, wary fits in the context of the sentence
correctly.

S15. Ans.(d)
Sol. impression, commitments fits in the context of the sentence correctly.
PART-2

Directions (1-10): In each question, there are five sentences or parts of


sentences that form a paragraph. Identify the sentence(s) or part(s) of
sentence(s) that is/are correct in terms of grammar and usage (including
spelling, punctuation and logical consistency). Then, choose the most
appropriate option.

Q1. A. In 1849, a poor Bavarian imigrant named Levi Strauss


B. Landed in San Francisco, California,
C. at the invitation of his brother-in-law David Stem
D. owner of dry goods business.
E. This dry goods business would later became known as Levi Strauss &
Company.
(a) B only
(b) B and C
(c) A and B
(d) A only
(e) A, B and D

Q2. A. In response to the allegations and condemnation pouring in,


B. Nike implemented comprehensive policy changes in their labour policy.
C. Perhaps sensing the rising tide of labour concerns,
D. from the public would become a prominent media issue,
E. Nike sought to be a industry leader in employee relations.
(a) D and E
(b) D only
(c) A and E
(d) A and D
(e) B, C and E

Q3. A. Charges and counterchanges mean nothing.


B. to the few million who have lost their home.
C. The nightmare is far from over, for the government
D. is still unable to reach hundreds who are marooned.
E. The death count have just begun.
(a) A only
(b) C only
(c) A and C
(d) A, C and D
(e) D only
Q4. A. I did not know what to make of you.
B. Because you’d lived in India, I associate you more with my parents than with
me.
C. And yet you were unlike my cousins in Calcutta, who seem so innocent and
obedient when I visited them.
D. You were not curious about me in the least.
E. Although you did make effort to meet me.
(a) A only
(b) A and B
(c) A and E
(d) D only
(e) A and D

Q5. A. Relations are improving between the United States and Libya.
B. President Bush telephoned the Libyan leader, Col. Muammar el-Qaddafi,
C. on Monday, after Libya has paid about $1.5 billion
D. to State Department to clear up terrorism-related claims
E. from bombings and hijackings during the 1980’s.
(a) A only
(b) A and B
(c) A and E
(d) C only
(e) C and D

Q6. A. I’ve had a intense year.


B. Due to the book launch and movie release, I’ve been out there on display,
C. focusing too much on the external world.
D. Its time to look within, get in touch with myself again and create something
new.
E. As part of that, I have to stop the blog, interacting with media and other events.
(a) A only
(b) A and B
(c) A and E
(d) C only
(e) B and C

Q7. A. Now setting up Microsoft Project Manager to work with your company’s
existing
B. IT infrastructure can be a complicated task.
C. Systems will have to replace to make them compatible, space has to be
allocated
D. for new hardware and there are considerable cost in implementing a desirable
system.
E. A data center has to be setup and your IT department has to complete all this
in addition to their normal duties.
(a) A only
(b) B only
(c) B and E
(d) C only
(e) B and D

Q8. A. We are heading into an age in which jobs are


B. likely to be invented and made obsolete faster and faster.
C. The chances of today’s college kids working in the same jobs for the same
companies for their whole careers is about zero.
D. In such a age, the greatest survival skill you can have is the ability to learn
how to learn.
E. The best way to learn how to learn is to love to learn, and the best way to love
to learn is to have teachers who inspire.
(a) A only
(b) A and B
(c) A, B and E
(d) B and C only
(e) C and D

Q9. A. A New Zealand company called HortResearch is a world renowned fruit


science company,
B. and they develop a unique and innovative fruit and food products for
sustainability.
C. One of their previous innovative technologies were in developing a bio
sensor to measure the hormone levels in saliva.
D. This can be monitored in real time and also adopted to sports.
E. Right now, it is investigating technologies for measuring biochemical
indicators of stress and performance in real time.
(a) A only
(b) A and B
(c) A and E
(d) E only
(e) A, C and E

Q10. A. Maragadavalli was running her father’s household ever since her mother
had died when she was 13.
B. It took a long time for her family to settle down after that, and she was 26 by
the time she had got married.
C. With 13 years of running a household under her belt, she married into a family
meticulously run by mother-in-law.
D. She liked the fact that there were servant maids to help with the housework.
E. She had very less to do around the house except cut vegetable and wait for her
husband to come back from work.
(a) A only
(b) A and D
(c) A and E
(d) D only
(e) A, C and E

Directions (11-15): In the following, there are sentences or parts of


sentence labelled A, B, C, D and E. Choose for your answer the fragment
that carries an Error. Ignore punctuation error, if any.

Q11. A. And its equally important to spread the news about companies that treat
working parents
B. with fairness and respect, so that we all can try to work at these companies,
C. and send a message to others treating working parents
D. justly is a valuable employee recruitment and retention advantage.
E. No error
(a) A
(b) B
(c) C
(d) D
(e) E

Q12. A. I’ve been in Delhi for four days now


B. and so far the stay had been uneventful,
C. barring a stray incident
D. where I walked into a lamppost and then walked around Connaught Place
E. with a bloody nose.
(a) A
(b) B
(c) C
(d) D
(e) E

Q13. A. To be a citizen is to possess the rights


B. enshrined in our constitution,
C. and equally, the obligation of duty.
D. When one citizen upholds an individual right for another,
E. he enacts his duty to himself.
(a) A
(b) B
(c) C
(d) D
(e) E

Q14. A. This study takes its place alongside earlier research showing
B. that kids who are allowed to serve their own food
C. take smaller portions that they would typically be served
D. and ate 25 percentage less, and that kids tend to eat more
E. when the portions on their plates have been double.
(a) A
(b) B
(c) C
(d) D
(e) E

Q15. A. Sixty years is too small a period to acquire new civilizational traits.
B. and to mould our DNA. When that happens, we will automatically
C. realize the importance of the rule of law, the true meaning of freedom and
democracy
D. and then we will behave like citizens who will not allow anyone
E. to sell their votes, and MPs will not be on sold.
(a) A
(b) B
(c) C
(d) D
(e) E

Solutions
S1. Ans.(a)
Sol. B only. The word immigrant is spelt incorrectly in A. B has no errors. C is
incorrect because David Stern is parenthetical hence should be surrounded by
commas. D is incorrect because dry goods business needs a determiner – ‘owner
of a dry goods business’. E is incorrect in tense ‘would later become’ and not
‘would later became’.

S2. Ans.(d)
Sol. A and D. B is incorrect because the plural pronoun ‘their’ is used for Nike
(company) C is incorrect because of the comma at the end separating it from the
remaining part of the phrase ‘labour concerns from the public’. E has the incorrect
article a before industry – an industry is correct.
S3. Ans.(d)
Sol. A, C and D are correct. B is incorrect in the singular use of ‘million’ with
few – it should be few millions. E needs to link the singular subject the death
count(use singular verb -has). Countercharge as one word is correct like
counterchallenge.

S4. Ans.(e)
Sol. A and D are correct. B has a tense error, it should be I associated… C also
has a tense error, it should be who seemed so innocent… E is incorrect, although
is a conjunction; it is here as an adverb – it is better replaced with however.

S5. Ans.(b)
Sol. A and B. Fragment C has a tense error ‘has paid ‘should be ‘had paid’
because of ‘telephoned’ in the earlier fragment. D should have ‘the’ before State
Department. E has an inappropriate apostrophe in the plural of 1980s. A and B
are correct. Hence option B.

S6. Ans.(e)
Sol. B and C. A is incorrect – it should be an intense year. In D it’s is needed
instead of its. E has a parallelism error. “to stop the blog, media interactions and
other events” would be correct with all the items in the series as nouns.

S7. Ans.(b)
Sol. B only. A is incorrect because there should be a comma after introductory
adverbs like now. (Now the trouble began is correct – now is not an introductory
word.) B is correct. C is incorrect – systems will have to replace should read as
to be replaced. D is incorrect. It should there are considerable costs. E is incorrect
IT department is singular (the verb has is correct) but the pronoun ‘their’ is
incorrect. The pronoun should be its.

S8. Ans.(c)
Sol. A, B and E are correct. ‘head into’ is idiomatically correct. (Motorola is
heading into an abyss) C is incorrect because ‘chances’ needs a plural verb – the
sentence has is. In such a age is incorrect – it should be “an age”.

S9. Ans.(c)
Sol. A and E only. B is incorrect – the pronoun “they” is inconsistent with
HortResearch which is singular, also, “a unique and innovative fruit and food
products” should be corrected to “unique and innovative fruit and food products”.
C is incorrect “one of their” is inconsistent with HortResearch, and one
of previous innovative technologies is singular, hence the verb should be was and
not were. D is incorrect because adopt is confused with adapt. A and E are correct.
S10. Ans.(d)
Sol. D only. A has a tense error – when since is used as conjunction to show the
time the perfect tense is a must. The sentence should read “Maragadavalli had
been running her father’s household ever since …”. Also there is ambiguity in “
… mother had died when she was 13.” B also has a tense error. “By the time she
got married” rather than “she had got married” – the unnecessary shift in tense
makes the sentence meaningless. E is incorrect. The intensifier ‘very’ is used
incorrectly with the comparative less, it should read very little rather than very
less.

S11. Ans.(a)
Sol. It’s should be used instead of its in the first part of the sentence.

S12. Ans.(d)
Sol. D should read as ‘in which I walked into …’ or ‘when I walked into …’

S13. Ans.(d)
Sol. the preposition ‘for’ should be corrected to ‘of’ – ‘right of another’, rather
than ‘right for another’

S14. Ans.(d)
Sol. 25 percentage less is erroneous – 25 percent less is correct usage. Use the
word “percent” with number; use “percentage” without using a number.
For example: 50 percent children, and a considerable percentage of children.

S15. Ans.(e)
Sol. We will not allow anyone to sell their votes has a problem in pronoun
agreement, anyone is singular hence he or she should be used rather than the
plural their. Sixty years as period is singular, hence the singular verb is correct.
PART-3
Directions (1-15): In each question below, the word at the top of the question
is used in four different ways. Choose the option in which the usage of the
word is INCORRECT or INAPPROPRIATE.

Q1. ENTER
(a) He was assigned the task of entering the names of all eligible voters.
(b) She decided to enter her child into the best school in the neighborhood.
(c) His parents advised him to acquire an MBA before entering upon a career.
(d) When money is involved other considerations should not enter.
(e) All sentences are correct.

Q2. WITHDRAW
(a) The steno was asked to withdraw the offending words from the letter she
typed.
(b) The college withdrew his academic credit after it was established that he had
copied in the test.
(c) The minister withdrew his name from the list of nominees.
(d) After the loss of her husband she had withdrawn farther and farther into
herself.
(e) All sentences are correct.

Q3. DRAW
(a) People are afraid that the festival will draw undesirable elements to the town.
(b) The deposits are expected to draw interest close to 10 percent.
(c) The teacher drew the children into the room to see the pictures.
(d) To avoid confusion later, a list was drawn of all those who would attend the
function.
(e) All sentences are correct.

Q4. STACK
(a) There were stacks of books on the bedside table and floor.
(b) The committee is stacked with members from energy-producing states.
(c) She is continually stacked by headaches.
(d) The convict tried to stack the jury.
(e) All sentences are correct.

Q5. DEVOLVE
(a) His death devolved the end of an era in superlative cinematic techniques.
(b) The senator devolved the duties of office upon a group of aides.
(c) The estate devolved to an heir that everybody had assumed to be dead.
(d) After several hours the discussion had devolved into a shouting match.
(e) All sentences are correct.

Q6. DESIST
(a) Kindly desist from making so much noise.
(b) He desisted further efforts to dissuade them.
(c) Before the medical exam you must desist from food.
(d) During Ramadan, Muslims desist from smoking during the day.
(e) All sentences are correct.

Q7. DISSIPATE
(a) The wind finally dissipated the smoke.
(b) The industrious people from Gujarat are dissipated over the whole of India.
(c) The young sometimes seem to dissipate their energy in useless pursuits.
(d) The dark clouds which threatened rain finally dissipated.
(e) All sentences are correct.

Q8. EVINCE
(a) She evinced her approval by smiling.
(b) The man who was caught stealing did not evince any remorse.
(c) The proposal evinced a storm of protest from the members.
(d) She did not evince the least surprise at seeing me there.
(e) All sentences are correct.

Q9. DECOY
(a) He acted as a decoy to draw the dogs’ attention away from the children.
(b) The policewoman acted as a decoy when the police were trying to catch the
murderer.
(c) The bird-trap was empty as he had caught a decoy, and he had to kill it.
(d) They used flares to decoy enemy missiles.
(e) All sentences are correct.

Q10. FEIGN
(a) She feigned illness to avoid going to the party.
(b) The politician feigned the attention of his listeners to non-controversial issues.
(c) On the phone, he feigned his sister’s voice and said that he was not at home.
(d) She introduced me as her cousin, and everyone feigned to believe it.
(e) All sentences are correct.

Q11. INDICT
(a) They were indicted for conspiracy and spent a year in jail.
(b) In Bosnia, US troops have usually declined to take part in the arrest of indicted
war criminals.
(c) Last week, he was indicted by a grand jury.
(d) The critics have the right to praise or indict a literary work.
(e) All sentences are correct.

Q12. RAMBLE
(a) Where would American cinema be without the car ramble or the road movie?
(b) Avoid long rambling sentences, jargon or unexplained acronyms.
(c) We have tours to suit all levels of ability from a gentle ramble to a Himalayan
expedition.
(d) The internet forum gives you an opportunity to ramble on a bit.
(e) All sentences are correct.

Q13. TRANSIT
(a) Electric cars are a nice idea, but they won’t work for mass transit or cargo.
(b) Passengers holding a direct airside transit visa will not be able to pass through
immigration control.
(c) The transit of the property into the parties’ joint names was irrelevant.
(d) Mumbai and Delhi are building the infrastructure for rapid transit systems.
(e) All sentences are correct.

Q14. PLACATE
(a) The regulations imposed in the West to placate environmental opposition to
GM foods effectively prevent the Third World from developing GM foods.
(b) The facts are incorrect and they have placated members of all political parties
across the country.
(c) However, his statement did little to placate the unions who say that even
negotiations cannot help in this matter.
(d) The government has a greater imperative to placate the farming lobby than
protecting wildlife because of its political leverage.
(e) All sentences are correct.

Q15. ABDICATE
(a) People have decided to ask the King to abdicate in favour of his son.
(b) We cannot abdicate the responsibility to deal with these acts in the correct
manner.
(c) Our people have opted for democracy as a way of life from which we will not
abdicate.
(d) People wonder whether computer programmes will abdicate the role of the
human brain.
(e) All sentences are correct.
Solutions
S1. Ans.(b)
Sol. ‘to enter her child into the best school’ this phrase is incorrect in terms of
usage. In all other sentences, the usage of the word ‘enter’ is correct.
S2. Ans.(a)
Sol. ‘to withdraw the offending words from the letter she typed’ the use of the
word ‘withdraw’ is incorrect in this sentence.

S3. Ans.(d)
Sol. In sentence D, the word ‘draw’ is incorrectly used.

S4. Ans.(c)
Sol. In sentence C, the use of the word STACK is inappropriate.

S5. Ans.(a)
Sol. DEVOLVE-transfer or delegate (power) to a lower level, especially from
central government to local or regional administration.
pass into (a different state, especially a worse one); degenerate.
In sentence A, The word ‘DEVOLVE’ is incorrectly used. In other sentence,
DEVOLVE is correctly used.

S6. Ans.(b)
Sol. DESIST-stop doing something; cease or abstain. In sentence B, the word
DESIST is incorrectly used.

S7. Ans.(b)
Sol. DISSIPATE-(with reference to a feeling or emotion) disappear or cause to
disappear. The word DISSIPATE is incorrectly used in sentence B.

S8. Ans.(c)
Sol. EVINCE-reveal the presence of (a quality or feeling); indicate. The word
‘’evince’ is incorrectly used in option C.

S9. Ans.(c)
Sol. DECOY- lure or entice (a person or animal) away from their intended course,
typically into a trap. According to the meaning of the word ‘decoy’, the usage in
sentence C is incorrect.

S10. Ans.(b)
Sol. FEIGN-pretend to be affected by (a feeling, state, or injury). The word feign
is incorrectly used in the sentence B.

S11. Ans.(d)
Sol. INDICT-formally accuse of or charge with a crime. The word ‘indict’ is
incorrectly used in option D.
S12. Ans.(a)
Sol. RAMBLE-talk or write at length in a confused or inconsequential way.
a walk taken for pleasure in the countryside. The word ‘ramble’ is incorrect in
option A.

S13. Ans.(c)
Sol. TRANSIT-the action of passing through or across a place.
the carrying of people or things from one place to another. The word ‘transit’ is
incorrect in sentence C.

S14. Ans.(b)
Sol. PLACATE-make (someone) less angry or hostile. The word ‘placate’ is
incorrect in sentence B.

S15. Ans.(d)
Sol. ABDICATE-(of a monarch) renounce one's throne.
fail to fulfil or undertake (a responsibility or duty). ‘abdicate’ is incorrectly used
in option D.

PART-4

Directions (1-15): In the following questions a paragraph is given and a


sentence or a phrase/clause is missing in this paragraph. In some questions
last sentence is missing. From the given options choose the most suitable
sentence or phrase/clause that completes the idea expressed in the given
paragraph.

Q1. Perhaps the simplest and easiest way to understand is the argument of
the First Cause. I may say that when I was a young man and was debating
these questions very seriously in my mind, I for a long time accepted the
argument of the First Cause, until one day, at the age of eighteen, I read
John Stuart Mill’s Autobiography, and I there found this sentence: “My
father taught me that the question ‘Who made me?’ cannot be answered,
since it immediately suggest the further question ‘Who made god?’’’
_______________

If everything must have a cause, then God must have a cause. If there can
be anything without a cause, it may just as well be the world as God, so that
there cannot be any validity in that argument.
(a) There is no reason why the world could have come into being without a
cause.
(b) That argument, I suppose, does not carry very much weight nowadays,
because, in the first place, cause is not quite what is used to be.
(c) That very simple sentence showed me, as I still think, the fallacy in the
argument of the First Cause.
(d) It brings us to the central truth that God is the ultimate source and essence of
everything.
(e) None of the above fits in the paragraph correctly.

Q2. Then there is a very common argument from natural law. That was a
favorite argument all through the eighteenth century, especially under the
influence of Sir Isaac Newton and his cosmogony. People observed the
planets going around the sun according to the law of gravitation, and they
thought that God had given a behest to these planets to move in that
particular fashion, and that was why they did so. _______________
(a) Nowadays, we explain the law of gravitation in a somewhat complicated
fashion that Einstein has introduced.
(b) Modern science has failed to explain this incongruity.
(c) You no longer have the sort of natural law that you had in the Newtonian
system.
(d) That was, of course, a convenient and simple explanation that saved them
the trouble of looking any further for explanations of the law of gravitation.
(e) None of the above fits in the paragraph correctly.

Q3. The fountains mingle with the river,


And the rivers with the ocean;
The winds of heaven mix forever,
With a sweet emotion;
_________________________;
(a) This is the power of love
(b) Nothing in the world is single
(c) This is the seed of creation
(d) What’s life without love
(e) None of the above fits in the paragraph correctly.

Q4. All things by a law divine


In one another’s being mingle:
______________________________
(a) Why not I with thine?
(b) Let’s make a jingle
(c) It takes two to tango
(d) God is not away from us
(e) None of the above fits in the paragraph correctly.

Q5. Conventional education makes independent thinking extremely


difficult. _______________ To be different from the group or to resist
environment is not easy and is often risky as long as we worship success
(a) Creativity is crushed by orthodoxy.
(b) Innovative thinking is the key.
(c) This has ruined many careers.
(d) Conformity leads to mediocrity.
(e) None of the above fits in the paragraph correctly.

Q6. Though there is a higher and wider significance to life, of what value is
our education if we never discover it? We may be highly educated, but if
we are without deep integration of thought and feeling, our lives are
incomplete, contradictory and torn with many fears; _______________.

(a) the ‘well-educated’ are ignoramus louts


(b) and as long as education does not cultivate an integrated outlook on life, it
has very little significance
(c) and integrity is the key to spiritual evolution
(d) it has been rightly remarked, “I never let school to interfere in my
education”
(e) None of the above fits in the paragraph correctly.

Q7. The function of education is to create human beings who are integrated
and therefore intelligent. _______________. We may take degrees and be
mechanically efficient without being intelligent. Intelligence is not mere
information; it is not derived from books, nor does it consist of clever self-
defensive responses and aggressive assertions.

(a) Education should help us to discover lasting values so that we do not merely
cling to formulas or repeat slogans
(b) Education should not encourage the individual to conform to society or to be
negatively harmonious with it
(c) One who has not studied may be more intelligent than the learned
(d) Unfortunately, the present system of education is making us subservient,
mechanical and deeply thoughtless
(e) None of the above fits in the paragraph correctly.

Q8. Politicians may be corrupt, but have to seek reelection, and to that
extent are accountable to voters. But civil servants are virtually
unsackable, unaccountable and widely corrupt. You cannot change this
overnight. _______________

(a) However, you can create jobs for the unemployed.


(b) So, go easy on making temporary workers permanent.
(c) But you can halt the growth of unsackable, unaccountable staff.
(d) So, devise safeguards against false accusations.
(e) None of the above fits in the paragraph correctly.

Q9. The “grand sweep of history” has become a much overused cliché. It
incorporated the belief that change stemmed from big ideas that motivated
individuals, classes and nations. _______________.

(a) The Bolshevik Revolution, whose impact dominated the 20th century, was
prompted by disillusionment among the proletariat
(b) This finds support in Namier’s view that big ideas are less important than
mundane and even base considerations
(c) Mass movements, cannot be judged by pronouncements of those who
manage to filch them
(d) Thus, the French Revolution happened because the idea of liberty, equality
and fraternity motivated people to overturn the decrepit absolute monarchy
(e) None of the above fits in the paragraph correctly.

Q10. A country that retains the death penalty needs constantly to fine-tune
its clemency jurisprudence as the second best option. The Supreme Court’s
latest verdict on death row convicts is a thoughtful exposition of the law in
this regard. ______________________________

(a) Commuting the death sentences of 15 convicts to life sentences has


significantly expanded the scope for judicial intervention to save the lives of
convicts after the rejection of their mercy petitions.
(b) The court has laid down fresh rules to humanize the treatment of those
facing the gallows, right up to the moment of their execution and even after that.
(c) The breadth of this ruling is not as impressive: it fails to remove all lingering
doubts about the rule against undue delay.
(d) The court has crafted a new rule that families of convicts ought to be
informed in writing as soon as their mercy petitions are rejected.
(e) None of the above fits in the paragraph correctly.

Q11. After successfully eradicating smallpox in 1980, India has now gone
three straight years without reporting any new case of poliomyelitis
infection (“polio”). This qualifies it to receive the World Health
Organisation’s (WHO) certification for being polio-free. Undoubtedly, this
is a victory that has been fought every inch of the way by myriad agencies
on a number of fronts and against what seemed like insurmountable odds.
______________________________
(a) The lessons learnt are precious beyond words and the expectation is that
these will be harnessed to fight other infectious diseases that plague the country.
(b) There is also the remaining challenge of treating and rehabilitating those
who have already been crippled by the disease.
(c) Of course, polio vaccination is not a cure-all solution for all infectious
diseases.
(d) In the mid-1990s the vaccination programme that was undertaken involved
the government, United Nations bodies, charitable organizations and private
donors.
(e) None of the above fits in the paragraph correctly.

Q12. Up to this point, Jordan Belfort is no different from countless eager


MBA graduate in India who work in the stock or bond markets for global
financial firms. Belfort’s lifestyle, while perhaps more (or less?) excessive
than that of India’s super-rich, is still something a lot of us covet. He
acquires a harem, a hot blonde wife, a daily dose of recreational drugs, a
yacht, a yellow Jaguar and a white Ferrari.

(a) As a job creator, he transforms hopeless, low-end drug dealers into corporate
sharks, and even gives a desperate single mother benefits that the US’ social
welfare system overlooks.
(b) He lowers himself to unfathomed moral depths even as he soars to new
heights of success.
(c) Surely many of us will laud Belfort when he says: “At least as a rich man,
when I have to face my problems, I can show up in the back of a stretch
limousine, wearing a two-thousand-dollar suit and a twenty-thousand-dollar
gold watch!”
(d) These are charismatic brands that several of India’s merchant princes flaunt
and made more familiar to us through thousands of Bollywood fantasies.
(e) None of the above fits in the paragraph correctly.

Q13. The yearning for money as succor drives contemporary capitalism. If


every revolution and alternative has failed, why not work to enable the one
that actually exists, why not do what your stockbroker tells you, and keep
investing to circulate money in the economy? The original Forbes magazine
expose that labelled Belfort “The Wolf of Wall Street” likened him to a
“twisted Robin Hood” who takes from the rich and given to himself and his
squad of losers.

(a) It is this observation that makes Scorsese’s The Wolf of Wall Street
profound, locating and attacking the very appeal of money.
(b) Scorsese refuses to dish out false platitudes that “crime does not pay” nor
does he echo the sentimentalism of Oliver Stone’s Wall Street movies.
(c) As a job creator, he transforms hopeless, low-end drug dealers into corporate
sharks.
(d) Is that not what we expect from the market, what keeps housewives glued to
the television, watching CNBC for the latest stock information, and what drives
many to start demat accounts?
(e) None of the above fits in the paragraph correctly.

Q14. Like Company Limited, Scorsese’s The Wolf of Wall Street in an


exploration of the contemporary world that few would have expected from
these two artistes, given their refined sensibilities. The Wolf of Wall Street
attacks the lifestyle of the middle-class, the world of advertising and
consumerism, the lust for the good life and the protection it offers.

(a) the lust for good life propels us to struggle and survive in this big bad world.
(b) behind the veneer of consumerism is an effort to assert one’s individuality.
(c) the elite and the downtrodden are unaffected by this false glamour.
(d) The visible surface and texture of contemporary life corrupts us all, making
us wolves thronging the pack of the alpha male rather than being benign, though
gullible, sheep.
(e) None of the above fits in the paragraph correctly.

Q15. It is obvious today that America has defaulted on this promissory note
insofar as her citizens of colour are concerned. Instead of honouring this
sacred obligation, America has given the Negro people a bad check, a check
which has come back marked “insufficient funds.”

(a) But we refuse to believe that the bank of justice is bankrupt.


(b) So we have come to cash this check – a check that will give us upon demand
the riches of freedom and the security of justice.
(c) This note was a promise that all men, yes, black men as well as white men,
would be guaranteed the unalienable rights of life, liberty, and the pursuit of
happiness.
(d) In a sense, we have come to our nation’s capital to cash a check.
(e) None of the above fits in the paragraph correctly.
Solutions

S1. Ans.(c)
Sol. Option A, It is contrary to the main argument discussed in the passage,
which tries to refute the argument of the First Cause.
Option B, It does not go with the theme of the paragraph, as discussed above.
Option C, As the author believes that there is no validity in the argument of the
First Cause, so the third choice is correct.
Option D, Misleading choice. Out of scope of the passage, as it does not talk
about the qualities of God.

S2. Ans.(d)
Sol. Option A, Gravitation is not the main focus of the argument; moreover, the
para talks about Newton, and not Einstein.
Option B, Does not follow; why would Modern science explain a popular belief.
Option C, Does not make sense as the passage nowhere says that natural law
was a part of the Newtonian system.
Option D, This option takes the argument forward, ‘that was……’ is the link
that takes the case further by questioning the validity of the conventional
wisdom. Hence, option D is the answer

S3. Ans.(b)
Sol. The lines are simple and direct. You have to find something that concludes
the idea. So a, d, and c are quite tempting, although quite general in inference.
Option A, One cannot deduce it from the argument.
Option B, concludes the idea mentioned in each line, hence, it is the answer.
Option C, Again, this option is too farfetched.
Option D, Once again, this option is too farfetched.

S4. Ans.(a)
Sol. Now, this should be pretty logical for those who believe in love.
Option A, Hits the bulls eye, the only choice that logically completes the lines.
Hence, option A is the answer.
Option B, The choice is too creative although it rhymes well with the previous
line.
Option C, ‘It takes two to tango’ is generally used with a negative sense, so out
of question.
Option D, Is out of scope, do not be misled by the word ‘divine’.

S5. Ans.(d)
Sol. Option A, B; The passage nowhere talks about creativity or innovative
thinking, so options A and B are out.
Option C, Takes the negative tone too far, so we have to eliminate this choice.
Option D, is clearly the missing logical link, which is taken further by the last
line of the passage. Mentioned in the last line ‘to resist environment is not easy
and often risky’ supports this statement. Hence, it is the right answer.

S6. Ans.(b)
Sol. Option A, The first choice is downright offensive, therefore not possible.
Option B, It includes both the elements ‘education’ and ‘integration’, hence, the
right choice and is the answer.
Option C, Out of scope as the passage is about the ‘value of education’ and not
spiritual evolution.
Option D, It is well said but out of place because the paragraph does not say that
education is not necessary.

S7. Ans.(c)
Sol. Option A, It provides extra information which is out of scope of the
passage.
Option B, It does not go with the theme of the passage, which talks about the
role of education in developing intelligence.
Option C, It fills in the missing link. It has been illustrated in the last two lines.
Option D, This does not go with the last line, which tries to define real
intelligence.

S8. Ans.(c)
Sol. Option A, It strays from the theme of the topic and hence, eliminated.
Option B, The previous line talks about the civil servants, so no connection
between the two.
Option C, It justifies the you-cannot-change-it-overnight element in the
previous line, hence, the answer.
Option D, It is totally unrelated to the topic.

S9. Ans.(d)
Sol. Option A, The passage talks about big ideas motivating the individuals or
nations, this option talks about negative motivation of the proletariat.
Option B, It is counter to the theme of the paragraph which talks about the
sweeping influence of the big ideas.
Option C, This is contradictory to the main idea of the paragraph.
Option D, Gives an illustration of how big ideas catch the popular imagination
and create a wave of history.

S10. Ans.(a)
Sol. Option A, provides a solid example to support the main idea about fine-
tuning the clemency jurisprudence.
Option B, It is an extension of the topic, but it deviate from the topic of mercy
petitions or clemency jurisprudence.
Option C, Goes against the tone of the paragraph which is positive, ‘thoughtful
exposition’ has been used
Option D, It provides secondary information but fails to logically complement
the last sentence.

S11. Ans.(a)
Sol. Option A, Goes with the tone of the paragraph which is laudatory.
Option B, It does not follow the last sentence which is positive.
Option C, It dilutes the tone of the paragraph which is positive.
Option D, ‘the vaccination programme’ does not have its antecedent in the
previous line.

S12. Ans.(d)
Sol. Option A, This could have been a possible choice, but it does not flow from
the last sentence.
Option B, This option is quite tempting, but misses on the ‘India’ element
interspersed throughout the paragraph.
Option C, Last sentence in option C does not refer to any ‘problems’ as
mentioned in this statement.
Option D, It gels both the elements mentioned in the given lines, global brands
and the great Indian dream.

S13. Ans.(d)
Sol. Option A, It is not possible as the previous line talks about Forbes’ review
of the movie.
Option B, The paragraph nowhere talks about Scorsese, so this option is
eliminated.
Option C, The last line does not have Belfort as the main subject, so ‘he’ cannot
be used for lack of clear antecedent.
Option D, The author tries to argue that the greed of common man to make a
fortune is not much different from a fraudster like Belfort who manipulates the
stock market. Option D is in line with the theme of the paragraph. Hence, option
D is the answer.

S14. Ans.(d)
Sol. Option A, It does not go with the negative tone of the last line, ‘attacks the
lifestyle.’
Option B, Again, this option changes the tone of the passage which is negative.
Option C, This an extraneous piece of information which falls outside the scope
of the discussion.
Option D, As it explains how it ‘attacks the middle class’ and uses the analogy
of wolves and sheep to support the case.

S15. Ans.(a)
Sol. Let us eliminate the options:
Option B talks about how does ‘voting’ come into picture. Passage does not
mention that or is not related to that. Hence, option (B) is not the answer.
Option C looks tempting but does not flow from the passage.
Option D sounds like a U-turn and hence, cannot be the answer.
Option A is the only option left out. Hence it is the answer.

PART-5
Directions (1-15): Four sentences are given with a blank in each. Five words
are also given. The blank in each sentence can be filled by one or more words.
Similarly, each word given in the choices can go into any number of sentences.
Identify the number of sentences each word can go into and mark as your
answer the maximum number of sentences any word can go into.

Q1. A. The millionaire has __________ Rs. 10,00,000 for this priceless artifact.
B. In a/an __________ to rescue the child who was caught in fire, she sustained
burns.
C. I went to the railway station to __________ goodbye to my friend.
D. I was asked to __________ a price by the auctioneer.
(a) quoted
(b) attempt
(c) bid
(d) say
(e) spend

Q2. A. He __________ many spine-chilling anecdotes to us.


B. The law extends to several __________ groups.
C. This person is not __________ to me in any way, he is a trickster.
D. The two groups are __________ to each other.
(a) allied
(b) connected
(c) related
(d) recounted
(e) narrated

Q3. A. Can you please __________ the bell?


B. The words spoken by him still __________ in my ears.
C. Mahatma Gandhi was the __________ leader of the Non-Cooperation
movement.
D. I shall give you a __________ after reaching home.
(a) sound
(b) ring
(c) chief
(d) call
(e) sell

Q4. A. Many students still __________ around the college, even after the classes
are over.
B. Every citizen of India should __________ his head in shame at the brutal
killings in the name of religion.
C. There is ample space in the wardrobe for you to __________ your clothes.
D. “__________ the perpetrators of the crime!” shouted the angry mob.
(a) execute
(b) loiter
(c) hang
(d) bend
(e) store

Q5. A. This book pertaining to fine arts is beyond a __________ man’s


comprehension.
B. I helped mother to __________ the table for breakfast.
C. In her hour of distress she had to __________ her hopes on her relatives to
help her.
D. He requested me to __________ some money.
(a) pin
(b) lay
(c) lend
(d) ordinary
(e) common

Q6. A. My friends tried to cheer me up, as I was in a depressed __________ of


mind.
B. All the senior leaders of the party have assembled together to __________ a
new election strategy.
C. He wanted to change the __________ of his spectacles.
D. Can you please __________ this picture for me?
(a) build
(b) bend
(c) frame
(d) state
(e) draw

Q7. A. Despite being hardworking he could never __________ success in his


career.
B. He is such a spoilsport that he cannot __________ a joke against himself.
C. This toothpaste has the __________ of mint.
D. The __________ of mango is relished by the young and old alike.
(a) taste
(b) flavour
(c) enjoy
(d) experience
(e) small

Q8. A. Akbar was known for his benevolence, people were happy during his
__________.
B. Cleopatra has an ambition to __________ the world as a queen of beauty.
C. Better to __________ in hell than to serve in heaven.
D. One should not allow indolence to __________ over oneself.
(a) reign
(b) dominate
(c) rule
(d) tenure
(e) overpower

Q9. A. I do not __________ him as a brilliant performer.


B. The __________ of interest offered by Nationalized banks is very low.
C. At any __________ I shall see that you get over the crisis.
D. He rose from the __________ of a soldier to that of a commissioned officer.
(a) rank
(b) cost
(c) rate
(d) value
(e) see

Q10. A. The __________ accused in the murder, was sentenced to death.


B. __________ time soap operas on the television charge exorbitant rates from
advertisers.
C. Abolition of illiteracy should be the __________ concern of every state
government.
D. The office of the Vice-chancellor is in the __________ building of the
University.
(a) mainly
(b) prime
(c) primitive
(d) chief
(e) only

Q11. A. The little boy was pampered by his grandparents, who catered to his
every __________.
B. “Your __________ is my command”, said the genie to the prince.
C. I __________ you could understand me better.
D. Being an easygoing person, she behaves according to her own __________
and fancy.
(a) whim
(b) demand
(c) wish
(d) hope
(e) need

Q12. A. I could finish my work __________ ahead of time.


B. He is __________ off when compared to his other siblings.
C. By his attitude, I could make out that he does not mean __________.
D. She is __________ dressed when compared to other women in the gathering.
(a) much
(b) better
(c) well
(d) good
(e) harm

Q13. A. For many years India was __________ in a tricky situation, whether to
go in for a nuclear weapon or not.
B. The marathon innings of the batsman came to an end when he was finally
__________ by the same bowler who troubled him the most.
C. The notorious burglar was finally __________ by the cops.
D. I was __________ unaware when my photograph was clicked.
(a) held
(b) seized
(c) apprehended
(d) caught
(e) trapped

Q14. A. The management took no __________ of the problems pertaining to the


workers.
B. The class teacher was called to __________ for the dismal performance of the
students in the examination.
C. Many young cricket fans __________ Tendulkar as a hero.
D. The principal promised the students that he would __________ their demands.
(a) consider
(b) explain
(c) accurate
(d) estimate
(e) notice

Q15. A. The jail authorities have decided to __________ some of the prisoners,
who possessed a clean record, as a gesture of good will.
B. He decided to __________ himself of all responsibilities by handing over
charge to his successor.
C. Since he is a spendthrift he is very __________ in spending money.
D. Government aided schools are now offering to teach computer course to
students __________ of charge.
(a) devoid
(b) release
(c) lavish
(d) free
(e) parole

SOLUTIONS

S1. Ans.(c)
Sol. ‘bid ‘fits in all the above sentences correctly.

S2. Ans.(c)
Sol. ‘related ‘ fits in all the above sentences correctly.

S3. Ans.(b)
Sol. ‘ring ‘fits in all the above sentences correctly.

S4. Ans.(c)
Sol. ‘hang ‘fits in all the above sentences correctly.

S5. Ans.(b)
Sol. ‘lay ‘fits in all the above sentences correctly.

S6. Ans.(c)
Sol. frame -fits in all the above sentences correctly.

S7. Ans.(a)
Sol. taste -fits in three sentences correctly.
S8. Ans.(c)
Sol. ‘rule ‘fits in all the above sentences correctly.

S9. Ans.(c)
Sol. ‘rate ‘ fits in all the above sentences correctly.

S10. Ans.(b)
Sol. fits in all the above sentences correctly.

S11. Ans.(c)
Sol. ‘WISH’ fits in all the above sentences correctly.

S12. Ans.(c)
Sol. ‘well ‘fits in all the above sentences correctly.

S13. Ans.(d)
Sol. caught -fits in all the above sentences correctly.

S14. Ans.(a)
Sol. ‘consider ‘ fits in all the above sentences correctly.

S15. Ans.(d)
Sol. ‘free ‘fits in all the above sentences correctly.

PART-6

Directions (1-10): In each of the following questions, a paragraph with a


blank is given. From the five choices given below, select the sentence which
can go into the blank to make the paragraph logically coherent.

Q1. Srinagar is the capital of Kashmir. There are very beautiful scenes all
round. The Dal Lake is one among them. [__________] We can hire one of
them and voyage along the length and breadth of the lake or live in it for a week
or so.
(a) There are several guides to take us round.
(b)There we can see a number of house boats waiting to be hired by the tourists.
(c)It is difficult to count them.
(d)It was dark inside and bright outside.
(e) There are several tourists looking around the place.

Q2.Polio-affected children are found everywhere in India. Recently the


Government has started Polio-eradication Scheme. [__________] Unless we
take care to co-operate the purpose cannot be fulfilled.

(a) We must help children to take the proper vaccine.


(b) We must take the children to get vaccinated.
(c) Children must remain without any movement.
(d) We must make children exercise.
(e) We need experts to undertake such projects.

Q3. Many film-stars have recently migrated to the area of television.


[__________] As the viewers increase their popularity also increases. T.V
serials appear to be more paying than the feature films.

(a) Television gives a better chance of action.


(b) There they have a greater number of viewers.
(c) Television is a house hold affair.
(d) If we do not like a TV program we can turn it off.
(e) Television is useful in many ways.

Q4. Man is trying to find out modern means of producing electric power.
The solar panel is one of them. [__________] This electricity can light
lamps, turn fans or work small household appliances.

(a) It is very cheap and affordable.


(b) We ourselves can make one such device.
(c) It converts sunlight into electricity.
(d) It is easy to work and beautiful to look at.
(e) Electricity is a good substitute for sunlight.

Q5. Air-pollution is one of the gravest problems faced by city-dwellers.


Foul gas liberated from heaps of waste matter is one source of pollution.
[__________] Recently the Government has made arrangements to
measure it. If it is above the allowable limit the vehicles are prohibited
from plying along the public roads.

(a) Another source is the smoke emitted by vehicles.


(b) Foul water in the channels is another.
(c) The smell from toddy shops pollutes the air.
(d) Open drainage pollutes the air.
(e) People suffer due to smoke emitted by vehicles.

Q6. Corruption has become rampant in India. Corrupt officials and


politicians are ruling over the hapless common man. [__________] People
must join together and force them to take necessary legal action.

(a) Politicians with integrity simply withdraw from the scene.


(b) Such officials are welcomed by the general public.
(c) Even police authorities are reluctant to face them.
(d) They do it with the connivance of topmost authorities.
(e) Corruption is the breeding ground for all evils.

Q7. Feature films have a very great influence upon the common people.
They have no hesitation in regarding actors as gods. [__________] They
must utilize this love and regards to serve the people, not to exploit them.

(a) Gods are expected to protect the devotees.


(b) Though they are gods they are human enough to err.
(c) Gods never walk on earth or collect money.
(d) Some people are ready even to die for them.
(e) Actors are viewed as gods.

Q8. Most of us fall victim to some sort of disease or the other. We can avoid
diseases by leading a hygienic life. It is better to bear in mind a simple
truth. [__________] The consciousness of this truth will lead us to
happiness.

(a) It is impossible to have constant health.


(b) Hygienic life is less costly than medical treatment.
(c) Man is mortal and likely to die at any moment.
(d) Happiness is always evasive.
(e) Health needs good care as we may fall victim to some sort of disease.
Q9. A doctor has succeeded in raising a variety of monkeys similar to
human beings. They clean their teeth with brushes and mop their face with
towels. [__________] In that case these animals may be called monkey –
man and we may be called man – monkeys.

(a) Perhaps they may learn and use human language.


(b) Sometimes they may die after a few days.
(c) The method adopted by the doctor is wonderful.
(d) We can expect a lot of change in the case of other animals also.
(e) Monkeys are good subjects unlike human beings.

Q10. It is wonderful how a word can acquire diagonally opposite meaning.


Gandhiji called the socially downcast people by the name Harijans.
[__________] But now the Harijans themselves feel that it is a word of
insult.

(a) Gandhiji was a social reformer.


(b) Gandhiji had many Harijan friends.
(c) Gandhiji expected to bring them respect by calling them the children of God.
(d) Harijans have risen to such a high position that they feel their brand name
‘Harijan’ to be degrading.
(e) Only Harijans are children of God.

Directions (11-15): Each of the following questions has a paragraph from


which the last sentence has been deleted. From the given options, choose the
one that completes the paragraph in the most appropriate way.

Q11. Lower winter temperatures were common in Europe during the


second half of the 17th century, famously allowing frost fairs to be held on
the frozen Thames in London before riverine developments increased the
flow rate. These cold winters coincided with the Maunder minimum in
solar activity when the Sun remained virtually free of sunspots for almost
50 years. However, establishing that this was not just a chance occurrence
requires that the relationship continue to hold over a long interval, such
that cold European winters become less frequent when solar activity is high
and then more common again when solar activity falls. Various indicators
show that during the recent minimum of the 11 year sunspot cycle, the Sun
has been quieter than at any time in the previous 90 years.

(a) This means that solar activity during the current sunspot minimum has fallen
to levels unknown since the start of the 20th century.
(b) This yields an opportunity for a better test of the relationship between solar
activity and cold European winters.
(c) This proves that cold winters occur more commonly in the UK during low
solar activity.
(d) This regional and seasonal effect relating to European winters may have a
global effect.
(e) None of the above

Q12. Debt is more common in families with disabled children: the parents
were unable to keep up with any local property taxes, water, and telephone
bills, and were not likely to be able to afford basic items such as a family
holiday once a year,a bicycle, or even two pairs of shoes. A disabled baby
needs more nappies. Families’ ability to work grows difficult, and finding
childcare is a real burden. Households with disabled childrenwill depend
more on social security benefits and are faced with the additional financial
costs associated with caring for a disabled child.

(a) There is a strong link between child disability and poverty.


(b) The highest prevalence of childhood disability is found in the poorest
families.
(c) It is an adverse and serious social gradient that families with disabled face.
(d) But thanks to science, these children live longer and medicines keep them
alive.
(e) None of the above

Q13. What a super film experience Green Zone is! From the firecracker
opening to the sucker-punch climax, the film is a non-stop adrenalin rush.
The hand-held camera and natural light make you feel as if you are seeing
the action from the front, as if you have access to footage shot from a
sniper’s sights. Whether it is a Bourne-in-Baghdad kind of relentless action
thriller or a strong statement against the U.S. war in Iraq, (incidentally, it
is both) Green Zone succeeds as a pure cinema, delivering thrills, spills and
chills in breathless succession hardly giving anyone time to breathe.

(a) This is a movie that takes you on a thrilling, provocative, exhilarating ride.
(b) There is really nothing more you could ask for from a movie.
(c) Green Zone effectively knits several strands together to make a cohesive
whole.
(d) The plot is taut and truthful.
(e) None of the above
Q14. Talented youth can ill-afford to resign to their fate just because they
can’t properly communicate in English. They should confront the
challenges which should, in fact, bring out their best. A little confidence
and hard work are all that is needed for them to climb up the career
ladder. For that they need to develop communication skills in English,
shape up their personalities and acquire the much-needed knowledge.

(a) Knowledge and communication skills are the key ingredients that make up
the recipe for success.
(b) Students have to act as leaders in the college itself.
(c) Success will automatically follow.
(d) Speaking and writing in English are important, thinking in English is twice
as important.
(e) None of the above

Q15. Philosophy of music has been dominated by the view that the best
music is autonomous and formally complex. As recently as 1990,
philosophy of popular music consisted of variations on a single theme.
Philosophers defended the twin assumptions that popular music is
essentially different from “serious” or art music, and that the former is
aesthetically inferior to the latter.

(a) As a result, music could not be regarded as art if it lacked genius and
autonomy.
(b) As a result, popular music competes with and replaces local and regional
folk traditions.
(c) As a result, most philosophers concentrated on identifying the aesthetic
deficiencies inherent in popular music.
(d) As a result, philosophers have investigated popular music by identifying and
critiquing key concepts that shape our response to this music.
(e) None of the above

Solutions
S1. Ans.(b)
Sol. There we can see a number of house boats waiting to be hired by the
tourists. This sentence is the continuation of the idea expressed in the first
sentence.
S2. Ans.(b)
Sol. We must take the children to get vaccinated.

S3. Ans.(b)
Sol. There they have a greater number of viewers. The next sentence also talks
about the viewers.

S4. Ans.(c)
Sol. It converts sunlight into electricity.

S5. Ans.(a)
Sol. Another source is the smoke emitted by vehicles. In previous sentence, first
source is talked about.

S6. Ans.(c)
Sol. Even police authorities are reluctant to face them.

S7. Ans.(d)
Sol. Some people are ready even to die for them.

S8. Ans.(b)
Sol. Hygienic life is less costly than medical treatment.

S9. Ans.(a)
Sol. Perhaps they may learn and use human language.

S10. Ans.(c)
Sol. Gandhiji expected to bring them respect by calling them the children of
God.

S11. Ans.(b)
Sol. “...establishing that this was not just a chance occurrence requires that the
relationship continue to hold over a long interval...” is the crux of the paragraph.
Hence the sun being “quiet” is an opportunity to find this correlation. Options
(C) and (D) can be very easily eliminated as not related to the purpose of the
paragraph. Option (A) is true, is an inference not related to the purpose of the
paragraph.

S12. Ans.(c)
Sol. This statement is a very low level inference that logically closes the
paragraph. The paragraph is not sufficient to establish the link as in option (A).
Option (B) goes farther away from the paragraph. Option (D) is unrelated to the
purpose of the paragraph.
S13. Ans.(b)
Sol. All options may appear correct. The scoring option, however, has to close
the paragraph, andnot merely continue it. Option (A) will be repetitive. Options
(C) and (D) will continue the paragraph.

S14. Ans.(c)
Sol. The purpose of the paragraph is in the first sentence - “can ill-afford to
resign to their fate.” Option (A) is already clearly stated-it just states the same
thing in different words. Option (B) takes off on a tangent and brings in
leadership; option (D) also does not close the paragraph. Option (C) just does
that-the paragraph has no loose ends.

S15. Ans.(c)
Sol. The purpose of the paragraph is: Philosophers consider popular and serious
music different. The former lacks complexity and autonomy, the later is
variations on a single them-and that popular music is inferior. “As a result” will
discuss its direct consequences and close the paragraph. Hence option (C)
scores. Option (A) is stated. Option (B) is irrelevant in “replace” and “folk
music”. Option (D) is irrelevant in “our response.”

PART-7

Directions (1-15): In the following passage, some of the words have been
left out, each of which is indicated by a number and a word is given for
each blank. If the given word is correct then choose appropriate option (E)
as correct answer, otherwise find the suitable word from the options given
against each number and fill up the blanks with appropriate words to make
the paragraph meaningfully complete.

WHAT is the sum of ……1……(physical) science? Compared with the


……2………… (commercial) universe and with ……3……(conceivable)
time, not to speak of infinity and eternity, it is the observation of a mere
……4…… (pointless), the experience of an ……5…… (instant). Are we
……6…… (quarantine) in founding anything upon such data, except that
which we are obliged to found upon them – the daily rules and processes
……7……(unnecessary) for the natural life of man? We call the discoveries of
science ……8……;(sublime) and truly. But the sublimity belongs not to that
which they ……9……(conceal) but to that which they suggest. And that which
they suggest is, that through this material glory and beauty, of which we see a
little and imagine more, there speaks to us a being whose nature is akin to ours,
and who has made our hearts capable of such converse. Astronomy has its
practical uses, without which man’s ……10……(intend) would scarcely rouse
itself of those speculations; but its greatest result is a ……11……(revelation)
of immensity pervaded by one informing mind; and this revelation is made by
astronomy only in the same sense in which the ……12……(stethoscope)
reveals the stars to the eye of the astronomer.
Science finds no law for the thoughts which, with her aid, are ministered to man
by the starry skies. Science can ……13……(disapprove) the hues of sunset,
but she cannot tell from what urns of pain and ……14……(pleasure) its
pensiveness is poured. These things are felt by all men, felt the more in
proportion as the mind is higher. They are a part of human nature; and why
should they not be as sound a basis for ……15……(philology) as any other
part? But if they are, the solid wall of material law melts away, and through the
whole order of the material world pours the influence, the personal influence, of
a spirit corresponding to our own.

Q1.
(a) spiritual
(b) materialistic
(c) corporal
(d) disbursal
(e) The given word is correct.

Q2.
(a) transient
(b) sumptuous
(c) incomplete
(d) comprehensible
(e) The given word is correct.

Q3.
(a) innuendo
(b) insurmountable
(c) achievable
(d) approachable
(e) The given word is correct.

Q4.
(a) point
(b) conjecture
(c) context
(d) coincidence
(e) The given word is correct.

Q5.
(a) intact
(b) constant
(c) contest
(d) command
(e) The given word is correct.

Q6.
(a) challan
(b) warranted
(c) guaranty
(d) plausible
(e) The given word is correct.

Q7.
(a) necessary
(b) obligatory
(c) unrequited
(d) dormant
(e) The given word is correct.

Q8.
(a) unfound
(b) profound
(c) supreme
(d) soliloquy
(e) The given word is correct.

Q9.
(a) subjective
(b) ill-gotten
(c) reveal
(d) illustration
(e) The given word is correct.

Q10.
(a) intellect
(b) intrepid
(c) insolent
(d) interpretation
(e) The given word is correct.

Q11.
(a) uninterested
(b) reflection
(c) remorse
(d) replete
(e) The given word is correct.

Q12.
(a) sphygmomanometer
(b) seismograph
(c) barometer
(d) telescope
(e) The given word is correct.

Q13.
(a) disdain
(b) explain
(c) certify
(d) dominate
(e) The given word is correct.

Q14.
(a) displeasure
(b) wonder
(c) disastrous
(d) supplement
(e) The given word is correct.

Q15.
(a) dermatology
(b) biology
(c) sociology
(d) philosophy
(e) The given word is correct.
Solutions

S1. Ans.(e)
Sol. The given word “physical” is correct.

S2. Ans.(d)
Sol. Comprehensible – is correct word that defines the nature of universe.

S3. Ans.(e)
Sol. The given word conceivable is correct. conceivable-capable of being
imagined or grasped mentally.

S4. Ans.(a)
Sol. ‘point’ is correct word that fits in the context of the paragraph correctly.

S5. Ans.(e)
Sol. The given word Instant- is correct.

S6. Ans.(b)
Sol. Warranted- justify or necessitate (a course of action). ‘Warranted’ is correct
word.

S7. Ans.(a)
Sol. ‘Necessary’ is correct word according the conveyed meaning in the
sentence.

S8. Ans.(e)
Sol. The word ‘sublime’ is correct, it means- sublime-of very great excellence
or beauty.

S9. Ans.(c)
Sol. Reveal -fits in the context correctly.

S10. Ans.(a)
Sol. Intellect- fits in the context correctly.

S11. Ans.(e)
Sol. Revelation is correct word.

S12. Ans.(d)
Sol. Telescope- is correct word in the given context.
S13. Ans.(b)
Sol. Explain- is correct word.

S14. Ans.(e)
Sol. Pleasure- the given word is correct.

S15. Ans.(d)
Sol. Philosophy is correct word.

PART-8

Directions (1-15): In each of the following questions, a part of the paragraph


or sentence has been Bold. From the choices given to you, you are required
to choose the one, which would best replace the Bold part in terms of
grammatical usage.

Q1. The tendency to eat late, though it has never been tested properly, many
nutritionists believe, as a factor in putting on weight.
(a) many nutritionists believe, to be a factor in putting on weight.
(b) many nutritionists believe factor in putting on weight.
(c) is believed by many nutritionists to be a factor in putting on weight.
(d) is believed by many nutritionists as a factor in putting on weight.
(e) None of the above replace the bold part.

Q2. On the billion or so Windows machines, some 63% still uses old XP version
launched in 2001, in America, the proportion of XP users are over 80%.
(a) some 63% still use the old XP version launched in 2001, in America the
proportion of XP users are
(b) some 63% still uses the old XP version launched in 2001; in America, the
proportion of XP users are
(c) some 63% still use the old XP version launched in 2001 in America, the
proportion of XP users is
(d) some 63% still use the old XP version launched in 2001; in America, the
proportion of XP users is
(e) None of the above replace the bold part.
Q3. Though subjected to varying influences, the distinctive features of China's
artistic activity possess a unity which is quite as complete as that of our
Western art.
(a) possess a unity which is quite as complete as that of our Western art.
(b) possesses a unity which are quite as complete as our Western art.
(c) possesses a unity which is quite complete like our Western art.
(d) possess a unity which is quite complete like our Western art.
(e) None of the above replace the bold part.

Q4. To know by parts is science, knowing the whole as a whole is philosophy.


(a) Knowledge of parts is science,
(b) Knowing by parts is science,
(c) To know partially is science,
(d) To know by parts is science,
(e) None of the above replace the bold part.

Q5. In the fission reactions that nuclear power generation rely on today, heavy
elements such as uranium breaks into smaller ones.
(a) relies on today, heavy elements such as uranium break into smaller ones.
(b) rely on today, heavy elements such as uranium break into smaller ones.
(c) relies on today, heavy elements such as uranium breaks into smaller ones.
(d) rely on today, heavy elements such as uranium breaks into smaller ones.
(e) None of the above replace the bold part.

Q6. The nature of our party system has changed in the past few decades, and the
lesson of the past decade had been that the party that did not hold the White
House had every incentive to refuse to co-operate with the president's party.
(a) was that the party that does not hold the White House had every incentive
(b) is that the party that does not hold the White House has every incentive
(c) has been that the party that had not held the White House had every incentive
(d) had been that the party that did not hold the White House had every incentive
(e) None of the above replace the bold part.

Q7. Looking at the path of German GDP and the fact that total private sector
hours didn't fall by that much, it becomes clear that Germany's unique
recession and recovery was about more than just its choice in labour market
policies.
(a) Looking at the path of German GDP and that total private sector hours didn't
fall much,
(b) Looking at the path of German GDP and the fact that total private sector hours
didn't fall much,
(c) When one looks at the path of German GDP and the fact that total private
sector hours didn't fall much,
(d) When one is looking at the path of German GDP and that total private sector
hours didn't fall much,
(e) None of the above replace the bold part.

Q8. The Beijing Language and Culture University Press, the largest publisher of
Chinese-language textbooks, says South Korea and Japan are their biggest
customers.
(a) say South Korea and Japan are their biggest customers.
(b) says South Korea and Japan are it's biggest customers.
(c) says South Korea and Japan are their biggest customers.
(d) says South Korea and Japan are its biggest customers.
(e) None of the above replace the bold part.

Q9. There are two pieces to Mr Keats' argument; that English is open-source
as other languages are not; and that is why it has spread.
(a) Mr Keats' argument, that English is open-source as other languages are not
and that
(b) Mr Keats' argument: that English is open-source as other languages are not,
and that that
(c) Mr Keats' argument - that English is open-source as other languages are not-
and that
(d) Mr Keats' argument; that English is open-source as other languages are not;
and that that
(e) None of the above replace the bold part.

Q10. The number of foreigners going to India for fertility


treatments, specially surrogacy, has dramatically increased in recent years.
(a) especially surrogacy, have dramatically increased in past years.
(b) especially surrogacy, has dramatically increased in recent years.
(c) specially surrogacy, have dramatically increased in recent years.
(d) specially surrogacy, have shown dramatic improvement in recent years.
(e) None of the above replace the bold part.

Q11. The LHC is a magnificent engineering project, who's many "gee-whiz"


features have been widely reported.
(a) whose many "gee-whiz" feature have been widely reported.
(b) who's "gee-whiz" features has been reported widely.
(c) whose many "gee-whiz" features has been widely reported.
(d) who's many "gee-whiz" features have been widely reported.
(e) None of the above replace the bold part.

Q12. Venezuelan President Hugo Chavez, outspoken as usual, compared the


choice of Liu for the Nobel Prize with Obama.
(a) compared the choice of Liu for Nobel Prize with that of Obama.
(b) compared the choice of Liu with Obama for the Nobel Prize.
(c) compared the choice of Liu for the Nobel Prize to that of Obama.
(d) compared the choice of Liu and Obama to the Nobel Prize.
(e) None of the above replace the bold part.

Q13. The OECD expects average GDP growth among their mostly rich
members to slow from 2.8% in 2010 and 2.3% in 2011, before bouncing back
to 2.8% in 2012.
(a) their most rich members to slow from 2.8% in 2010 and 2.3% in 2011,
(b) their mostly rich members to slow from 2.8% in 2010 and 2.3% in 2011,
(c) its most rich members to slow between 2.8% in 2010 to 2.3% in 2011,
(d) its mostly rich members to slow from 2.8% in 2010 to 2.3% in 2011,
(e) None of the above replace the bold part.

Q14. A sphere cannot be represented on a flat plane without distortion, which


will mean that all map projections will distort in one way or another.
(a) which means all map projections distort in one way or another.
(b) that means all map projections will distort one way or another.
(c) which will mean that all map projections distorts one way or other.
(d) which means all map projections will distort in one way or others.
(e) None of the above replace the bold part.

Q15. The blame for rising obesity rates have been pinned on many things,
including having more calorific diet, the spread of processed food, a lack of
exercise and modern man's generally more stressful lot.
(a) has been pinned on many things, including a more calorific diet.
(b) have been pinned to many things, including more calorific diets,
(c) have been pinned on many things, including a calorie rich diet,
(d) has been pinned on many things: having a more calorific diet,
(e) None of the above replace the bold part.

Solutions

S1. Ans.(c)
Sol. Answer Option C. There are two clauses in this sentence: The tendency to
eat late is believed (by nutritionists) to be a factor in putting on weight and
(though) it has never been tested properly. The first clause can also be correctly
constructed as "The tendency to eat late, many nutritionists believe, is a factor in
putting on weight." But, this is not among the options. "believed ... as a factor" is
incorrect.
S2. Ans.(d)
Sol. Answer Option D. Portions (percent, fractions etc...) the verb will agree with
the noun what follows the of. (50 of the people come from ...... 50% of the money
comes from ... etc) In this sentence the noun is machines; hence, the verb 'use' is
correct. When there is already a comma in any of the punctuation that replaces a
conjunction has to be a semicolon. In the above sentence, there is no conjunction.
Option C makes the sentence incorrect because the sentence is not saying
"launched in 2001 in America." "The proportion... "is the subject in the next
clause; hence, the verb has to be singular 'is'.

S3. Ans.(a)
Sol. Answer Option A. There is no error in the original sentence nor does it need
any improvement. 'features' needs a plural verb 'possess'. To establish comparison
'that of' is necessary.

S4. Ans.(b)
Sol. Answer Option B. Faulty parallelism in all the other options.

S5. Ans.(a)
Sol. Answer Option A. Pure subject-verb agreement question. The subject
'nuclear power generation' required the singular verb 'relies' and 'heavy elements'
requires the plural verb 'break'.

S6. Ans.(b)
Sol. Answer Option B. Tense inconsistency. The lesson of the past decade has to
be either "is' or 'was', hence had been is eliminated. The option with was A
continues the sentence into the present tense and is illogical. The consistent
present tense in option B makes the whole idea a fact. (simple present tense is
used to state facts)

S7. Ans.(c)
Sol. Answer Option C. Modifier error if 'it' is retained as the subject. 'it' cannot
be 'looking.' Option A and B are, hence, eliminated. Option D unnecessarily uses
the continuous tense, and has a parallelism error. When one looks at...... the path
and the fact are parallel. Hence option C is the only correct choice.

S8. Ans.(d)
Sol. Answer Option D. The verb 'says' and the pronoun 'its' are consistent with
the subject "The Beijing Language and Culture University Press".

S9. Ans.(b)
Sol. Answer Option B. A colon correctly introduces the list of
ideas. Semicolon in its place is incorrect. Dashes (option C) are incorrect because
the idea (between the dashes) is not parenthetical. The second 'that' is necessary
to maintain parallelism. The possessive of Keats can be either Keats' or Keats's.

S10. Ans.(b)
Sol. Answer Option B. Specially vs especially (Specially means exclusively for,
especially means - for a particular purpose); 'especially' suit the context better
because 'infertility treatments' are mentioned as that class of treatments the
tourists go for. Option C and D are eliminated. 'The number of foreigners...' is the
subject. "the number" is always singular, hence 'has' is the correct verb.

S11. Ans.(a)
Sol. Answer Option A. Who's vs whose-whose is the possessive of who. 'many
features' is plural, hence the verb has to be the plural 'have.'

S12. Ans.(c)
Sol. Answer Option C. Choice is compared, hence that of (choice of) is
essential. Otherwise choice and Obama are directly compared making it an error.
To identify either the similarities or the differences between two things, use
"compare to." To identify both the similarities and the differences (at the same
time), use "compare with." ('Compared with' places the things compared)
on equal level.) India is often compared with China. India is often compared to
the US. In comparing with something, one finds or discusses both things that are
alike and things that are different.

S13. Ans.(d)
Sol. Answer Option D. The OECD is singular (that is why the verb is 'expects'
rather than expect; the pronoun has to be consistent, hence its is necessary-their is
incorrect. 'mostly rich members' is correct, 'their most rich members' is vague.
'from' needs to be followed by 'to', not 'and'. Option D corrects these errors.

S14. Ans.(a)
Sol. Answer Option A. The shift to future tense is unnecessary from the present
'cannot be represented' as it is a fact. It is an error to change ‘which’ to ‘that’, as
‘which’ introduces a descriptive clause. 'one way or another' is the correct idiom.

S15. Ans.(a)
Sol. Answer Option A. Since the subject is 'the blame'-the verb has to be singular
'has'. 'pinned on' is correct idiom. 'A more calorific diet' is parallel with 'the spread
of...' and a lack of...' Hence option D is faulty parallelism.
PAT-9
Directions (1-7): In each of the following questions a related pair of words or
phrases is followed by four pairs of words or phrases. Select the pair that
best expresses a relationship that is least similar to the one expressed in the
question pair.

Q1. DELUSION: HALLUCINATION


(a) cheat: counterfeit
(b) sophism: deception
(c) eidolon: ghost
(d) phantom: reality
(e) None of the above is correct

Q2. JUVENILE: MATURE


(a) calumny: slander
(b) fallacy: verity
(c) exhortation: discourage
(d) solvency: debts
(e) None of the above is correct

Q3. SPINACH: PINK


(a) apple: red
(b) pear: blue
(c) turquoise: orange
(d) rose: green
(e) None of the above is correct

Q4. CABLE: ELECTRICITY


(a) wire: sound
(b) artery: blood
(c) lamp: oil
(d) canal: water
(e) None of the above is correct

Q5. EXOTIC: INDIGENOUS


(a) routine: extraordinary
(b) culture: refinement
(c) callow: mature
(d) tame: wild
(e) None of the above is correct
Q6. CLOTHES: BODY
(a) antimacassar: chair
(b) carpet: floor
(c) frame: picture
(d) paint: signboard
(e) None of the above is correct

Q7. ASININE: VACUOUS


(a) nefarious: villainous
(b) heinous: flagitious
(c) deciduous: evergreen
(d) synonymous: equal
(e) None of the above is correct

Directions (8-15): In each question, there are four sentences. Each sentence
has pairs of words/phrases that are italicized and highlighted. From the
italicized and highlighted word(s)/phrase(s), select the most appropriate
word(s)/phrase(s) to form correct sentences. Then choose the best option.

Q8. (i) The vacuum cleaner came with several complementary


[A]/complimentary [B] attachments for cleaning different surfaces.
(ii) In an age infatuated of machines, life becomes amoral [A]/immoral
[B], without moral bearings, devoid of moral categories.
(iii) The trainer mounted the horse and rode for some distance in order to dispel
[A]/expel [B] the fears of the child.
(iv) It was as if she had seen a ghost that her face had a deadly [A]/deathly
[B] pallor.
(a) ABAB
(b) AAAB
(c) BAAA
(d) BAAB
(e) BBBB

Q9. (i) When he was upset, one could see his juggler [A]/jugular [B] veins
standing out in his neck.
(ii) At this junction [A]/juncture [B] students are required to make careful
decisions about their future course.
(iii) He made a few unexceptionable [A]/unexceptional [B] propositions, all
quite relevant to the discussion.
(iv) As an architect she was involved in the development of urbane [A]/urban
[B] housing projects in the city.
(a) ABAB
(b) ABBA
(c) BAAA
(d) BBAB
(e) AAAA

Q10. (i) The speaker could not shout over the babble [A]/bauble [B] of the
crowd and he was not heard.
(ii) She felt bad [A]/badly [B] about having to lie to her friend; but she had no
choice.
(iii) The farmers were loading bails [A]/bales [B] of hay onto the truck.
(iv) The students waited for the answer to a question that has vexed them for
many days with baited [A]/bated [B] breath.
(a) ABAB
(b) AABB
(c) BABB
(d) BAAB
(e) BBBB

Q11. (i) On television, we have been witnessing the baneful [A]/baleful


[B] results of war as Israel attacks Hamas in Gaza.
(ii) The wrestler appeared to be tall and burly [A]/burley [B] as he stood by the
TV announcer.
(iii) Clouds smoke billowed [A]/bellowed [B] into the air as the fire engulfed the
building.
(iv) The old patriarch uttered a malediction [A]/benediction [B] against the rival
clan.
(a) BAAA
(b) AAAA
(c) ABAB
(d) AAAB
(e) BBBB

Q12. (i) Sudha Murthy’s beneficent [A]/beneficial [B] activities have resulted in
many organizations that help the poor.
(ii) She could not think of any earthly [A]/earthy [B] reason to go out with him
in such heavy rain.
(iii) His egoism [A]/egotism [B] prevented him from making practical decisions.
(iv) To go out of the building, you are requested to use the egress [A]/ingress
[B] behind the building.
(a) AABA
(b) BBBA
(c) BABB
(d) BBAA
(e) AAAA
Q13. (i) She demonstrated immanent [A]/imminent [B] good sense in her dress
and behaviour.
(ii) The thunder was heard immediately [A]/imminently [B] after they saw
lightning.
(iii) He went to the diary [A]/dairy [B] section to pick a few packets of milk.
(iv) Her nervousness was obvious with her forehead damp [A]/dank [B] with
perspiration.
(a) ABBA
(b) BBBA
(c) AABA
(d) BBAB
(e) BBBB

Q14. (i) In the internet age there is no dearth [A]/death [B] of uncensored,
firsthand information.
(ii) He would often digress [A]/regress [B] into his past and talk at length about
his childhood.
(iii) His small physique worked against him minimizing [A]/diminishing [B] his
authority as a CEO.
(iv) She had a very official [A]/officious [B] manner, always offering her services
even when she was not asked nor needed.
(a) ABAB
(b) ABBB
(c) BABA
(d) BAAB
(e) BBBB

Q15. (i) The newlywed couple was facilitated [A]/felicitated [B] by their friends
before they left the town.
(ii) If my girlfriend invited me to dinner I would feign [A]/fain [B] do it for sure.
(iii) We hung [A]/hanged [B] the painting on the wall.
(iv) The law has a structure [A]/stricture [B] against sale and possession of guns.
(a) BBAB
(b) ABAB
(c) BBAB
(d) AAAB
(e) AAAA
Solutions

S1. Ans.(d)
Sol. Answer option D, phantom and reality are antonyms; all the others including
the key pair are synonyms.

S2. Ans.(a)
Sol. Answer option A, the key pair is opposites. Option A is synonyms. calumny-
the making of false and defamatory statements about someone in order to damage
their reputation; slander.

S3. Ans.(a)
Sol. Spinach is not pink; hence the least similar is Apple and Red.

S4. Ans.(c)
Sol. Cable conducts electricity, lamp and oil do not have a similar relationship.

S5. Ans.(b)
Sol. The key pair is antonyms; option B is synonyms. Culture and refinement are
synonyms in applied meaning.

S6. Ans.(d)
Sol. Clothes protect the body; similarly, the first three options are related to each
other. Antimacassar is a piece of ornamented cloth that protects the back
of chair from hair oils.

S7. Ans.(c)
Sol. The key pair is synonymous. Option c is opposite.
asinine-extremely stupid or foolish. vacuous-having or showing a lack of
thought or intelligence; mindless.
deciduous-(of a tree or shrub) shedding its leaves annually.

S8. Ans.(b)
Sol. AAAB. Complementary fits the sentence because attachments cannot be
complimentary – given free. Without moral bearings etc. makes one amoral –
being neither moral nor immoral, rather than immoral. Dispel means to remove;
expel means to throw out. Deadly means dangerous; deathly means death-like.

S9. Ans.(d)
Sol. BBAB. Jugular is the main vein in the neck; juggler is a person who does
tricks. Junction is where two paths meet; juncture means an important point in
one’s development or history. Unexceptionable means unobjectionable;
unexceptional means ordinary/commonplace. Urbane means suave or
sophisticated; urban means related to the city.

S10. Ans.(b)
Sol. AABB. Babble means meaningless and loud talk; bauble means a
trinket/trifle. She felt bad because feel is a stative verb in the context; stative verbs
need to be followed by an adjective (bad) and not an adverb (badly). Bail as a
noun is a container; bale (noun) means a large bundle. Bated breath is the correct
idiom; baited (past tense of bait) means to entice or lure.

S11. Ans.(b)
Sol. AAAA. Baneful means poisonous, deadly or destructive; baleful means
sinister. Burly means strong and heavy build. Burley is a type of tobacco. Billow
means to rise in waves/spirals. Bellow is to make a loud and hollow sound.
Malediction means a curse; benediction means a blessing.

S12. Ans.(a)
Sol. AABA. Beneficent means performing acts of Kindness and charity;
beneficial is conducive to personal or social well-being. Earthly means possible;
earthy means practical. Egoism emphasizes self-interest; egotism means
conceited or proud. Egress is way out and ingress is a way in.

S13. Ans.(c)
Sol. AABA. Immanent means in-build; imminent is waiting to happen.
Immediately means directly or straightaway which correct in the context. Diary
is a record of events and dairy is where milk products are stored. Damp implies
slightly wet. Dank is unpleasantly wet (stinking)

S14. Ans.(b)
Sol. ABBB. Dearth means scarcity, hence correct. Digress means to stray away
from the topic; regress means to go back in time. Minimize means to keep to
minimum; diminish means to cause to appear less. Officious means volunteering
one’s services where they are neither asked nor needed or meddlesome.

S15. Ans.(c)
Sol. BBAB. Facilitate means to help bring about; felicitate means to offer
congratulations. Feign means to pretend , fain means willing, obliged, constrained
etc. The past tense of hang is hung and hanged; but hanged is used only in the
context of hanged to death and in no other. Stricture means restriction.
PART-10

Directions (1-15): A word has been used in the in four different ways in the
sentences that follows. Choose the option corresponding to the sentence in
which the usage of the word is incorrect or inappropriate.

Q1. Insert
A. The writer was asked to insert a change in the manuscript.
B. It is his habit to insert new topics in the discussion.
C. The country is planning to insert a new satellite into the orbit.
D. The ceremony was announced through an insert in the newspapers.
(a) A
(b) B
(c) C
(d) D
(e) None of the above is correct usage.

Q2. Write
A. The boy writes Sanskrit with ease.
B. The ailing old asked his attorneys to write his will.
C. He was asked to write in a cheque for the amount he owed.
D. He is type of man who writes his own ticket.
(a) A
(b) B
(c) C
(d) D
(e) None of the above is correct usage.

Q3. Scale
A. Thick scales covered the animal.
B. The government brought in policies to scale imports.
C. He scaled the fish by scraping it with a knife.
D. Hard water scales the boiler.
(a) A
(b) B
(c) C
(d) D
(e) None of the above is correct usage.
Q4. Make
A. The children intended to make a lot of fun during the picnic.
B. One should not make a disturbance near hospitals.
C. The housekeeping department makes beds twice a day.
D. It was difficult for the family to make ends meet.
(a) A
(b) B
(c) C
(d) D
(e) None of the above is correct usage.

Q5. Gain
A. The clock gains a minute per day.
B. The day is gaining warmth.
C. The trekkers gained the peak in the evening.
D. The amplifier helped the signal gain intensity.
(a) A
(b) B
(c) C
(d) D
(e) None of the above is correct usage.

Q6. Show
A. The excavation showed a staggering number of artefacts.
B. When they arrived a waiter showed them to the table.
C. She didn’t show up for her appointment.
D. The general meeting of the opposition party turned out to be a show of
strength.
(a) A
(b) B
(c) C
(d) D
(e) None of the above is correct usage.

Q7. Trade
A. The shop stocked novelties for the tourist trade.
B. The brothers found it beneficial to trade places in their business.
C. At this dealer, one can trade the old car on a new one.
D. This magazine has limited circulation as it is a trade publication.
(a) A
(b) B
(c) C
(d) D
(e) None of the above is correct usage.

Q8. Pay
A. It is not possible to pay someone like him to do that.
B. It is futile to think that crime pays.
C. You could make the call from a pay telephone.
D. Such a reckless act will mean that you have to pay the devil.
(a) A
(b) B
(c) C
(d) D
(e) None of the above is correct usage.

Q9. Act
A. Trees act against the wind.
B. Under cover of darkness the police waited to catch thief in the act.
C. His life is hard act to follow.
D. The legislature did not act on the bill.
(a) A
(b) B
(c) C
(d) D
(e) None of the above is correct usage.

Q10. Affect
A. The sight affected her to tears.
B. Our beliefs are affected by our upbringing.
C. Drops of water affect roundness.
D. He issued a statement to the affect that he would resign.
(a) A
(b) B
(c) C
(d) D
(e) None of the above is correct usage.

Q11. Bark
A. The street was noisy with the vendors barking their wares.
B. The rhinoceros is famous for its very thick bark.
C. You seem to be barking up the wrong tree in blaming your neighbour.
D. The bark of the cinnamon tree is used as a spice.
(a) A
(b) B
(c) C
(d) D
(e) None of the above is correct usage.

Q12. Cover
A. It appears as if the ruling coalition is trying to cover the scandal.
B. The new book on the history of science covers a lot of ground.
C. I am unwilling to cover for her in the meeting.
D. The project was a cover for intelligence operations.
(a) A
(b) B
(c) C
(d) D
(e) None of the above is correct usage.

Q13. Inform
A. The policy is based on the principles that inform bilateral relations.
B. As an expert musician, he is well informed with the keyboard.
C. Compassion for fellow beings informs all her novels.
D. The Prime Minster was informed about the crisis in the party.
(a) A
(b) B
(c) C
(d) D
(e) None of the above is correct usage.

Q14. Give
A. She gave birth last Thursday.
B. The students gave the SBI PO Exam last Sunday.
C. The professor freely gave of his time to the students.
D. As the time passed, optimism gave place to worry.
(a) A
(b) B
(c) C
(d) D
(e) None of the above is correct usage.

Q15. Pass
A. The throne passed to the king’s son.
B. The court passed on the legality of the wiretapping.
C. My experience of the Himalayas passed all expectations.
D. He is not the one to pass an opportunity for promotion.
(a) A
(b) B
(c) C
(d) D
(e) None of the above is correct usage.

Solutions

S1. Ans.(b);
Sol. Sentence B is better phrased with ‘introduce’ rather than ‘insert’. A means:
to put or thrust in. C means: to put or introduce into the body of something or
interpolate. In C, insert is a noun which means: something that is inserted, written
or printed material inserted (as between the leaves of a book).

S2. Ans.(c);
Sol. “write a cheque” or “write out a cheque” would correct the sentence. In A it
is used with the simple meaning of inscribing the characters. In B, it used with
the meaning daft or draw up. In D the idiom “to write one’s own ticket” means to
select a course of action entirely according to one’s wishes.

S3. Ans.(b);
Sol. This can be corrected by adding up/down–scale up/down imports–which will
mean adjust. As it stands the sentence is meaningless. In sentence A, scale is
noun. In sentences C and D scale is used as verb.

S4. Ans.(a);
Sol. The sentence can be corrected by substituting ‘have’ for make. Sentence B
has the meaning ‘create’. Sentence C has the meaning arrange. Sentence D uses
the idiom ‘make ends meet’ which means “to make one’s needs adequate to meet
one’s means”.

S5. Ans.(d);
Sol. “gain in intensity” would correct the sentence. Sentence A means the clocks
goes ‘faster’ by a minute every day. Sentence B means ‘increase’. Sentence D
means ‘to arrive at’, ‘reach’ or’ attain’.

S6. Ans.(a);
Sol. If ‘showed’ is replaced by ‘exposed’, the sentence would be correct. In
sentence B, it is used to means to conduct; guide. In sentence C, the idiom ‘show
up’ (informal) means: to make an appearance. Sentence d means: a display.

S7. Ans.(c);
Sol. The idiom ‘trade in’ means to ‘turn in as payment or part payment for a
purchase or bill.’ Hence sentence C can be corrected as …. Trade the old car in
on a new one. Or’ .. trade for a new one “in which case it will mean exchange.
Option A uses the word to mean market. Option B uses it to mean exchange.
Option D uses it as an adjective meaning ‘meant for industry’.

S8. Ans.(d);
Sol. The idiom “the devil to pay” means severe consequences; hence the sentence
is to be corrected to” … will mean that you have the devil to pay.” Option A
means to “hire”. Option B means “to be worth the expense or effort”. Option C,
pay telephone; or pay station means a public telephone–one uses a coin to make
a call.

S9. Ans.(a);
Sol. Sentence A is meaningless and as the intended meaning is not clear, we
cannot correct it either. Sentence B means when ‘in the process if’. Sentence C
means the sum of a person’s actions or effects that serve to create an impression
or set an example. Sentence D means to give a decision.

S10. Ans.(d);
Sol. Affect needs to be replaced with effect. In sentence A affect is used to mean
‘provided the stimulus’. In B, it means influence. In C, affect means ‘tend
toward.’

S11. Ans.(b);
Sol. Sentence B can be corrected by using ‘skin’ instead of bark. In sentence A,
‘barking their wares’ means advertising with persistent outcry. In sentence C, to
bark up the wrong tree (idiom) to misdirect one’s efforts, or to proceed under a
misapprehension. In sentence D, bark is used as noun to refer to the outer covering
of the tree stem.

S12. Ans.(a);
Sol. The sentence can be corrected by writing ‘cover up the scandal’ rather than
cover the scandal. Sentence B means ‘includes a lot of information’. Sentence C
means to substitute. Sentence D means a pretext.

S13. Ans.(b);
Sol. Sentence B will be corrected if instead of informed, acquainted is used. In
sentence A, ‘inform’ is used with the specific contextual meaning: to give
character or essence to something. Sentence C also has a specific contextual
meaning of ‘inform’ which is: to be the characteristic quality of. In D, it has the
common meaning ‘communicated the knowledge of.’

S14. Ans.(b);
Sol. Students do not ‘give the SBI PO’; they take the SBI PO. (Take tests not give
tests). In sentence A, gave birth is correctly used to mean ‘to have a baby.’ In C,
the idiom ‘give of’ means to make available or provide generously. In D’ give
place to’ means replace with or succeeded by.

S15. Ans.(d);
Sol. This can be corrected by making it” … not the one to pass up an
opportunity…” the idiom ‘pass up’ means to let go without accepting or taking
advantage of. Sentence A means to go from the control of one to another.
Sentence C means exceed or surpass.

PART-11

Directions (1-15): In each of the following questions a capitalized pair of


words is given, followed by five numbered pairs of words. Four of the
numbered pairs exhibit the same relationship between the words as the
capitalized pair of words. Identify the numbered pair which does not exhibit
the same relationship as the capitalized pair.

Q1. CHILD: PAEDIATRIC


(a) heart : cardiologist
(b) nerves : neurologist
(c) tumours : oncologist
(d) blood : haematologist
(e) eyes : gynaecologist

Q2. COLOURS : RANGOLI


(a) lights : diwali
(b) kites : pongal
(c) rakhi : baisakhi
(d) carols : Christmas
(e) colours : Holi

Q3. COOK : EAT


(a) hard word : success
(b) exam : result
(c) design : plain
(d) thirst : drink
(e) hunger : eat
Q4. PARSIMONIOUS : STINGY
(a) sumptuous : lavish
(b) indigent : wealthy
(c) niggardly : miserly
(d) puerile : juvenile
(e) penury : poverty

Q5. LECTURER : COLLEGE


(a) teacher : school
(b) peon : office
(c) student : theatre
(d) reader : university
(e) actor : movies

Q6. DENTIST : TOOTH


(a) aurist : ear
(b) trichologist : hair
(c) cardiologist : lungs
(d) dermatologist : skin
(e) optician : eye

Q7. CONSONANCE : DISCORD


(a) laud : criticize
(b) digest : divest
(c) conciseness : lengthiness
(d) aversion : affection
(e) like : dislike

Q8. INIMICAL : HOSTILE


(a) bounteous : abundant
(b) mellifluous : melodious
(c) lucid : clear
(d) tranquil : placid
(e) docile : obstinate

Q9. DICTATOR : TYRANNY


(a) patriot : rebelliousness
(b) philanthropist : benevolence
(c) altruist : selflessness
(d) miser : parsimony
(e) democrat : discussion
Q10. BOTANY : PLANTS
(a) ornithology : birds
(b) ichthyology : fish
(c) zoology : animals
(d) philology : politics
(e) psychology : behaviour

Q11. APPLE : FRUIT


(a) bat : insect
(b) tulip : flower
(c) dodo : bird
(d) whale : mammal
(e) snake : reptile

Q12. LION : CUB


(a) deer : fawn
(b) horse : foal
(c) cow : calf
(d) goose : gosling
(e) goat : nanny

Q13. IGLOO : ESKIMO


(a) nest : bird
(b) monastery : monk
(c) jail : accused
(d) nunnery : nun
(e) bee : hive

Q14. TEACHER : INSTRUCT


(a) actor : entertain
(b) soldier : defend
(c) moralist : preach
(d) judge : govern
(e) ruler : rule

Q15. CHURLISH : REFINE


(a) hinder : assist
(b) commodious : congested
(c) ambiguous : explicit
(d) digress : deviate
(e) radiant : brilliance
Solutions
S1. Ans.(e)
Sol. A doctor for children is paediatric, for heart is cardiologist, for
blood haematologist and for tumours oncologist and for nerves neurologist.
Choice (E) (a gynaecologist doesn’t treat the eye)

S2. Ans.(c)
Sol. Colors are a part of rangoli and holi, lights of Diwali, kites of Pongal and
carols of Christmas.
Choice (C) (Rakhi is not a part of Baisakhi)

S3. Ans.(c)
Sol. In order to eat we must cook, in order to succeed work hard, to get result take
an exam, to drink be thirsty and hungry to eat.

S4. Ans.(b)
Sol. The words in each pair are closely connected – parsimonious is stingy,
sumptuous is lavish, niggardly is miserly, puerile is juvenile. Indigent and
wealthy are opposites.

S5 Ans.(c)
Sol. A lecturer works in a college just as a teacher works in a school, a peon in
an office, a reader in a university and an actor in movies.
Choice (C) (a student does not work at a theatre)

S6. Ans.(c)
Sol. A cardiologist treats heart ailments hence choice (C) is wrong.

S7. Ans.(b)
Sol. The relationship between the capitalized pair of words is antonymous.
Choice (B) does not denote this relationship.

S8. Ans.(e)
Sol. The relationship between the capitalized pair of words is synonymous.
Choice (E) represents an antonymous relationship.

S9. Ans.(a)
Sol. Dictator symbolizes tyranny. Choice (A) is different because patriot does not
symbolize rebelliousness.

S10. Ans.(d)
Sol. All the other pairs of choices represent the study of living beings. Only choice
(D) is different.
S11. Ans.(a)
Sol. Choice (A) is different because bat is not an insect. It is a mammal.

S12. Ans.(e)
Sol. The young one of a goat is a lamb not nanny. Hence, choice (E) is different.

S13. Ans.(c)
Sol. Jail is not a dwelling place for the accused, hence choice (C) is different.

S14. Ans.(d)
Sol. The duty of a teacher is to instruct while the duty of a judge is to dispense
justice not govern unlike it is represented in choice (d).

S15. Ans.(d)
Sol. The relationship between the capitalized pair of words is antonymous.
Choice (D) does not denote an antonymous relationship.

Read more: http://www.bankersadda.com/p/as-we-all-know-sbi-po-is-here-and-


we_5.html#ixzz4aBtOrvdN

PART-12

Directions (1-5): In each of the following sentences, parts of the sentence


are left blank. Beneath each sentence, five different ways of completing the
sentence are indicated. Choose the best alternative from among the five
options.

Q1. Cars __________ to accommodate wheelchair users are vital to


disabled people to get out and about and enjoy __________ lives.
(a) designed, their
(b) modified, healthy
(c) adapted, active
(d) modified, itinerant
(e) adopted, itinerant
Q2. Some __________ defending, especially in the second half of the game
allowed the opposition to __________ and the defending champions lost the
game.
(a) slack, score
(b) superb, lose
(c) robust, win
(d) superb, score
(e) slack, lose

Q3. Print, paper, and newspapers enabled the rise of new types of
__________ based on expanded popular participation.
(a) class conflicts
(b) cultural forms
(c) social norms
(d) political systems
(e) ethical dilemmas

Q4. Businesses attempt to __________ the conflicts of interest between


groups to ensure __________ operation of the organization.
(a) compromise, profitable
(b) reconcile, efficient
(c) reconcile, commercial
(d) stimulate, efficient
(e) encourage, professional

Q5. One in three people on our planet have no __________ to modern


energy to light and heat the __________ in which they live.
(a) resource, homes
(b) right, houses
(c) contact, nations
(d) access, dwellings
(e) source, countries

Directions (6-15): Read the following passage carefully and choose the best
answer to each question out of the five given alternatives.

This industry preys on pestilence for profits. As the primary sector thrives under
the benevolent gaze of the rain gods, according to the CMIE forecast for
Business Today, the production of food grains will touch 187 million tons in
1994-95-up from 182 million tons in 1993-94-pesticides are likely to be a key
input in the country’s race for a quantum jump in agricultural productivity.
That’s because a not-insignificant 30 percent of the production of food grains in
the country is destroyed by insects, pests, plant pathogens, rodents, and birds
every year. And even though the per capita consumption of pesticides is
currently low-which is also a pointer to the industry’s potential – this country is
still the world’s third largest consumer of pesticides.
Classified by target species, pesticides can be divided into four broad
categories. Insecticides-like monocrotophos and fenvalerate – are used for
killing insects. Herbicides – such as butachlor and anilophos – remove weeds
and unwanted plants. Fungicides – like nickel chloride – kill fungi. And
fumigants and rodenticides – such as zinc and aluminium phosphide – are used
to kill rodents.
At present, insecticides contribute to almost 75 percent of the turnover of the
pesticides industry in value terms and 85 percent in terms of volume. This is at
variance with the trend in the West, where insecticides account for just 32
percent of pesticides consumption: it is herbicides and fungicides which account
for the largest share of consumption in those countries.
In terms of manufacturing technology, the production of pesticides can be
classified into two main categories; technical-grade materials and formulations.
Technical-grade-material-the basic chemical of high purity-is manufactured in
organized units, with the top 10 units accounting for more than 80 percent of
production. Most of them have a dominant market share in one or two key
products.
However, most pesticides are used as formulations, which are produced by the
processing of technical grade materials and are manufactured by both large and
small-scale units. In fact, the Insecticides Act of 1968 stipulates that 50 percent
of the production of technical-grade pesticides must be supplied by every
manufacturer to non-associated formulators.
While the stipulation aims at ensuring the sale of pesticides at cheaper prices,
arbitrary control has led to a conflict of interests. Formulators complain that
technical-grade manufacturers operate a cartel. The latter, however, claim that
rising input costs-raw materials constitute 60 percent of the selling price of
pesticides-are forcing hikes in selling prices.
The pesticides industry has over 80 registered technical-grade manufacturers
and about 800 registered formulators. About 160 formulators are associated
with technical-grade manufactures and boast of the advantage of being able to
obtain raw materials easily, even during the peak consumption season.
As the level of technology required is relatively low, formulators have low fixed
investment per unit of output. At 35 percent, the pesticides industry’s average
capacity utilization is rather low. And this, notwithstanding the 1974 ban
imposed by the government on the addition of formulation capacity. However,
firms can expand their formulation capacities so long as such expansions are
linked to the increased production of technical-grade material.
One of the main reasons for low capacity utilization in this industry is the
seasonal nature of the demand for pesticides. The maximum amount of sales is
recorded between July and November, which is reflected in the high inventories
that are built up in the first quarter of the year. As the active ingredient
deteriorates over time, a large number of formulations have a limited shelf-life.
At another level, the industry is characterized by the practice of credit sales to
the trade. These credits – which are typically for 60 to 90 days-coupled with the
high level of inventories-to cope with demand fluctuations-contribute to the
working capital-intensive nature of the industry. That’s why most manufacturers
have diversified, the most common diversification being pharmaceuticals. An
analysis of pesticides sales as a percentage of the total sales of the major players
confirms that most pesticides markers are well-diversified.

Q6. The growth in the production of food grains in 1994-95 over 1993-94 is
predicted to be roughly
(a) 187 million tons.
(b) 2 percent.
(c) 3 percent.
(d) 182 million tons.
(e) 7 billion

Q7. This country is the third largest consumer of pesticides in spite of


(a) a low per capita consumption of pesticides.
(b) 30 percent of production being destroyed by insects.
(c) our dependence on monsoons.
(d) food grain production being insignificant.
(e) poor technology.

Q8. The market for technical-grade pesticides in India is dominated by


(a) insecticide manufacturers.
(b) small scale sector.
(c) just ten units.
(d) large scale sector.
(e) is because there is no cartel of manufacturers.

Q9. The relationship between formulators and producers of technical-


grade material
(a) is determined by the market.
(b) is rather strong in India.
(c) depends on their end – use.
(d) is partly governed by law.
(e) the seasonal nature of demand.

Q10. The hike in selling prices


(a) is blamed by formulators on manufacturers.
(b) is the consequence of administered pricing.
(c) is caused by the Act of 1968.
(d) is because there is no cartel of manufacturers.
(e) our dependence on monsoons.

Q11. The percentage of formulators who can boast of being able to obtain
raw materials easily is
(a) 60
(b) 10
(c) 50
(d) 20
(e) 45

Q12. High inventories are built up during


(a) October, November, December.
(b) April, May, June.
(c) July, August, September.
(d) low per capita consumption.
(e) None of these

Q13. The pesticides industry is characterized by credit sales, the typical


credit is
(a) 30 percent of sales.
(b) 2 to 3 months.
(c) to the customers of technical-grade material.
(d) 100 percent of sale.
(e) 7 months

Q14. That the pesticides makers are well diversified is indicated by the
(a) dominance of pesticides sales in the total sales.
(b) reduction in pesticides production.
(c) analysis of pesticides sales as a percentage of total sales.
(d) hike in prices of pesticides.
(e) small scale sector.

Q15. Capacity utilization in the pesticides industry is low because of


(a) a ban on expansion.
(b) the seasonal nature of demand.
(c) poor technology.
(d) low per capita consumption.
(e) is determined by the market.
Solutions

S1. Ans.(c)
Sol. adapted, active. There is nothing in the context to justify healthy or
itinerant. option {A} ‘enjoy their lives’ is grammatically correct but logically it
does not justify “to get out and about” – hence active is justified. Cars can be
adapted, modified or designed. The best combination, however, is option {C}.

S2. Ans.(a)
Sol. slack, score. Fits in the context of the sentence correctly.

S3. Ans.(d)
Sol. political systems. ‘based on expanded popular participation’ and ‘print,
paper, and newspaper’ (media) control the word in the blank. political systems
relate to both, whereas all the other options are not necessarily related to one or
the other.

S4. Ans.(b)
Sol. reconcile, efficient. Conflicts need to be reconciled if operations are to be
efficient. Conflict of interests controls the first blank. Operations control the
second word. Some of the other options (e.g., option d) may make sense -but it
is not natural communication or logic, it is contrived sense.

S5. Ans.(d)
Sol. access, dwellings. ‘modern energy’ controls the first blank in the context,
‘in which they live’ and ‘heat and light’ control the next blank., hence
dwellings.

S6. Ans.(c)
Sol. 3 percent.

S7. Ans.(a)
Sol. a low per capita consumption of pesticides.

S8. Ans.(c)
Sol. The market for technical-grade pesticides in India is dominated by just ten
units.

S9. Ans.(d)
Sol. The relationship between formulators and producers of technical-grade
material is partly governed by law.

S10. Ans.(a)
Sol. The hike in selling prices is blamed by formulators on manufacturers.

S11. Ans.(d)
Sol. The percentage of formulators who can boast of being able to obtain raw
materials easily is 20.

S12. Ans.(b)
Sol. High inventories are built up during April, May, June.

S13. Ans.(b)
Sol. The pesticides industry is characterized by credit sales, the typical credit is
2 to 3 months.

S14. Ans.(c)
Sol. That the pesticides makers are well diversified is indicated by the analysis
of pesticides sales as a percentage of total sales.

S15. Ans.(b)
Sol. Capacity utilization in the pesticides industry is low because of the
seasonal nature of demand.

PART-13

Directions (1-15): Five alternative summaries are given below the text.
Choose the option that best captures the essence of the text.

Q1. The pursuit of a sustainable global society of low CO2 emitters requires a
tremendous effort. Precisely for this reason, it also requires a broadly shared
ethical basis. This would guide the negotiating parties in such a way that they
look not only for solutions to a part of the problem, but first and foremost at a
comprehensive solution to the entire problem. The climate change issue is too
important to be left in the care of politicians. It is therefore imperative that not
only nation states, but the business community and citizens combine their efforts
to save our planet's climate. That is not only a scientific necessity; it is an ethical
imperative.
(a) Since a global society of low CO2 emitters requires tremendous effort, it
requires a shared ethical basis on which a comprehensive solution can emerge. It
is imperative that not only politicians but business and citizens are involved to
save the planet’s climate.
(b) Tremendous efforts and a shared ethical basis are required
for sustainable global society of low CO2 emitters that can negotiate a
comprehensive solution to climate change. It is imperative that nations and
citizens work together to save the planet’s climate.
(c) The scientific and ethical imperative to find a comprehensive solution to
climate change is that there are combined efforts form politicians, business and
citizens to constitute a sustainable society of low CO2 emitters.
(d) A global society of low CO2 emitters requires tremendous effort, and a shared
ethical basis. It is a scientific and ethical imperative that not only politicians but
business and citizens are involved to save the planet’s climate.
(e) Since a global society of low CO2 emitters requires tremendous effort, it
requires a shared ethical basis so that a comprehensive solution can emerge. Since
politicians cannot be trusted, business and citizens must get involved in the efforts
to save the planet’s climate.

Q2. If foreign institutions are to be allowed at all, it is better that they operate
within an appropriate framework of regulation. If not, unscrupulous operators can
use the “foreign” tag to exploit poorly informed students who do not have the
scores to enter a good national institution or the finances to travel abroad to
acquire a good education. In an environment where good higher educational
facilities are in short supply, such operators could get away with charging high
fees for courses backed by inadequately qualified faculty,
inferior infrastructure and substandard equipment.
(a) If foreign universities are to be allowed, there should be regulated so that the
unsuspecting poor and the low scorers are not exploited by unscrupulous
operators.
(b) The operation of foreign universities must be regulated to prevent the
exploitation of students by using their foreign tag to charge high fees for poor
facilities.
(c) If the operation of foreign universities is not regulated they are likely to charge
the students high fees using their “foreign tag” and offer poor facilities in return.
(d) If foreign universities are to be allowed, their operations must be regulated;
otherwise, unscrupulous elements will use the foreign tag to charge high fees
even for poor facilities.
(e) Students who have low scores and those who cannot afford to study abroad
are likely to be exploited by foreign universities if these universities are allowed
to operate without regulations.

Q3. Nutrition is important to ensure proper brain formation and development,


which starts in the womb: development of the brain goes on during early
childhood. Iodine deficiency is known to affect a child’s Intelligence Quotient
(IQ) adversely. It has also been established that children with deficient growth
before age two are at an increased risk of chronic disease as adults, especially if
they gain weight rapidly in the later stages of childhood. A low birthweight baby,
who is stunted and underweight in its infancy and gains weight rapidly in
childhood and adult life, is much more prone to chronic conditions such as
cardiovascular disease and diabetes.
(a) Nutrition is important not only for the formation and development of the brain
in the womb, and for the intelligent quotient as a child, but also for the physical
well-being in adult life.
(b) Nutrition affects brain formation and development; iodine is related to IQ.
Low weight at birth and rapid weight-gain in childhood increases the risk of heart
disease and diabetes in adult life.
(c) Nutrition affects a child’s brain formation and development. Lack of iodine
causes low IQ, whereas low weight babies develop heart disease and diabetes in
later life.
(d) Starting in the womb, nutrition affects the child’s brain formation and
development. A low baby is more prone to develop cardiovascular disease and
diabetes if it gains weight during childhood.
(e) Nutrition is important for brain formation and development as well as for the
overall health. Iodine can affect the child’s IQ, and low weight at birth can lead
to chronic conditions in adult life.

Q4. When a species evolves traits that seem to have little to do with individual
survival-bright colours, say, or oversize horns, it is typically the male alone who
sports these excesses. Observing this, Charles Darwin proposed the idea of
“selection in relation to sex” as a follow-up to his theory of natural selection. He
defined it as the struggle between members of one sex, “generally male,” to
possess the other. The plumage of peacocks attracts peahens. The stag’s antlers
are there to fight off other stags and so on.
(a) The plumage of peacocks and the stag’s antlers, according to Charles Darwin,
have little to do with survival but are evolutionary male excesses to possess the
female.
(b) Bright colours or oversize horns, according to Charles Darwin, are
evolutionary excesses unrelated to individual survival but help the male in the
struggle to possess the female.
(c) Charles Darwin defined the plumage of peacocks and the stag’s antlers as
excesses in the struggle between members of one sex to possess the other.
(d) According to Charles Darwin, when a species evolves excesses that have
nothing to do with survival they are generally male and useful to fight off the
other male.
(e) In his follow up to natural selection Charles Darwin defined evolutionary
excesses as having little to do with survival and more to do with male domination
of the female.

Q5. Some people think sports cars are threatened with extinction by tightening
restrictions on carbon-dioxide emissions and unacceptable fuel-guzzling. They
fear the roar of the V8 will be replaced by the whirr of the electric armature – and
that motoring will never be the same again. Well, it ought to be quieter, that is
true. But they need not fear that electric cars will be slower. The secret is that
electric motors are better than combustion engines. They have more oomph, and
no need of a gearbox to deliver it.
(a) Contrary to what some people think sports cars will go even faster with electric
power.
(b) The belief that sports cars will become extinct is a misplaced one, as cars with
electric motors will go faster than combustion engines.
(c) The restrictions on carbon emissions need not make the sports cars extinct as
faster electric cars will easily replace them.
(d) Contrary to popular belief that electric cars are slower, they are in fact better
and faster than combustion engines.
(e) Though more noisy than the combustion engines cars with electric motors
deliver more power even without a gearbox; they will replace the combustion
engines.

Q6. From the beginning, the cloud hanging over the whole hydrogen enterprise
has not been the power source as such, but the intractable difficulty of distributing
and storing the stuff. It is not hard to see why. Hydrogen atoms are the smallest
and lightest in the universe. The next heaviest element in the periodic table, the
inert gas helium, is used for detecting cracks in pressure vessels and the like. Even
though helium atoms are four times chunkier than hydrogen atoms, they are still
small enough to find all the weak spots as they worm their way through the
crystalline structure of solid steel several centimeters thick. If hydrogen were
used as a crack detector, it would escape four times faster.
(a) Hydrogen enterprise is doomed to ultimate failure because hydrogen like
helium can be used as a crack detector.
(b) Storing and distributing hydrogen is difficult as it is made up of the smallest
and lightest atoms will leak through the weak spots even through walls several
centimeters thick.
(c) Hydrogen enterprise cannot succeed as storing and distributing it is a
formidable challenge because it can leak through even steel four times faster than
helium.
(d) As a power source the difficulty of hydrogen enterprise is the difficulty in
storing and distributing it because of its lightness.
(e) The difficulty that hydrogen enterprise faces is storing and distribution, as
hydrogen atoms are the smallest and the lightest and can escape through the
structure of even solid steel.

Q7. Criminals watch television too, and there is evidence they are also changing
their behaviour. Most of the techniques used in crime shows are, after all, at least
grounded in truth. Bleach, which destroys DNA, is now more likely to be used
by murderers to cover their tracks. The wearing of gloves is more common, as is
the taping shut – rather than the DNA-laden licking – of envelopes. Investigators
comb crime scenes ever more finely now for new kinds of evidence, which is
creating problems with the tracking and storage of evidence.
(a) Since criminals are also influenced by television shows, they change
their behaviour to leave fewer traces, forcing investigators to search for new
kinds of evidence.
(b) Criminals learn from television shows that bleach can destroy DNA and
have learnt to cover their tracks giving rise to the need for new kinds of evidence.
(c) The problem of having to search for new kinds of evidence arises from
criminals who watch television learning to cover their tracks better.
(d) Since criminals learn from television that bleach destroys DNA, wearing
gloves, and taping envelopes etc., cover their tracks, investigators now look for
new evidence.
(e) Since criminals are also influenced by the television shows they watch; they
have learnt to cover their tracks effectively posing a problem for investigators.

Q8. Storing energy is one of the biggest obstacles to the widespread adoption of
alternative sources of power. Batteries can be bulky and slow to charge.
Hydrogen, which can be made electrolytically from water and used to power fuel
cells, is difficult to handle. But there may be an alternative: magnesium. As school
chemistry lessons show, metallic magnesium is highly reactive and stores a lot of
energy. Even a small amount of magnesium ribbon burns in a flame with a
satisfying white heat. Researchers are now devising ways to extract energy from
magnesium in a more controlled fashion.
(a) Problems with the storage and handling of conventional energy sources have
forced the researchers to turn their attention to magnesium which stores a lot of
energy naturally.
(b) As hydrogen and dry cells pose great obstacles to storing energy, researchers
are looking at extracting energy from magnesium which stores a lot of energy.
(c) As storing energy is a big obstacle to the adoption of alternative sources of
power, researchers are looking at extracting energy from magnesium which stores
a lot of energy.
(d) Researchers are devising ways to extract energy form magnesium, as
magnesium stores a lot of energy compared to the bulky dry cells and hydrogen.
(e) Alternative energy researchers are now turning their attention towards
magnesium which stores a lot of energy and because dry cells and hydrogen are
difficult to handle.

Q9. Many preventive measures for cognitive decline and for preventing
Alzheimer’s disease – mental stimulation, exercise, and a variety of dietary
supplements – have been studied over the years. However, an independent panel
convened this week by the National Institutes of Health determined that the value
of these strategies for delaying the onset and/or reducing the severity of decline
or disease hasn’t been demonstrated in rigorous studies. Alzheimer’s disease is a
feared and heart-breaking disease, we wish we could tell people that taking a pill
or doing a puzzle every day would prevent this terrible disease, but current
evidence doesn’t support this.
(a) The National Institutes of Health has determined that the strategies for
delaying, reducing the severity of Alzheimer’s disease hasn’t been demonstrated
in rigorous studies.
(b) Alzheimer’s disease cannot be prevented, delayed or its severity reduced by
medication or through mental exercises.
(c) There is no evidence whether Alzheimer’s disease can be prevented, delayed
or its severity reduced by pills or by doing puzzles.
(d) An independent panel convened by the National Institutes of Health has
determined that medication or solving puzzle does not affect Alzheimer’s disease.
(e) An independent panel convened by the National Institutes of Health has
reported that it has found insufficient evidence to support preventive measures
for Alzheimer’s disease.

Q10. The mass production of optical mice has made the highly sophisticated
sensors on which they rely very inexpensive. Additionally, advances in
electronics and optics have yielded sensors that are both small and extremely
precise. A generic optical mouse, costing only a few dollars, is capable of
capturing and comparing surface images several thousand times per second.
Often, this high resolution enables their use on a variety of surfaces-
both traditional and ad hoc (e.g., palms, pants, bed covers).
(a) Advances in electronic and optics have facilitated the use of highly
sophisticated sensors for high precision, low cost, multi-surface optical tracking
in generic optical mice.
(b) The mass production of optical mice has made the optical sensors
inexpensive; sensors are now cheap, precise and can be used on any surface.
(c) Mass production and advances in electronics and optics have yielded sensors
that are cheap, high resolution, and capable of being used on any surface.
(d) Advances in technology and mass production have made the generic optical
mice capable of high precision, low cost, and multi-surface optical tracking.
(e) Advances in technology and mass production have made the generic optical
mice sophisticated devices capable of being used on different surfaces.

Q11. Heating and squishing microalgae in a pressure-cooker can fast-forward the


crude-oil-making process from millennia to minutes. University of Michigan
professors are working to understand and improve this procedure in an effort to
speed up development of affordable bio-fuels that could replace fossil fuels and
power today’s engines. They are also examining the possibility of other new fuel
sources such as E. coli bacteria that would feed on waste products from previous
bio-oil batches.
(a) Professors at Michigan University are working to understand and improve the
making of affordable bio-fuels to replace fossil fuels and then convert the waste
further into fuel.
(b) University of Michigan professors are working towards pressure-cooking
algae into bio-fuel and seeking to use E. coli to further convert the waste into bio-
fuel.
(c) University of Michigan professors are working towards pressure-cooking
algae into bio-fuel and seeking to recycle the waste into new source material for
future fuel batches.
(d) University of Michigan professors are working towards converting
microalgae into bio-fuel and seeking to recycle the waste into new source
material for future fuel batches.
(e) None of the above

Q12. Whales seem to stir up strong feelings. For conservationists, the majestic
mammals have been in urgent need of protection ever since factory ships began
slaughtering them in the middle of the last century. But advocates of whaling
present themselves as protectors of traditional culture, diets, and the rights of
indigenous people. It is difficult to find any common ground, even when an
honest attempt is made.
(a) Advocates present traditional culture, diets, and people’s rights, and
conservationists cite the need for protection for and against whaling.
(b) It is difficult to find any common ground in the arguments of conservationists
and advocates of whaling – except that both are emotional.
(c) Whales stir up strong feelings in conservationists as well as advocates of
whaling – with valid and irreconcilable reasoning on both sides.
(d) Other than strong feelings the arguments of people who are for and against
whaling are irreconcilable.
(e) None of the above

Q13. Although new battery technologies are emerging, their weight and
size is likely to remain a drag on the development of electric and hybrid cars,
forcing manufacturers to come up with new and inventive ways to shed weight
and free up space. One solution which researchers are exploring is to build cars
using a hybrid material: a carbon composite that is also capable of storing
electrical energy. That way, car designers could combine structural form with
electrical function.
(a) The manufacturers and designers of electric and hybrid cars are constrained
by the weight and size of the batteries; hybrid material seems to be the only
solution.
(b) Since manufacturers and designers are constrained by the bulkiness of the
batteries, researchers are exploring hybrid material capable of storing electrical
energy.
(c) By building cars with multifunction material, researchers are exploring a
solution to the weight and size of batteries which are a drag on the development
of electric and hybrid cars.
(d) The structural form and function of electric and hybrid cars can be the solution
to the bulkiness of the batteries that constrain both the manufacturers and the
designers.
(e) None of the above

Q14. The earth’s volcanoes appear for the most part in three types of setting. The
most familiar, and most of the most dangerous, are found where one tectonic plate
overrides another, as happens in the ring of fire around the Pacific. Then there are
those which sit over isolate “hotspots” of upwelling magma from deep in the
earth, like the volcanoes of Hawaii. Finally, there are those – a great many, but
normally deep under the ocean – formed at the spreading ridges where tectonic
plates pull away from each other and new crust is formed. Iceland is peculiarly
volcanic because it is formed by the intersection of a hotspot and a mid-ocean
ridge.
(a) The earth’s volcanoes exist in three settings: where one tectonic plate
overrides another, over hotspots of upwelling magma, and under the oceans
where plates are moving apart.
(b) There are three types of volcanoes: the most dangerous ones over the tectonic
plates, over the isolated hotspots of upwelling, and on ocean beds at the ridges of
tectonic plates.
(c) Volcanoes are of three types: where one tectonic plate overrides another, over
hotspots of magma upwelling from within the earth, and ones at the spreading
ridges where tectonic plates pull away from each other.
(d) The earth’s volcanoes appear in three types: where one tectonic plate overrides
another, over hotspots of upwelling magma, and where plates are moving apart.
(e) None of the above

Q15. It sounds implausible. Roboticists have struggled for decades to understand


bipedal locomotion, and even today’s most sophisticated robots require huge
amounts of energy and computer power to walk on two legs. But Dr Herr's
credentials are sound. He is a leading authority on the biomechanics of legs, and
in the past decade he has made several advances in the development of artificial
legs and assistive walking devices, or “orthoses,” enabling amputees to walk with
a more natural gait than was previously possible.
(a) It is an unbelievable task that Dr. Herr has achieved in developing assistive
walking devices, or “orthoses”; roboticists are still far away from understanding
bipedal locomotion.
(b) Even when robots consume huge computing power and energy to
walk, Dr Herr’s “orthoses,” enable amputees to walk with a more natural gait.
(c) Dr Herr’s “orthoses” or assistive devices for amputees challenge the
roboticists who have failed to simulate bipedal motion without consuming huge
computer power and energy.
(d) Dr Herr has been successful in developing “orthoses” enabling amputees to
walk with a natural gait; even sophisticated robots haven’t come close to natural
bipedal locomotion.
(e) None of the above

Solutions

S1. Ans.(b)
Sol. The following are the highlights of the paragraph: … sustainable global
society of low CO2 emitters requires a tremendous effort. … shared ethical basis.
…a comprehensive solution … politicians. … Nation states … business
community and citizens …scientific necessity; it is an ethical imperative. Except
for not specifying the politicians, business etc. Option (B) captures everything (it
states nations and citizens – which can accommodate all). The other options on
careful examination can be seen to distort something or the other.

S2. Ans.(d)
Sol. Option (A) has the vague “poor” in it. Option (B) misses out the “if they are
to be allowed part” which is significant to the précis. Option (C) states “they are
likely to charge” – the paragraph mentions only the unscrupulous elements may
do this. Option (E) assumes only students with low scores etc., will go to these
universities. Option (D) is the best choice, with the least problems.

S3. Ans.(a)
Sol. The paragraph is about how nutrition is important for child’s mental and
physical wellbeing – this begins in the womb, and continues into adult life. This
essence is captured by option (A). The other options highlight the specifics of the
paragraph. Though not incorrect, they struggle to include the details and miss the
“essence” of the paragraph.
S4. Ans.(b)
Sol. Options (A) and (C) specifically explain the plumage of peacocks and the
stag’s antlers – these are examples for bright colours or oversize horns and such
other traits. Hence, though correct the purpose of the paragraph is general. Option
(D) is also alright except that it does not specify why the males fight. Option (E)
is incorrect in “male domination of the female.”

S5. Ans.(c)
Sol. Only option (C) mentions “restrictions on carbon emissions” and includes
what other options elaborate on.

S6. Ans.(e)
Sol. “ultimate failure” eliminates option (A). No mention of “hydrogen
enterprise” eliminates option (B). “cannot succeed” eliminates option (C). Option
(D) in comparison to (E) does not explain “lightness.” Option (E) is brief and to
the point.

S7. Ans.(a)
Sol. The main points are Criminals watch television … changing
their behaviour … (examples) cover their tracks … for new kinds of evidence …
problems with the tracking and storage of evidence. The last point is not
mentioned in any of the options. Option (A) has these points and does not distort
any of the points. Option (B) emphasizes bleach etc., rather than the gist. Option
(C) is correct but loses out top option (A) because of phrasing – option (A) is
better phrased. Option (D) is similar to option (B). Option (E) does not specify
the problem. In comparison, option (A) is the best.

S8. Ans.(c)
Sol. Option (A) completely misses the point in “conventional sources” – the
paragraph is about “alternative sources.” Option (B) is correct but emphasizes the
problem with hydrogen and dry cells whereas the first sentence states the problem
explicitly as storing – hydrogen and dry cells are examples. Option (D) misses
the “alternative energy” part. Option (E) “difficult to handle” is vague; and the
emphasis has shifted as in option (B).

S9. Ans.(e)
Sol. Option (A), (B) and (C) are either erroneous or incomplete as they omit the
independent panel. Between option (D) and (E), it is an easy choice in favour of
option (E) as option (D) is too general and vague.

S10. Ans.(d)
Sol. Option (A) misses mass production. Option (B) misses “advances in
technology.” Option (C) confuses sensors with mice. Option (D) does not distort
the message. Option (E) misses high resolution, inexpensive etc.

S11. Ans.(c)
Sol. Option (A) does not mention the microalgae part. Option (B) is unnecessarily
specific about E. coli-the paragraph states “such as E. coli bacteria.” Option (D)
is correct but does not tell us how the algae can be converted into bio fuel which
option (C) does. Option (C) best captures the essence.

S12. Ans.(c)
Sol. Option (C) is closes to the given paragraph. Option (A) leaves out the
feelings part which is central to the paragraph. Option (B) is a miscommunication.
The paragraph talks about strong feelings forestalling a compromise – the purpose
is not to suggest that they are merely emotional. Option (D) is meaningless – one
cannot make sense of it, especially in the light of the paragraph.

S13. Ans.(c)
Sol. Option (B) would have been the best option if it had mentioned “hybrid
material to build cars.” As it stands the summary states they are exploring hybrid
material which is incomplete. “…only solution” eliminates option (A). Option
(D) is a sadly inadequate précis and unclear.

S14. Ans.(a)
Sol. According to the paragraph volcanoes exist in three settings; there is no need
to interpret this as three types of volcanoes – there may even be many
types within these settings. Though the other options are not as concise
as option (A), they also have other deficiencies missing out on important part like
under the ocean etc. Option (A) captures the essence of the paragraph.

S15. Ans.(d)
Sol. Option (A) states “far away from understanding bipedal locomotion” – it is
the computing power and energy that are the issues, hence this is a distortion.
Option (C) is eliminated because of “challenge the roboticists…” which is
incorrect. Between options (B) and (D), option (D) scores over (B) only for the
reason that “even sophisticated robots haven’t come close to natural bipedal
locomotion” is better than “robots consume huge computing power and energy to
walk” in the light of the purpose of the paragraph – the energy factor only
reinforces the difficulty of mastering “natural bipedal locomotion.”
PART-14

Directions (1-15):In the following questions, you are required to identify and
assess the logical Relationship between a given pair of words in the question,
then choose a pair of words from the options that exhibit the same logical
relationship as the original pair in the questions.

Q1.NECROMANCER : WITCHCRAFT
(a) puritan : excellence
(b) gourmand : starvation
(c) swindler : knavery
(d) lawyer : graft
(e) None of the above

Q2.EXTRACT : TOOTH
(a) cut : nail
(b) uproot : stump
(c) pull out : pin
(d) cut : wood
(e) None of the above

Q3.HUG : AFFECTION
(a) dour : humous
(b) nod : assent
(c) virus : influenza
(d) feet : walk
(e) None of the above

Q4.ISLAND : ARCHIPELAGO
(a) shoulder : armpits
(b) stomach : intestine
(c) shoe : laces
(d) toe : foot
(e) None of the above

Q5.CODICIL : WILL
(a) book : chapter
(b) outhouse : premises
(c) annexure : book
(d) limb : extremity
(e) None of the above

Q6.GETS ON : BUS
(a) hopes : fear
(b) enters : account book
(c) disembarks : airplane
(d) boards : rowboat
(e) None of the above

Q7.MUMBLE : INDISTINCT
(a) scribble : illegible
(b) screech : gentle
(c) swagger : timid
(d) sneeze : deliberate
(e) None of the above

Q8.FOOTBALL : SPORT
(a) rhythm : poetry
(b) verse : literature
(c) dancing : ballet
(d) research : biology
(e) None of the above

Q9.SAILOR : LIGHTHOUSE
(a) snake : hiss
(b) air raid : siren
(c) car horn : driver
(d) sleeper : smoke alarm
(e) None of the above

Q10.SIMMER : BOIL
(a) glide : drift
(b) gambol : play
(c) drizzle : downpour
(d) stagnate : flow
(e) None of the above

Q11.MITIGATE : PUNISHMENT
(a) change : order
(b) place : placement
(c) monotonous : change
(d) alleviate : pain
(e) None of the above

Q12.SAPLING : TREE
(a) puppy : dog
(b) canine : feline
(c) cat : lion
(d) poodle : terrier
(e) None of the above

Q13.TYRE : RUBBER
(a) oasis : desert
(b) house : roof
(c) paper : wood
(d) pebble : boulder
(e) None of the above

Q14.ANGLER : BAIT
(a) mouse : mousetrap
(b) hunter : decoy
(c) language : code
(d) treasure : map
(e) None of the above

Q15.BACTERIUM : COLONY
(a) microbe : disease
(b) whale : school
(c) shoal : fish
(d) virus : immunization
(e) None of the above

Solutions

S1. Ans.(c)
Sol. A necromancer is one who practices witchcraft, while a swindler practices
knavery.

S2. Ans.(b)
Sol. Just as you extract a tooth, you uproot a stump.

S3. Ans.(b)
Sol. The way a hug shows affection; in the same manner a nod shows the
expression of approval or agreement.
S4. Ans.(d)
Sol. Islands are parts of an archipelago, just as a toe is a part of a foot. (A part to
whole analogy)

S5. Ans.(c)
Sol. A codicil is an addition to a will, just like an annexure is an addition to a
book.

S6. Ans.(d)
Sol. We get on a bus, just as we board a rowboat.

S7. Ans.(a)
Sol. A mumble is indistinct, while a scribble is illegible.

S8. Ans.(b)
Sol. Football is a type of sport, just as verse is a type of literature.

S9. Ans.(d)
Sol. A sailor is warned by a light house, just as a person who is sleeping is warned
by a smoke alarm.

S10. Ans.(c)
Sol. This is a degree of intensity analogy. Simmer is a milder form of boiling just
as a drizzle is a milder form of a downpour.

S11. Ans.(d)
Sol. Just as we mitigate (lessen) punishment, we alleviate (lessen) pain.

S12. Ans.(a)
Sol. A sapling is a baby tree, just as a puppy is a baby dog.

S13. Ans.(c)
Sol. A tyre is made from rubber, while paper is made out of wood. In other words,
rubber is a raw material for tyre as wood is a raw material for paper.

S14. Ans.(b)
Sol. An angler uses a bait to attract fish, as a hunter uses a decoy.

S15. Ans.(b)
Sol. A colony of bacterium, as a school of whales.
PART-15

Directions (1-10): For each of the words given below, a contextual usage is
provided. From the alternatives given, pick the word that is the most
inappropriate as a substitute in the given context and mark its number as
your answer.

Q1. Rebuff: The party president virtually rebuffed the party’s state unit
president.
(a) snubbed
(b) repudiated
(c) spurned
(d) cold shouldered
(e) pampered

Q2. Abrogation: The party supported the abrogation of certain clauses of the
constitution.
(a) repudiation
(b) revocation
(c) annulment
(d) desiccation
(e) cancellation

Q3. Vestigial: Any vestigial feeling of affection he had for her was extinguished
by her outrageous actions.
(a) flourishing
(b) surviving
(c) lingering
(d) residual
(e) remaining

Q4. Exacerbate: Such nuclearisation would only exacerbate the tension between
the two neighbouring countries.
(a) inflame
(b) augment
(c) aggravate
(d) ameliorate
(e) increase
Q5. Enjoins: The doctrine enjoins upon the government to protect the resources
for the enjoyment of the general public.
(a) appeals to
(b) entreats
(c) implores
(d) spurs
(e) forces

Q6. Allay: The scheme needs to be popularized through persuasion, allaying the
genuine concerns of the affected parties.
(a) alleviating
(b) palliating
(c) clarifying
(d) dispelling
(e) easing

Q7. Equitably: The gift of nature has to be equitably distributed among human
beings.
(a) impartially
(b) fairly
(c) dispassionately
(d) equally
(e) justly

Q8. Imperative: Minimizing human exposure to radiation is imperative.


(a) obligatory
(b) compulsory
(c) perfunctory
(d) mandatory
(e) essential

Q9. Embroil: The organization has been embroiled in this controversy since last
month.
(a) mired
(b) ensnared
(c) engulfed
(d) involved
(e) entangled

Q10. Moot: The moot questions that now need to be answered is whether this
system would help the bank to recover its dues.
(a) unresolved
(b) contestable
(c) debatable
(d) disputed
(e) dubious

Directions (11-15): Four sentences are given with a blank in each. Five words are
also given. The blank in each sentence can be filled by one or more of the four
words given. Similarly, each word given in the choices can go into any number
of sentences. Identify the number of sentences each word can go into and mark
as your answer the maximum number of sentences any word can go into.

Q11. A. They had to __________ off the picnic as it was raining.


B. She went to the __________ box to speak to her mother.
C. Can you please __________ a taxi?
D. __________ me up at eight 8 O’clock, I have an appointment at nine.
(a) ring
(b) book
(c) call
(d) phone
(e) arrange

Q12. A. Do you __________ any musical instrument?


B. The team was ready to __________ in the interschool match.
C. Children love to __________ in the park.
D. He tried to __________ for time while his friend got away.
(a) enjoy
(b) compete
(c) stall
(d) play
(e) learn

Q13. A. She works so hard that she often falls asleep __________ the newspaper.
B. She will soon get __________ at them.
C. Anyone who is __________ eighteen can vote in India.
D. Though honest, he is not __________ cheating at cards.
(a) above
(b) over
(c) below
(d) on
(e) back

Q14. A. The children had picnic on the __________ of the river.


B. He keeps all his valuables in the __________ locker.
C. Can I __________ on you to support my claim?
D. The car is running round the __________.
(a) bank
(b) depend
(c) edge
(d) shore
(e) rely

Q15. A. They couldn’t get along, as they were on different intellectual


__________.
B. The __________ taking off from the airport was a beautiful sight.
C. It was __________ that he was guilty.
D. There were several policemen in __________ clothes.
(a) planes
(b) plain
(c) clear
(d) civil
(e) levels

Solutions

S1. Ans.(e)
Sol. ‘Rebuff’ means an abrupt rejection and so it means snubbed, repudiated
(refuse to accept) spurned (reject with disdain) cold-shouldered (refuse to talk or
notice someone). It does not mean ‘pampered’.

S2. Ans.(d)
Sol. ‘Abrogation’ means repeal or do away with. It can mean cancellation,
repudiation (refusal to accept), revocation (ending the validity of a decree)
annulment (declare a contract invalid). It cannot mean desiccation (dry out).

S3. Ans.(a)
Sol. ‘Vestigial’ means very small remnant of something and so surviving
(continue to live), lingering (reluctant to leave), residual (remaining part) it can
mean remaining. It cannot mean flourishing (develop vigorously), which is
almost the opposite.

S4. Ans.(d)
Sol. ‘Exacerbate’ means make worse and hence in the context inflame
(aggravate), augment (increase) aggravate (make worse) are same. It does not
mean ameliorate (make better), which is the opposite.
S5. Ans.(e)
Sol. ‘Enjoins’ means urges and hence can mean spurs, appeals to (requests),
entreats (asks earnestly) implores (begs earnestly) but not forces (which implies
compulsion).

S6. Ans.(c)
Sol. ‘Allaying’ means diminishing or ending and so it means easing, alleviate
(make less severe), palliate (make less severe), dispel (remove). It cannot mean
clarified (make clear).

S7. Ans.(d)
Sol. ‘Equitably’ means fairly and impartially and so impartially, fairly and
dispassionately (nationally, impartially), and justly. It does not
necessarily have be to equally.

S8. Ans.(c)
Sol. ‘Imperative’ means of vital importance. Hence, it can mean essential,
obligatory (compulsory) compulsory (cursive) and mandatory (required by law).
It cannot mean perfunctory (carried out with minimum of effort).

S9. Ans.(c)
Sol. ‘Embroiled’ means involved deeply in a conflict. Hence, it means entangled,
mired (meet with difficulties) ensnared (caught in a trap) involved (connected).
It does not mean engulf (surrounded).

S10. Ans.(e)
Sol. ‘Moot’ means subject to debate, dispute and so it means contestable
debatable, disputable and unresolved (not yet solved). It does not mean dubious
(doubtful).

S11. Ans.(d)
Sol. Phone fits in the sentence B and D correctly and maximum times.

S12. Ans.(d)
Sol. Play -fits in the given sentences maximum times and correctly defines the
conveyed meaning.

S13. Ans.(b)
Sol. over -fits in the sentence A, B, and C correctly and maximum times.

S14. Ans.(a)
Sol. ‘bank’ fits in the sentence A, B, and C correctly and maximum times.
S15. Ans.(b)
Sol. Plain- fits in the context correctly maximum times in sentences A, C and D.

PART-16

Directions (1-10): In each of the following questions, five statements


conveying the same idea age given. Select the one which expresses the idea in
the most concise manner and mark its number as your answer. Please note
that all of them may be grammatically correct and mean the same but you
have to select the most concise way of expressing the idea.

Q1.
(a) People who are resilient accept reality the way it comes, they firmly believe
that life has a meaning, and it these people who have an uncanny ability to
improvise.
(b) Resilient people accept reality and believe that life is meaningful and have an
uncanny ability to improvise.
(c) Resilient people are those who accept reality, firmly believe that life is
meaningful and these people have an uncanny ability to improvise.
(d) Resilient people besides accept reality firmly believe that life has a meaning
and it is they who have an uncanny ability to improvise.
(e) Resilient people are these who accept in reality that life is meaningful when
we firmly believe that uncanny ability must be improvised.

Q2.
(a) It is one of modern India’s greatest achievements that the caste system has
been reversed to a significant extent, in favour of those people who belong to the
oppressed classes of the Indian society.
(b) One of the greatest achievements of modern India is that the caste system has
been reversed to a significant extent, favouring those who belong to the oppressed
classes of the Indian society.
(c) One of the greatest achievements of modern India is, the caste system has been
reduced significantly in favour of the people belonging to the oppressed classes
of the society.
(d) It is one of modern India’s greatest achievements that the caste system has
been reduced significantly, in favour of the oppressed.
(e) One of the great achievements of modern India is that the caste system has
reversals to a significant extent, favouring those who belong to the oppressed
classes of the Indian society.

Q3.
(a) Politicians talk about how important it is to keep abreast of the rapidly
changing times, though, it is most often, these very same politicians who rarely
practice what they preach.
(b) Politicians frequently talk about how important it is for an individual to keep
abreast of the rapidly changing times, though it is most often they who rarely
practice what they preach.
(c) Politicians talk about how important it is to keep abreast of changing times,
though they rarely practice what they preach.
(d) Politicians talk about how important it is to keep abreast of the changing times,
though it is most often they who rarely practice what they preach.
(e) Politicians talk about how important it is to keep abreast in changing times,
though they rarely practice what they preach.

Q4.
(a) In this modern age which is driven by technology, e-mails and SMS have
taken away the need to written letters but printers are still having the last laugh.
(b) In a technology driven age, when e-mails and SMS have taken away the need
to write letters, printers are still having the last laugh.
(c) Modern age is being driven by modern technology and e-mails and SMS have
taken away the need to write letters but printers are still having the last laugh.
(d) In this modern age which is being driven by technology, e-mails and SMS
have taken away the need to write letters, but printers are still having the last
laugh.
(e) In an age which is driven by technology, needs of e-mails and SMS are so
high that there is no need to write letters.

Q5.
(a) The champions of the local breeds of cows believe that there is a conspiracy
to eliminate those breeds of cows which are developed locally to the advantage
of foreign bred cows, as they depend on foreign feed and are susceptible to
diseases acquired in foreign countries.
(b) Those people who champion for the local bred cows see a conspiracy to
eliminate locally bred varieties of cows, to the advantage of foreign bred cows
which depend on foreign feed and are susceptible to foreign diseases.
(c) Those who champion for the local breeds of cows, see a conspiracy to
eliminate those varieties of cows which are locally bred, to the advantage of
foreign-bred cows which depend on foreign feed and are susceptible to foreign
diseases.
(d) The champions of the local breeds of cows see a conspiracy to eliminate
indigenous varieties to the advantage of foreign-bred cows, which depend on
foreign feed and are susceptible to foreign diseases.
(e) The champions of the local breeds of cows believes that there is a conspiracy
to eliminate these breeds of cows which are locally developed to the advantage
of foreign bred cows, as they depend on foreign feed and are susceptible to
diseases acquired in foreign countries.

Q6.
(a) A temple, a mosque, a church or any religious place of worship, is capable of
being attacked by terrorists because it is difficult to have a security system that
prevents such attacks in places of worship.
(b) A temple, mosque or church is capable of being attacked by terrorists because
it is difficult to have a security that is prone to terrorist attacks.
(c) Almost all religious places are vulnerable to terrorist attacks for it is difficult
to have a fool proof security system for places of worship.
(d) Almost all religious places are vulnerable to terrorist attacks because it is
difficult to have a security system that is prevents terrorist attacks in places of
worship.
(e) Almost all religion places are vulnerable to terrorist attacks because it is
difficult to have a security system that prevents terrorist attacks in places of
worship.

Q7.
(a) In the Indian context, emigration to foreign countries had been a frequently
occurring process even before India was colonized by the British, when it was for
the purposes of carrying out trade and the spreading of religion.
(b) In the Indian context, emigration has been a continuous process since pre-
colonial times when it was for the purposes of trade and the propagation of
religion.
(c) In the Indian context emigration to foreign countries has been a continuous
process since pre-colonial times when it was for the purposes of trade to
popularize religion.
(d) In the Indian context, emigration to foreign countries has been a frequently
occurring process since pre-colonial times when it was for the purposes of trade
and the speeding of religion.
(e) In the Indian context to emigration to foreign countries a frequently occurring
process even before India was colonized by the British, for the only purpose of
carrying out trade and to spread religion.

Q8.
(a) The adaptable nature of the people of Indian origin and their initiative to play
a significant role, in the country in which they have originated, have made them
a force to reckon with for both the country to which they have emigrated and the
country in which they have originated.
(b) The Indian diaspora desire to play a significant role in the country in which
they have originated and their adaptable nature have made them an important
force to reckon with for not only the country to which they have emigrated but
also the country in which they have originated.
(c) The adaptability of the Indian diaspora, and their nature to initiate to play a
significant role in the country of origin have forced them to reckon with and for
the host country and the country of origin.
(d) Being adaptable by nature and possessing the desire to play a significant role
in the country in which they have originated, have made the Indian diaspora a
force to reckon with for both the country which they have emigrated and the
country in which they have originated.
(e) The adaptable nature of the Indian diaspora and their initiative to play a
significant role in the country of origin have made them a force to reckon with
for both the host country and the country of origin.

Q9.
(a) From the earliest times the English had been charmed by the glory of Greek
and Latin literature consequently they had overloaded their system of education
with classical studies.
(b) From the earliest times in the history of England the English people had been
charmed by the glory of Greek and Latin literature and in consequence they had
overloaded their system of education for children by adding a heavy load of
classical studies.
(c) Charmed by the glory of Greek and Latin literature, the English people and
overloaded their system of education for children by adding a heavy load of
classical studies, for the earliest times in the history of England.
(d) The English people were so fascinated by the glory of Greek and Latin
literature that they overloaded their system of education for children by adding a
heavy load of classical studies, from the earliest times in the history of England.
(e) Charmed by the glory of Greek and Latin literature, the English are overloaded
in their system of education as they have added a heavy load of classical studies,
from the earliest times in the history of England.

Q10.
(a) Ever since liberalization has come into existence people do not hesitate to
show off their wealth publicly.
(b) From the time liberalization has come into existence the desire to publicly
display one’s wealth without any hesitation has become stronger among people.
(c) Since the advent of liberalization, the hesitation to display one’s wealth
publicly has virtually vanished.
(d) From the time liberalization has come into existence the hesitation to display
one’s wealth publicly has virtually vanished among people.
(e) Liberalization has prevented the display of one’s wealth publicly without
hesitation.

Directions (11-15): In each of the following questions, a statement and five


choices are given. Select from among the choices, the most logical
complement and mark its number as your answer.

Q11. Religious fanaticism driving people to terrorize people of other religions


inside the country is
(a) as dangerous as terrorism coming from outside the country.
(b) natural and widespread.
(c) needed as a counter offensive.
(d) used to protect religious minorities.
(e) convenient to terrorists.

Q12. The past year, believe it or not, was surprisingly good.


(a) There were many terrorist attacks.
(b) The world economy was sluggish.
(c) How we feel about something depends on what we are expecting.
(d) In many countries unemployment rose.
(e) A positive mind shuns negative thoughts.

Q13. It is a measure of the nation’s skewed political priorities that bandhs are
evaluated in terms of whether they are partial or total.
(a) People enjoy a holiday.
(b) Political parties try to gain mileage.
(c) School children get a chance to keep away their home work for one more day.
(d) Daily wage earners get a rest.
(e) Its extraordinary cost both human and financial tends to get overlooked.

Q14. A war against Iraq looks probable.


(a) Poverty in India will be diminished.
(b) Meditation is a good antidote for stress.
(c) The tiger is a protected species.
(d) It could send oil prices soaring.
(e) Prices will soar high if there is a war.

Q15. Soya-the proven disease-fighter-appeals to the palate these days.


(a) Soya beans are now grown in India.
(b) Soya is rich in protein.
(c) Supermarkets now carry flavored soya milk, soya chunks etc, that look and
taste good.
(d) Soya is hundred percent vegetarian.
(e) School authorities are encouraging their students to drink soya milk.

Solutions

S1. Ans.(b)
Sol. Sentence B is the most concise and grammatically correct sentence among
the given sentences. All other Sentences are wordy and have different, faulty
structures. In sentence E, pronoun ‘these’ is incorrect.

S2. Ans.(d)
Sol. Sentence D is the most concise and grammatically correct sentence among
the given sentences. All other Sentences are wordy.

S3. Ans.(c)
Sol. Sentence C is the most concise and grammatically correct sentence among
the given sentences. All other Sentences are wordy.

S4. Ans.(b)
Sol. Sentence B is the most concise and grammatically correct sentence among
the given sentences. All other Sentences are wordy. Option E doesn’t mention
about printers, hence eliminated.
‘In a technology driven age’ this phrase makes the choice correct and easier to
find out.

S5. Ans.(d)
Sol. Sentence D is the most concise and grammatically correct sentence among
the given sentences. All other Sentences are wordy.

S6. Ans.(c)
Sol. Sentence C is the most concise and grammatically correct sentence among
the given sentences. All other Sentences are wordy.

S7. Ans.(b)
Sol. Sentence B is the most concise and grammatically correct sentence among
the given sentences.
S8. Ans.(c)
Sol. Sentence C expresses the idea in the most correct and concise way.

S9. Ans.(a)
Sol. Sentence A expresses the idea in the most correct and concise way.

S10. Ans.(c)
Sol. Sentence C expresses the idea in the most correct and concise way.

S11. Ans.(a)
Sol. Option A complement the idea expressed in the question correctly.

S12. Ans.(c)
Sol. Only option C correctly explain further (logically) the idea expressed in the
question sentence.

S13. Ans.(e)
Sol. Option E logically connects to the question sentence and explain it further.

S14. Ans.(d)
Sol. A war in Iraq can affect the prices. Hence, option D is logical complement
of the question sentence.

S15. Ans.(c)
Sol. The story expressed in the question sentence is about Soya. Option C
correctly explains further the Soya phenomena.

Read more: http://www.bankersadda.com/p/new-pattern-english-questions-for-


sbi_23.html#ixzz4aBurJbuj

PART-17
Directions (1-15): Rearrange the following sentences in the proper
sequence to form a meaningful paragraph; then answer the questions given
below them.

(A) One evidence of this is that many highly intelligent people are very low on
creativity.
(B) In other words they may find replicating a process very easy but creating a
process difficult.
(C) But it is unfortunate that in many organisations these replicates are rated
high and promoted over creators.
(D) Intelligence and creativity are two different things.
(E) They may understand the things very fast but are poor if any new thing is to
be developed; this in long term gives creators an upper edge.

Q1. Which of the following will be the SECOND sentence?


(a) A
(b) B
(c) C
(d) D
(e) E

Q2. Which of the following will be the FIFTH sentence?


(a) A
(b) B
(c) C
(d) D
(e) E

Q3. Which of the following will be the FOURTH sentence?


(a) A
(b) B
(c) C
(d) D
(e) E

Q4. Which of the following will be the FIRST sentence?


(a) A
(b) B
(c) C
(d) D
(e) E

Q5. Which of the following will be the THIRD sentence?


(a) A
(b) B
(c) C
(d) D
(e) E

(6-10):
(A) The percentage of population below poverty line has been decreasing.
(B) Therefore, it is a welcome sign, but we must guard against our escalating
rate of population growth.
(C) Poverty alleviation is one of the most significant programmes.
(D) Although this change is slow and gradual, it appears to be consistent.
(E) The extent of success of this programme can be sensed when we study the
proportion of people below poverty line.

Q6. Which sentence should come SECOND in the paragraph?


(a) A
(b) B
(c) C
(d) D
(e) E

Q7. Which sentence should come FOURTH in the paragraph?


(a) A
(b) B
(c) C
(d) D
(e) E

Q8. Which sentence should come FIFTH in the paragraph?


(a) A
(b) B
(c) C
(d) D
(e) E

Q9. Which sentence should come THIRD in the paragraph?


(a) A
(b) B
(c) C
(d) D
(e) E

Q10. Which sentence should come FIRST in the paragraph?


(a) A
(b) B
(c) C
(d) D
(e) E

(11-15):
(A) It is not even because grubbing for roots is good discipline.
(B) Moreover, he cannot learn what he ought to know about language from
talking about his own.
(C) Nor is it because they will gain satisfaction in recognizing the Latin roots of
the word satisfaction.
(D) It is because they cannot understand their own language unless they have
studied another.
(E) The reason is not so that they can sell things to the Brazilians, or study
German medical books or appreciate those beauties of Homer that are lost in
translation.
(F) Our citizens will have to learn at least one foreign language.
(G) The native of any country is immersed in his own language and never sees
it as a linguistic structure.

Q11. Which of the following will be the FIRST sentence?


(a) C
(b) A
(c) F
(d) G
(e) D

Q12. Which of the following will be the SECOND sentence?


(a) E
(b) C
(c) D
(d) B
(e) G

Q13. Which of the following will be the LAST sentence?


(a) D
(b) F
(c) C
(d) B
(e) A

Q14. Which of the following will be the FOURTH sentence?


(a) G
(b) B
(c) D
(d) E
(e) A

Q15. Which of the following will be the SIXTH sentence?


(a) C
(b) G
(c) F
(d) E
(e) B

Solutions

Sol- 1-5: The correct arrangement of sentences is: DABCE


D- This is the opening line of the paragraph which lays the idea that the paragraph
is about Intelligence and creativity.
A- D and A are mandatory pairs. Sentence A gives the idea that Intelligence and
creativity are different and sentence A is evidence of the given fact.
B- ‘they’ refers to highly intelligent people from sentence A.
C- B and C are mandatory pairs.
E- This is the concluding line of the paragraph

S1. Ans.(a)

S2. Ans.(e)

S3. Ans.(b)

S4. Ans.(d)

S5. Ans.(b)

Sol- 6-10:The correct arrangement of sentences is: CDBEA


C- This is the opening line of the paragraph which establishes the idea of Poverty
alleviation
D- C and D are mandatory pairs. Here ‘this change’ refers to poverty alleviation.
B- B and D are mandatory pairs.
E-This sentence further elaborates about the success of this programme.
A- This is the concluding sentence of the paragraph.

S6. Ans.(d)

S7. Ans.(e)

S8. Ans.(a)

S9. Ans.(b)

S10. Ans.(c)
Sol-11-15: The correct arrangement of sentences is: FECADGB
F- This sentence established the idea that the paragraph is about learning
the foreign language.
E-F and E are mandatory pairs.
C-A- sentence C and A follows sentence E respectively citing reasons for the
same.
D- This sentence gives the reason to learn the foreign language.
G- This sentence supports the reason given in previous sentence D.
B- This is the concluding line of the paragraph.

S11. Ans.(c)

S12. Ans.(a)

S13. Ans.(d)

S14. Ans.(e)

S15. Ans.(b)

PART-18

Directions (1-4): In each of the following sentences, a part of the sentence is


underlined/bold. Beneath each sentence, five different ways of phrasing the
underlined/bold part are indicated. Choose the best alternative among the
five.

Q1. Carbon-14 dating reveals that the artifacts recovered at Mesopotamia are
nearly 2,000 years as old as any of their supposed European predecessors.
(a) supposed older than any of their
(b) as old as their supposedly
(c) as old as their supposed
(d) older than any of their supposed
(e) None of the above
Q2. Unlike a typical automobile loan, which requires a fifteen to twenty-
percent down payment, the easy drive loan customer is not required to make
an initial deposit on the new vehicle.
(a) with easy drive loan buying there is no requirement of
(b) easy drive loan customers are not required to make
(c) for the easy drive loan customer there is no requirement of
(d) an easy drive loan does not require the buyer to make
(e) None of the above

Q3. In addition to having more protein than wheat does, the protein in rice is
higher quality than that in wheat, with more of the amino acids essentials to the
human diet.
(a) the protein in rice is higher quality than that in
(b) rice has protein of higher quality than that in
(c) the protein in rice is higher in quality than it is in rice
(d) rice protein is higher in quality than it is in
(e) None of the above

Q4. Had realized how close I was to failing, I would not have taken the SBI PO
exam in the second week of December.
(a) Had I realized how close
(b) When I realized how close
(c) If I would have realized
(d) If I realized earlier how close
(e) None of the above

Directions (5-8): In each question, there are pairs of words/phrases that


highlighted. From the highlighted word(s)/phrase(s), select the most
appropriate word(s)/phrase(s) to form correct sentences. Then, from the
options given, choose the best one.

Q5. So a blue-ribbon panel has made clear suggestions, and two respected
senators have proposed them as law-this seems nitpicking (A)/straightforward
(B) enough.

But the suggestions involve an intensive (A)/extensive (B) shake-up that will be
far from easy to make reality. Most notably, the 9/11 commission proposed two
new entities.

The first is a National Counterterrorism center, building (A)/countering (B) on


the Terrorist Threat Integration Center that was established after the September
11th attacks.
Whereas the current center amounts to a clearing house for terrorism-related
information, the proposed one would set operational (A)/functional
(B) priorities for fighting terrorism and be responsible for the results.
(a) ABBA
(b) BBAA
(c) BAAA
(d) BAAB
(e) BBBB

Q6. Pointless regulations foster (A)/abet (B) graft.

The more meddlesome (A)/irksome (B) the rule, the greater the incentive to
bribe officials not to enforce it.

An excellent new study by the World Bank, “Doing Business in 2005”, shows
that red tape is one of the chief obstacles (A)/stepping stones (B) to growth in
almost all poor countries.

The World Bank estimates that if a country in the worst-regulated quartile were
to join the best quartile, it would promote (A)/boost (B) its annual growth rate
by 2.2 percentage points.
(a) BAAA
(b) ABBA
(c) BABA
(d) ABAB
(e) AAAA

Q7. The change in personnel-rarely a good sign-seemed to presage (A)/prequel


(B) a change in strategy.

Mr. Kerry made two phone calls soliciting (A)/seeking (B) advice
from Mr Clinton, who was lying in a hospital’s bed awaiting quadruple bypass
surgery.

Mr Clinton apparently (A)/probably (B) told Mr Kerry to concentrate more on


the economy and to step up his attacks on Mr Bush.
That has not stopped John Edwards being warned (A)/praised (B) on the
campaign trail by loyalists “They’re going to run you right over and make you
look like idiots.
(a) ABAB
(b) BBAA
(c) AAAA
(d) AABA
(e) BBBB

Q8. In other words, there is probably much more to gain from promoting
(A)/slashing (B) red tape than from begging for more aid.

Especially since donors, unlike air travelers in Angola, are not exactly queuing
up (A)/ freewheeling (B) to open their wallets.

Mr. Putin said after Beslan that “we showed ourselves to be weak, and the weak
get beaten.” The implication (B)/ indication (B) is that he will now be even
tougher in Chechnya.

Not only is that likely to entertain (A)/stir up (B) more terrorism; it also ignores
one of the conflict’s main drivers, which is cast.
(a) AABA
(b) AABB
(c) BABA
(d) BAAB
(e) BBBB

Directions (9-15): In each of the questions, choose the meaning of


phrase/idiom (given in bold).

Q9. President Saddam Hussein’s effort to annex Kuwait was nothing short
of setting the Thames on fire.
(a) A try to achieve an impossible distinction
(b) To destroy the country for nothing
(c) To show his power and might
(d) To make a situation worse
(e) None of these

Q10. One should not stay idle at home; but be up and doing.
(a) Sleeping
(b) Active
(c) Working
(d) Reading
(e) None of these

Q11. Mr Arjun Singh snapped his fingers at Narasimha Rao and got himself in
hot water.
(a) To support
(b) To disregard
(c) To deprive
(d) To interfere
(e) None of these

Q12. We are afraid you may not be led astray in by Arvind’s bad company.
(a) Get into trouble
(b) Misguided
(c) Killed
(d) Lose the job
(e) None of these

Q13. To cry wolf


(a) To give false alarm
(b) To turn pale
(c) To ruin over self
(d) To overcome someone
(e) None of these

Q14. To be in dribs and drabs


(a) Without fail
(b) With much ease
(c) In small quantities
(d) With great difficulty
(e) None of these

Q15. A hard nut to crack is


(a) One who is very obstinate
(b) Very difficult child
(c) A walnut or a fruit
(d) Problem that is hard be solved
(e) None of these

Solutions

S1. Ans.(d)
Sol.’ older than any of their supposed’ in the question sentence, the comparison
is shown. Hence option D is correct in the given context.

S2. Ans.(d)
Sol.’ an easy drive loan does not require the buyer to make’ this is correct word
order according to the given context and express the idea completely and
correctly. Other options have faulty word structures.
S3. Ans.(b)
Sol.’ rice has protein of higher quality than that in’ this is correct word order and
express the idea clearly.

S4. Ans.(a)
Sol. ‘Had I realized how close’. This is the case of inversion. In this case, we use
verb before the subject.

S5. Ans.(b)
Sol. straightforward, extensive, building and operational are correct words that
express the conveyed idea appropriately.

S6. Ans.(d)
Sol. foster -encourage the development of (something, especially something
desirable).
irksome -irritating; annoying. The word ‘boost’ in sentence D gives us the hint to
mark the correct answer.

S7. Ans.(c)
Sol. presage-be a sign or warning of (an imminent event, typically an unwelcome
one).
solicit-ask for or try to obtain (something) from someone.

S8. Ans.(d)
Sol. slashing -vigorously incisive or effective.
implication- the conclusion that can be drawn from something although it is not
explicitly stated. The phrase ‘stir up’ in last sentence hints us towards correct
option.

S9. Ans.(a)
Sol. set the Thames on fire. To do wonderful or exciting things; to cause a great
or remarkable sensation in the world; to be extremely exciting, popular, famous,
renowned, etc.

S10. Ans.(b)
Sol. up and around; up and doing. Active again, especially after an illness or rest

S11. Ans.(b)
Sol. snap one's fingers at- Treat with contempt, scorn, disregard,

S12. Ans.(b)
Sol. lead someone astray- to direct or guide someone in the wrong direction
S13. Ans.(a)
Sol. To cry wolf-to cry or complain about something when nothing is really
wrong.

S14. Ans.(c)
Sol. in dribs and drabs -in small portions; bit by bit

S15. Ans.(d)
Sol. A hard nut to crack -a difficult problem to solve

PART-19

Directions (1-5): Four sentences are given with a blank in each. Five words
are also given. The blank in each sentence can be filled by one or more of the
four words given. Similarly, each word given in the choices can go into any
number of sentences. Identify the number of sentences each word can go into
and mark as your answer the maximum number of sentences any word can
go into.

Q1. A. What is the __________ value of the shares?


B. The __________ was released by the minister.
C. There is no need to __________ rooms, as it is offseason now.
D. The __________ for food items is always good.
(a) book
(b) market
(c) reserve
(d) product
(e) capital

Q2. A. The party was held at the __________ side.


B. They decided to __________ their resources.
C. I was shocked to see him lying in a __________ of blood.
D. The __________ room was over crowded.
(a) waiting
(b) combine
(c) pool
(d) lake
(e) share

Q3. A. I tried to __________ him off with my old PC.


B. The __________ tree is the life of the desert people.
C. What is the __________ today?
D. Do you __________ him regularly?
(a) palm
(b) date
(c) meet
(d) day
(e) see

Q4. A. The two brothers quarreled __________ themselves.


B. The sea lies __________ France and England.
C. This custom still exists __________ certain tribes.
D. The internet is __________ the most remarkable of modern inventions.
(a) between
(b) among
(c) with
(d) along
(e) in

Q5. A. Tickets for the __________ were booked months in advance.


B. They pushed hard but the door wouldn’t __________.
C. Better to keep quiet than __________ your ignorance.
D. “__________ me your tickets,” the inspector demanded.
(a) programme
(b) show
(c) display
(d) give
(e) play

Directions (6-15): A word has been used in four different ways in the
sentences that follows. Choose the option corresponding to the sentence in
which the usage is incorrect or inappropriate.

Q6. Deal
(a) He has a great deal of support from the people.
(b) His father deals in diamonds.
(c) Deal with this complaint the way you feel fit.
(d) She was asked to deal three books to each student.
(e) None of the above
Q7. See
(a) I can’t understand what she sees in him.
(b) I can still see through my daughter as she was years ago.
(c) The last fifty years have seen sweeping changes in technology.
(d) She could not see the task through as she fell ill.
(e) None of the above

Q8. Know
(a) Millions in our country have known poverty for generations.
(b) She knows to write well.
(c) We do not yet know the solution to this problem.
(d) Many top politicos were in the know of the scam for a very long time.
(e) None of the above

Q9. Look
(a) She is looking really pretty in that pink dress.
(b) We are looking at the possibility of relocating.
(c) We look to have good year in the B-School.
(d) The business seems to be looking up this year.
(e) None of the above

Q10. Mean
(a) The average of 3, 4, 5, 8 and 10 is 6 whereas the mean is 5.
(b) I believe; I’ll be meant to the job.
(c) His criticism of others is often petty and mean.
(d) His physical handicap makes his selection to the B – school no mean feat.
(e) None of the above

Q11. Tag
(a) They tagged the telephone trying in vain to reach each other.
(b) The shirt was tagged at Rs. 1500.
(c) He moved on with several people tagging along.
(d) The teacher asked them to add a tag question to every sentence.
(e) None of the above

Q12. Hold
(a) Can the roof hold all that weight?
(b) The restaurant holds 400 diners.
(c) The rule holds in most cases.
(d) Let’s hold till we have more data.
(e) None of the above

Q13. Form
(a) The captain of the team, generally low profile, was in unusual form during the
crisis.
(b) The map shows the form of North America with its mountains, rivers, and
plains.
(c) Being confined to one’s house under house arrest is a form of punishment.
(d) The inflected forms of a word can be represented by a stem and a list of
inflections to be attached.
(e) None of the above

Q14. Refer
(a) This story refers an incident during the Second World War
(b) He referred the complaint to another department.
(c) Students asked several questions referring to yesterday’s lecture.
(d) The teacher constantly refers the students to their studies.
(e) None of the above

Q15. Set
(a) Let’s set the error down to inexperience.
(b) It is wise to set food and money by in case of a future emergency.
(c) A parent must set a good example for the children.
(d) Evening was setting as I took the road over the hill.
(e) None of the above

Solutions

S1. Ans.(c)
Sol. The word ‘reserve’ fits correctly in the sentences A, C and D and express the
intended meaning.

S2. Ans.(d)
Sol. The word ‘lake’ correctly fits into the sentences A and C maximum times. In
option D, the word ‘waiting’ fits correctly but only once, hence doesn’t qualify
as correct choice.

S3. Ans.(b)
Sol. The word ‘date’ fits correctly in the sentences B and C and maximum time
appropriately. Hence the correct choice.

S4. Ans.(b)
Sol. The word ‘among’ correctly fits in the sentences C and D. for more than three
objects/persons, we use among usually. But sometimes we can use it in the case
of two too as in sentence A. hence, it can be used maximum times in option
C, D and A.

S5. Ans.(b)
Sol. The word ‘show’ can be used correctly and maximum times in the sentences
A, C and D. hence the appropriate choice.

S6. Ans.(d)
Sol. In option D, the word ‘deal’ doesn’t fit appropriately. In other sentences, it
is correctly used.

S7. Ans.(d)
Sol. In option A, B and C the word ‘see’ is correctly used but in option D, its
usage is inappropriate.

S8. Ans.(c)
Sol. The word ‘know’ is not correct according to the context of the sentence. In
other sentences, ‘know’ is correctly used.

S9. Ans.(b)
Sol. In option B, the usage of the word ‘look’ is not appropriate. It can be replaced
correctly by ‘considering’ or with any other word.

S10. Ans.(b)
Sol. In option B, the word mean is incorrect. It can be replaced by ‘considered’
or ‘selected’
Or any other word.

S11. Ans.(a)
Sol. In sentence A, the use of word ‘tag’ is not appropriate. In option C, the
phrasal verb ‘tag along’ is used. It means tag along (after someone)-to follow
along after someone; to go along with someone.

S12. Ans.(d)
Sol. In option D, the word ‘hold’ is incorrect. It can be possibly replaced by ‘wait’
or any other appropriate word.

S13. Ans.(b)
Sol. The word ‘form’ in the sentence B is inappropriate. In Option A, C and D
the usage of word ‘form’ is correct.

S14. Ans.(b)
Sol. In the second option, the use of the word ‘refer’ is not appropriate.
S15. Ans.(a)
Sol. In option A, the use of word ‘Set’ is not correct. We need some other word
to make the intended meaning clear.

PART-20

Directions (1-15): Each of the following questions has a paragraph from which
the last sentence has been deleted. From the given options, choose the one that
completes the paragraph in the most appropriate way.

Q1. Yes, nobody wants to be Japan, the fallen angel that went from one of
the fastest growing economies in the worth for more than three decades to
none that has slowed to a crawl for the past 18 years. No one wants to live
with the trauma of the deflation (falling prices) that Japan has repeatedly
experienced. No one wants to navigate the precarious government-debt
dynamic that Japan faces, with debt levels far above 100% of GDP – even if
one factors in Japanese government’s vast holdings of foreign – exchange
reserves. _______________.
(a) No one wants to go from being a world-beater to a poster child for economic
stagnation.
(b) And yet, visitors to Tokyo today see prosperity everywhere.
(c) Although hardly in crisis yet, Japan’s fiscal situation grows more alarming by
the day.
(d) Until now, the government has been able to finance its vast debts locally,
despite paying paltry interest even on longer-term borrowings.
(e) Remarkably, Japanese savers soak up some 95% of their government’s debt.

Q2. On 1 March, Philip Morris, a tobacco giant, sued eight American


retailers for selling counterfeit versions of its Marlboro cigarettes.
Governments are also boosting their efforts to crack down on counterfeiting
which deprives them of tax revenue in addition to harming legitimate
businesses. Thanks to the rise of the internet and of extended international
supply chains, and more recently, to the global economic downturn,
counterfeit goods are every-where. Fake Porsches and Ferraris zoom along
the streets of Bangkok. A German bank has discovered an ersatz gold ingot
made of tungsten in its reserves, according to a German television channel
investigating persistent reports that many of the world’s financial
institutions have been similarly hoodwinked. _______________.
(a) Counterfeiting used to be a luxury goods problem, but now people are trying
to traffic counterfeit items that have a wider effect on the economy.
(b) NASA, America’s space agency, has even bought suspect materials.
(c) Several factors have contributed to the growth of counterfeiting in recent
years.
(d) Fake goods are proliferating, to the dismay of companies and governments.
(e) The recession in the rich world may also have given a boost to counterfeit
goods.

Q3. The rate of conviction in SC and ST atrocity cases in the State has
reached 22 percent from 10 percent last year due to the proper
investigations done by the departments concerned and the increased
awareness that enabled the aggrieved persons to get justice. The government
has been appointing special public prosecutors in ‘most sensational’ and
long-pending cases and has issued guidelines to increase people’s
understanding of the SC/ST (Prevention of Atrocities) Act.
_______________.
(a) The people’s representatives should now acquaint people with the legal
remedies available for them.
(b) The Act has been implemented in letter and spirit.
(c) The Police Department act swiftly on complaints of atrocities.
(d) These measures facilitated speedy disposal of SC and ST atrocity cases.
(e) Investigations have paved the way for speedy delivery of justice.

Q4. Make no mistake: the setting matters. There are many ways to listen to
classical Indian music – in the private, somewhat sterile perfection of the
CDs and DVDs we play at home; in the concert sabhas of Mylapore and T.
Nagar; on the music channels on TV or on YouTube, which now carries a
little or a lot of almost everything, often in choppy, byte-size pieces. But I
happen to think that this music sounds best outside, on a hot spring or
summer night, with the taste of pollen and dust on your tongue and
mosquitoes circling around your feet. _______________.
(a) We tend to forget that much of the classical music of India, both Hindustani
and Carnatic, was meant for intimate settings like the royal court.
(b) Taking classical music out of ‘sabhas’ into vibrant cultural spaces brings out
its ethereal dimensions.
(c) Music is a tangible thing, to be felt in the pores of your skin no less than in
the ear; you just can’t do that very well in the concert halls.
(d) In the course of its transition to the modern concert stage, Carnatic music in
particular has sacrificed something of its delicacy.
(e) One of the important things about the settings is the recreation of a context for
listening in which subtlety and intimacy can assume their natural role.

Q5. In 2009, China overtook Germany to become the world’s largest


exporter. Exactly half the trade disputes that were filed at the World Trade
Organization (WTO) last year involved China. These facts are not unrelated.
As Pascal Lamy, the WTO’s chief, pointed out in January, the scope for
trade friction increases as countries trade more. _______________.
(a) Hence, China will test the WTO’s dispute-settlement system.
(b) Hence, disputes between China and other countries are only to be expected.
(c) Hence, China’s increasing propensity to bring disputes to the WTO is part of
a broader shift.
(d) Hence, China has moved from learning-by-watching, to being an active
participant in formal dispute settlement.
(e) Hence, more disputes may be inevitable; resolving them successfully is not.

Q6. Hundreds of farmers in long, faded cotton sarongs swarmed outside an


auditorium at Bangalore University last February. They were waiting for
India’s Environment Minister. This was the last of his public consultations
on the commercial release of BtBrinjal, a genetically modified (GM)
aubergine, created by Mahyco, an Indian hybrid-seed company, and
Monsanto, an American biotech giant. Waving placards and appetizing
images of aubergines, known in India as brinjal, they shouted themselves
hoarse praising the transgenic vegetable. But most of these men, registered
at the consultation as farmers, were in fact landless labourers with no
aubergine experience. The Minister was the first to call their bluff. The
companies, he said, without naming any, had bussed farmers from rural
districts, to play the pro-GM crowd at the hearing that
day._______________.
(a) The tactic failed miserably.
(b) The minister’s roadshow to canvas public views was unusual.
(c) Many were surprised at the Minister’s decision to snub the seed companies
and powerful domestic and American biotech lobbies.
(d) The Minister felt obliged to be responsible to science and responsive to
society.
(e) It was a setback for GM in India.

Q7. Indian’s industry going from strength to strength. Manufacturing grew


by 14.3% in the fourth quarter, compared with the same period last year.
Politicians celebrate the achievements of “India Inc”, applauding its
acquisitions abroad and welcoming the foreign investment it attracts. They
do not show anything like the same confidence in “Bharat Inc”, which is how
India’s rural economy is sometimes described. Bharat, which means India in
Hindi, is a different country. The rural heartland is courted for votes,
smothered with regulations, and shielded from the global economy that
corporate India is busy conquering. _______________.
(a) Indian agriculture has performed poorly because governments have treated it
as a source of votes rather than as an engine of growth.
(b) But its policymakers should treat farms as a potential source of growth, not
just of votes.
(c) Yet the government cannot achieve the growth it aspires to without robust
progress in agriculture, which still employs about half of India’s workforce.
(d) This year, for the first time in the country’s history, India’s factories may
contribute more to GDP than its farms, forests and fisheries.
(e) Indian agriculture can comfortably feed the country, but that remains the sum
of its achievement.

Q8. Thirty years ago the bosses of America’s car industry were shocked to
learn that Japan had overtaken America to become the world’s leading car
producer. They were even more shocked when they visited Japan to find out
what was going on. They found that the secret of Japan’s success did not lie
in cheap labour or government subsidies – their preferred explanations –
but in what was rapidly dubbed “lean manufacturing”. While Detroit slept,
Japan had transformed itself from a low-wage economy into a hotbed of
business innovation. _______________.
(a) Soon every factory around the world was lean – or a ruin.
(b) Management gurus are always glibly proclaiming revolutions.
(c) Now something comparable is taking place in the developing world.
(d) The rich world is losing its leadership in the sort of breakthrough ideas that
transform industries.
(e) Western carmakers learned the techniques of lean production from their
Japanese rivals.

Q9. When Parliament decided, in 1709, to create a law that would protect
books from piracy, the London-based publishers and booksellers who had
been pushing for such protection were overjoyed. When Queen Anne gave
her assent on 10th April the following year – 300 years ago – to “An act for
the encouragement of learning” they were less enthused. Parliament had
given them rights, but it had set a time limit on the: 21 years for books
already in print and 14 years for new ones, with an additional 14 years if the
author was still alive when the first term ran out. After that, the material
would enter the public domain so that anyone could reproduce it.
_______________.
(a) The lawmakers helped channel the spate of inventiveness that writers had in
the past.
(b) The lawmakers knew that authors do not generally consult the statute books
before deciding whether or not to pick up pen.
(c) The lawmakers did not bother about how such a deal can be made equitably.
(d) The lawmakers intended to balance the incentive to create with the interest
that people have in free access to knowledge.
(e) However, none of this should get in the way of the enforcement of copyright,
which remains a vital tool in the encouragement of learning.

Q10. Organ transplantation is one of the most impressive achievements of


modern medicine. It has brought hope to millions of patients suffering from
previously fatal organ failure. For many, it has made life longer and better.
It has benefited many professionals and industries, too, by becoming a new
source of pride, funding, and profit. Struggling to contain costs, health-care
payers are also among its beneficiaries. _______________.
(a) Kidney transplantation, for example, has proved to be less costly than dialysis.
(b) Transplant medicine has been grappling with a rapidly increasing gap between
the supply of organs and demand for them.
(c) If we are short of organs, then let us get more of them.
(d) Indeed, transplant ethics has been on a slippery slope almost since transplants
began.
(e) Organ transplantation, like mosquito repellent, should be used sparingly, and
only when there is no other choice.

Q11. How do you do it? Your colleagues, neighbours, family and friends,
how do they all do it? “I follow my nose,” says Dan Rhodes, author of Gold,
“I am always on the hunt for the next book that’s going to rock my world …
my favourite thing is still going into a shop and coming out with something
I’d never heard of.” But if you stand in any bookstore, you’re unlikely to see
many people using their noses, they just head straight for the “new” Salman
Rushdie or the “latest” Chetan Bhagat or the “most recent” Shobhaa De or
the “new bestseller” from Paulo Coelho:
(a) because they believe that books can change life.
(b) maybe that is the Catcher effect – most of us had read Catcher in the Rye and
Salinger’s other books in our late teens.
(c) It doesn’t seem to work that way now.
(d) It’s a matter of judging every book by its author.
(e) If it’s always been around you, you develop an instinct about it, else you are
never sure.

Q12. Marie Antoinette told her people to eat cake when they needed bread.
Our government encourages people to buy cars-from Rolls Royce to
the Nano-when they need affordable public transport. And when people,
especially women, want simple, basic health care-and clean water and
sanitation - they are being urged to inject their daughters with a Rs. 9,000
vaccine against cervical cancer.
(a) If some of us do these things, we should not be blamed.
(b) It has brought into focus several ethical and gender-related issues in the arena
of public health.
(c) The priorities of our decision makers are more than slightly skewed.
(d) What women need is basic healthcare, not costly medical experiments.
(e) The vaccine is supposed to protect them from cervical cancer.

Q13. A ride to Alappuzha, visiting the coir industries and boating in the
backwaters was next on the agenda. As the boats sliced the green water of
the VembanadKayal, we watched life on the backwaters of Kerala: women
selling foodstuff wrapped in banana leaves from the canoes, the transport
service, the boat stops as schools kids hopped from one boat to another to get
home, the different National waterways that the signs indicated and the
homes that had their families bathing, washing utensils and clothes.
(a) We were awestruck by the vast expanse of the lake.
(b) The backwaters were indeed the artery of this area.
(c) It was then we realized that this was India’s longest and largest lake.
(d) Life moves at a serene pace in the backwaters of Alappuzha.
(e) The VembanadKayal Wetlands is in the list of wetlands of international
importance.

Q14. The tragedy about data collection in India is that by the time primary
data is converted into useable information, it may be too late to aid policy
intervention. This is true of data collected by not just government agencies
such as the National Sample Survey Organization but also think tanks such
as National Council for Applied Economic Research (NCAER). One of the
criticisms of Human Development in India: Challenges for a Society in
Transition of Maryland, US – is that it is based on data collected at least six
years back.
(a) It brings out various dimensions of human development to understand social
inequalities, based on survey of 41,554 households.
(b) Many of its findings are an eye-opener, while some others a reaffirmation
of conclusion of other independent studies.
(c) Indicators used to measure development were household incomes and poverty
rates, land ownership and agriculture incomes, health and education.
(d) It does not capture the impact of the changes of the following years when the
economy grew at more than 8% on an average every year.
(e) Policymakers could draw inference from the findings to improve targeting of
programs aimed at inclusive growth.
Q15. Almost a decade after the launch of SarvaShikshaAbhiyan, the
achievement on universalizing elementary education is best described
as maxed. Massive spending by the Centre and states on setting up new
schools and hiring an army of teachers across the country has ensured
enrolment of children in schools has risen across rural India. Just about 4%
of children in the age group 6-14 are now estimated not to be attending
school.
(a) However, on any given day, the average attendance rate seems to be around
74% at the all-India level.
(b) However, that is the only happier part of the story.
(c) However, nearly 50% of children in class V cannot read the text for class II
without making a mistake.
(d) However, the poor quality of learning has ensured that the crores spent with
the objective of creating an educated, employable workforce are fruitless.
(e) These children would possibly grow up to be the educated employable
workforce in India.

Solutions

S1. Ans.(a)
Sol. The purpose of the paragraph is quite clear: no one wants to be an economy
like Japan. The first sentence almost states it succinctly. Answer option (a) not
only summarizes the paragraph also fulfils the purpose of the paragraph without
deviating from this purpose. Option (b) and (c) may help continue the paragraph
but bring it to a closure. The purpose of the paragraph is not the contrasting
details of these options. Options (d) and (e) may also be related to the paragraph
by stating how Japan is coping with the decline but are not related to the
purpose. Hence they too may help continue the paragraph but not close it.

S2. Ans.(d)
Sol. The purpose of the paragraph is to point out how widespread counterfeiting
has become. The example of Marlboro cigarettes and German bank are cited as
examples. Government also has become aware (trying to crack down) of the
menace. Without bringing in any new ideas and by summing up the ideas in the
paragraph option (d) smoothly closes the paragraph. Option (a), though on the
same topic of counterfeiting, is not related to the purpose of the paragraph –
especially about ‘people try to counterfeit items of wider economic impact.’
Option (b) merely helps to continue the paragraph with one more example.
Option (c) brings in ‘several factors’ which will not need to be explained.
Option (d)

S3. Ans.(d)
Sol. The paragraph states the increased conviction rates and the factors that led
to this increase. Option (a) takes off from this and states what peoples’
representatives should be doing. The writer’s purpose is not to communicate
this – what people’s representatives should be doing. Option (b) goes beyond
the scope of the details available in the paragraph – ‘in letter and spirit’ – the
paragraph only states the increase in conviction and not anything else. Option
(c) and (e) are merely details explaining the increase – do not help to close the
paragraph. Option (d) closes the paragraph by stating that the measures adopted
have contributed to achieving what is stated in the beginning of the paragraph –
closing it without any loose ends.

S4. Ans.(e)
Sol. The key to finding the answer choice here is the first sentence, “the setting
matters” and the second last sentence, “… But I happen to think that this music
sounds best outside.” If this is understood as the purpose of the paragraph, the
options that can be shortlisted are options (c) and (e). Options (c) is good only
as long as one emphasizes the ‘outside’ – then, ‘outside’ vs. ‘concert hall’ may
make sense. The major purpose of the paragraph is the ‘settings’ – and option
(e) continues this idea and closes it in the paragraph. Option (e) includes
whatever is contained in option (c) and adds to the ‘settings’ aspect of the
paragraph.

S5. Ans.(b)
Sol. If the volume of trade (exports) and disputes “are not unrelated,” what can
be expected is that China’s disputes will increase as in has become the no. 1
exporter. The word ‘hence’ (a constant in the options) is important. Hence or
therefore signals an effect. The effect of the data presented in the paragraph is
that china will have conflicts. The mildest way of stating this to say ‘conflicts
are expected.’ Option (b) hence closes the paragraph smoothly. Option has
irrelevant detail of settlement system. In option (c), ‘broader shift’ will require
further explanation. Option (d) talks about China’s involvement in dispute
settlement, which is not related to the purpose. Option (e) is almost right, but
‘resolving them successfully’ makes it unrelated.

S6. Ans.(a)
Sol. The paragraph narrates an incident. The purpose of the paragraph is nothing
more than that, if you read it carefully. People shouting themselves hoarse
supporting the issue were ‘planted’. The Minister merely called their bluff. The
most neutral sentence that can close this paragraph is that “their tactic failed” –
‘the tactic’ is enough as there is no other tactic in the paragraph. Option (b) goes
beyond the paragraph – we cannot decide that the show was unusual. Option (c)
has unsustainable implications. Many were surprised would mean – the Minister
might have supported the companies earlier; the option gets eliminated
straightaway. Option (e) may be considered. However, what was a setback for
GM in India will need further explanation.

S7. Ans.(c)
Sol. The answer is a direct take off from: “… the global economy that corporate
India is busy conquering.” Yet it cannot do that without agriculture. The first
sentence and the sentence, “they do not show anything like the same confidence
in “Bharat Inc…”, set the purpose of the paragraph. These ideas need to be
closed. Option (c) closes both. Option (a), (b) and (e) continue the paragraph.
Option (e) closes the first part of the paragraph leaving a loose end about
agriculture.

S8. Ans.(e)
Sol. Options (a) through (d) are generalizing or inferring from the given
paragraph. The paragraph is just about car manufacturers in Japan and the
Americans. The paragraph needs to be closes in relation to these aspects. Option
(e) does just that.

S9. Ans.(d)
Sol. The lawmakers give copyright for 14 or 28 years and then the book goes
into the public domain. This balances the creative urge to writer as well as the
interest people have in getting free access. Option (d) thus well closes the
paragraph. Option (a) is partial – does not conclude the public domain part.
Option (b) seems to suggest that the lawmakers were against the writer – which
is not the case in the paragraph. Option (c) too has the same shortcoming.
Option (e) is contrary to the intent of the paragraph.

S10. Ans.(a)
Sol. The answer is immediately linked to the second last sentence. “Organ
transplantation is one of the most impressive achievements… it has made life
longer and better. … benefited many professionals and industries. Struggling to
contain costs, health-care payers are also among its beneficiaries.” How they are
beneficiaries needs to be stated to logically close the paragraph. Once this is
understood, all the other options can be seen to be far cries.

S11. Ans.(d)
Sol. The sentence to complete this paragraph is a continuation of “But if you
stand in any book-store, you’re unlikely to see many people using their noses,
they just head straight for the “new” Salman Rushdie or the “latest” Chetan
Bhagat or the “most recent” Shobhaa De or the “new bestseller” from Paulo
Coelho… though the writer uses his nose to choose the book to read, other
people do not do this; they choose by the name of the author.

S12. Ans.(c)
Sol. The beginning of the paragraph tells us the purpose of the paragraph
through an example – lopsided priorities; trying to substitute for basic
necessities. Then, examples of the misplaced priorities of our government are
given. The paragraph is best concluded by stating the purpose, which otherwise
might go unnoticed. Option (a) is quite vague. Option (b) changes the basic
purpose of the paragraph. Option (d) and (e) merely states more about the last
example, but do not help to complete the paragraph.

S13. Ans.(b)
Sol. Since the writer describes the life cantered on the Vembanad Lake
(backwaters of Kerala), the best sentence to complete the paragraph is the one
that fulfils this purpose of the paragraph. Option (a), and (c) say almost the
same thing (the lake’s size and its importance) and is not purpose of the
paragraph. Option (d) too is not purpose of the paragraph, though it is connected
– but the serenity of the life is not the purpose of the description of the scene.

S14. Ans.(d)
Sol. This is a fairly straight forward choice. The paragraph states the problem
with data collection in India. The reports are delayed and become almost
irrelevant. The report on human development in India is based on data collected
six years back – the natural outcome is stated in option (d). Options (a) and (b)
cannot be criticisms, hence are not consistent with the paragraph. Option (c)
tells more about the methodology and does not conclude the paragraph. Option
(e) also does not justify the criticism mentioned in the paragraph.

S15. Ans.(d)
Sol. The purpose of the paragraph is defined by the first sentence itself, that the
achievement is mixed. However, no example or idea exists in the paragraph to
make it mixed. What is stated is positive. The last sentence, hence, needs to
state something to the contrary to complete the paragraph. Option (b), (c), and
(d) qualify for the answer options from this point of view. Option (b) does not
help to complete the paragraph. Option (c) is correct – can even be the answer
in the absence of option (d). Option (d) puts the whole scheme in perspective.
Option (c) draws attention to a specific deficiency of the scheme. Hence option
(d) scores.
PART-21

Directions (1-15): In the following questions, two sentences are given. There
may be an error in the sentence(s).

Q1. I. Yesterday I have written a letter to my friend.


II. My letter ought to have reached him this morning.
(a) if there is an error only in the first sentence;
(b) if there is an error only in the second sentence;
(c) if there are errors in both sentences; and
(d) if there is no error in either of the sentences.
(e) None of the above

Q2. I. He was so stammering that he stopped his speech between each word.
II. I shall give you sweets when you will pass the examination.
(a) if there is an error only in the first sentence;
(b) if there is an error only in the second sentence;
(c) if there are errors in both sentences; and
(d) if there is no error in either of the sentences.
(e) None of the above

Q3. I. I came to borrow a small sum of money from you.


II. Do you object to my sitting in your room for a while?
(a) if there is an error only in the first sentence;
(b) if there is an error only in the second sentence;
(c) if there are errors in both sentences; and
(d) if there is no error in either of the sentences.
(e) None of the above

Q4. I. When I see these pity children my eyes get filled with tears.
II. She expected me to feel pity on her and help her.
(a) if there is an error only in the first sentence;
(b) if there is an error only in the second sentence;
(c) if there are errors in both sentences; and
(d) if there is no error in either of the sentences.
(e) None of the above

Q5. I. They are always complaining about something or the other.


II. Do you think your parents will let you to go to England?
(a) if there is an error only in the first sentence;
(b) if there is an error only in the second sentence;
(c) if there are errors in both sentences; and
(d) if there is no error in either of the sentences.
(e) None of the above

Q6. I. It was dark inside; so I lighted a match.


II. I could not see anything but I heard some noises.
(a) if there is an error only in the first sentence;
(b) if there is an error only in the second sentence;
(c) if there are errors in both sentences; and
(d) if there is no error in either of the sentences.
(e) None of the above

Q7. I. Scarcely did I open my book than the light was put out.
II. No sooner did I reach the Railway Station than the train started.
(a) if there is an error only in the first sentence;
(b) if there is an error only in the second sentence;
(c) if there are errors in both sentences; and
(d) if there is no error in either of the sentences.
(e) None of the above

Q8. I. The Government borrowed millions of dollars from the World Bank.
II. Having received news of his death the Police stopped looking after him.
(a) if there is an error only in the first sentence;
(b) if there is an error only in the second sentence;
(c) if there are errors in both sentences; and
(d) if there is no error in either of the sentences.
(e) None of the above

Q9. I. The old should not be brushed away by society.


II. One mustn’t waste his time when there is so much to do.
(a) if there is an error only in the first sentence;
(b) if there is an error only in the second sentence;
(c) if there are errors in both sentences; and
(d) if there is no error in either of the sentences.
(e) None of the above

Q10. I. We all refrained telling her what we really thought.


II. He felt rejected by the man he admired most.
(a) if there is an error only in the first sentence;
(b) if there is an error only in the second sentence;
(c) if there are errors in both sentences; and
(d) if there is no error in either of the sentences.
(e) None of the above
Q11. I. The musician received a standing ovation by the audience.
II. Many voluntary organizations come forward to help the flood victims.
(a) if there is an error only in the first sentence;
(b) if there is an error only in the second sentence;
(c) if there are errors in both sentences; and
(d) if there is no error in either of the sentences.
(e) None of the above

Q12. I. My friend dropped in at my place, to pass some time with me.


II. The artists regaled the audience by their lively performance.
(a) if there is an error only in the first sentence;
(b) if there is an error only in the second sentence;
(c) if there are errors in both sentences; and
(d) if there is no error in either of the sentences.
(e) None of the above

Q13. I. We enjoyed very much at the picnic.


II. He was severely punished for his impatient behaviour.
(a) if there is an error only in the first sentence;
(b) if there is an error only in the second sentence;
(c) if there are errors in both sentences; and
(d) if there is no error in either of the sentences.
(e) None of the above

Q14. I. The bawling baby was rocked to sleep by its mother.


II. Let me congratulate you for your success in the examination.
(a) if there is an error only in the first sentence;
(b) if there is an error only in the second sentence;
(c) if there are errors in both sentences; and
(d) if there is no error in either of the sentences.
(e) None of the above

Q15. I. All the participants in the programme were presented mementos.


II. A warm welcome was accorded to the visiting dignitaries.
(a) if there is an error only in the first sentence;
(b) if there is an error only in the second sentence;
(c) if there are errors in both sentences; and
(d) if there is no error in either of the sentences.
(e) None of the above

Solutions
S1. Ans.(a)
Sol. Since the action has taken place yesterday; the verb must be in the past tense
– not present perfect – yesterday I wrote ……

S2. Ans.(c)
Sol. The conjunction so …… that is used to join two negatives or two positives.
Here it is ‘badly’ and ‘stopped his speech’ that are linked by so … that. He was
stammering so badly that …… here, use of conjunction is incorrect, we need
correct sentence structure.
In the second sentence the future tense is used in the subordinate clause. In the
subordinate clause of condition and time, the simple present is used instead of the
simple future … when you pass the examination.

S3. Ans.(d)
Sol. Both the sentences are correct.

S4. Ans.(c)
Sol. ‘Pity’ is a noun. It should be the adjective ‘pitiful’ as it qualifies children …
These pitiful children …… In the second sentence the preposition ‘for’ normally
follows the noun ‘pity’ …… to feel pity for ….

S5. Ans.(b)
Sol. ‘Let go’ means release or set at liberty so sentences should be … Let you go
to ……

S6. Ans.(b)
Sol. Sentence (a) is correct. In sentence (b), ‘I’ need not be repeated after ‘but’
since the subject is the same. ‘I could not see anything but heard some noises.

S7. Ans.(a)
Sol. Sentence (I) should be ‘Scarcely had I opened my book when… scarcely in
followed by when, correlative conjunction.

S8. Ans.(b)
Sol. In sentence (II) ‘looking after’ means ‘taking care of’ – e.g. Parents look after
children. ‘Looking for’ means searching which is the meaning intended here.

S9. Ans.(c)
Sol. In sentence (I), it should be brushed aside ‘not’ brushed away. ‘Aside’ means
‘to one side’, ‘out of the way’ where as ‘away’ means ‘to a distance’.
Sentence (II) should be ‘One mustn’t waste one’s time……’ One must be
followed by one’s,

S10. Ans.(c)
Sol. In sentence -I ‘refrained’ should be followed by the preposition ‘from’ – we
all refrained from telling her……
In sentence -II most must be preceded by the definite article ‘the … admired the
most. The is always used before an adjective in superlative degree.

S11. Ans.(c)
Sol. In sentence- I, received … from not by – received a standing ovation from
the audience. In sentence- II, the verb must be in the past tense – Many voluntary
organizations came forward to..

S12. Ans.(c)
Sol. In sentence I, - My friend dropped at my place to spend some time … not
pass time ……
In sentence -II, it should be regaled … With not by – The artists regaled the
audience with their …

S13. Ans.(a)
Sol. Sentence -I, ‘we enjoyed ourselves’, without the reflexive pronoun, it is
incomplete (enjoyed what?) Sentence II is correct.

S14. Ans.(b)
Sol. Sentence -II, congratulate on not for – let me congratulate you on your ……

S15. Ans.(a)
Sol. Sentence -I, the preposition should be ‘of’ not ‘in’. All the participants of
the programme

PART-22

Directions (1-15): Read the passage and answer the following questions:

Unemployment is the problem of every modern nation. The government is not


able to ensure a job for everyone. Following the conventional strategy of
creating employment, governments of many developing countries try to attract
employers (business houses/industrialists) by offering tax rebates and many
other facilities so that they locate their upcoming plants on their soil, and
thereby create industrial employment. But there is a limit to what industry can
bring. Also, industrial plants often create toxic waste which results in air and
water pollution and environmental problems which can outweigh whatever
implement benefit industrial employment may bring as substantial relief to the
dwindling economy of the host country and the profits of such foreign
investments are carried back to the parent company and foreign shareholders
aboard.

Self-employment has none of these drawbacks. The problem is that self-


employment is not as obviously glamorous is not as a shiny new factory. But
profits from self-employment remain in the country where they are produced. It
is too small to create environmental hazards. It also puts the poor person in
charge of his or her own working hours and conditions. The hours are flexible
and can be adapted to fit any family situation. It allows people to choose
between running a business full time and part time when they face a crisis, or to
put their business on hold and work full time for a salary. Self-employment is
tailor-made for anyone who is street-smart and has many acquired and inherited
traditional skills, rather than learning acquired from books and technical
schools. This means the illiterate and the poor can exploit their strengths, rather
than be held back by their weaknesses. It allows a person to turn their hobbies
into gainful employment. It allows individuals who cannot work well in
a rigid hierarchy to run their own show.

Financing the poor to start their own little ventures elevates their sense of pride
and self-respect. It offers a way out of welfare dependency, not just to become
wage slaves, but to open a store or start a manufacturing business. It can help
those who have found a job and are still nonetheless poor. It gives the victims of
prejudice who would not be hired because of their colour or national origin a
chance to earn a living. The average cost of creating self-employment is ten,
twenty or hundred times lesser than creating industry-based employment. It
helps isolated poor people gain self-confidence, step by step.

Obviously, self-employment has limits, but in many cases, it is the only


solution to help those whom economies refuse to hire and taxpayers do not want
to carry on their shoulders. The policy needed for the eradication of poverty
must be much wider and deeper than the policy for the provision of mere
employment. Real eradication of poverty begins when people are able to control
their own fate. Poor people are like bonsai trees. When you plant the best seed
of the tallest tree in a flower pot, you get a replica of the tallest tree, only inches
tall. There is nothing wrong with the seed you planted; only the soil base that is
too inadequate. Poor people are bonsai people. There is nothing wrong with
their seeds. Simply society never gave them the base to grow. All it takes to get
the poor people out of poverty is for us to create an enabling environment for
them. Once the poor are able to unleash their energy and creativity, poverty will
disappear very quickly.
Direction(1-3): Choose the word/group of words which is MOST
OPPOSITE in MEANING to The word/group of words printed in bold us
used in the passage.

Q1. RIGID
(a) Unstructured
(b) Flexible
(c) Soft
(d) Gentle
(e) Calm

Q2. STEP BY STEP


(a) All at once
(b) In quick succession
(c) In slow motion
(d) In a nutshell
(e) Once and for all

Q3. OFFERING
(a) Stealing
(b) Permitting
(c) Refusing
(d) Pretesting
(e) Questioning

Direction(4-6): Choose the word/group of words is MOST SIMILAR in


MEANING to the word printed in bold as used in the passage.

Q4. REMAIN
(a) Left-over
(b) Endure
(c) Stagnate
(d) Continue
(e) Linger

Q5. HIRED
(a) Allowed
(b) Rented
(c) Authorized
(d) Employed
(e) Delegated
Q6. LIMIT
(a) Maximum
(b) Finish
(c) Cap
(d) Decrease
(e) Barrier

Q7. Which of the following is a reason foreign investments do not


strengthen the economies of host nations?
(a) The parent company all the profit as tax to its nation.
(b) The profit of such enterprise does not remain in the host notion; rather it
goes back to the share-holders and owners of the parent company.
(c) The employees of the parent company demand extra pay from profits that
the companies earn from factories in another nation.
(d) The profit earned by such enterprises for anything beyond the salaries of
employees.
(e) None of these

Q8. What is the tone of the passage?


(a) Offensive
(b) Satirical
(c) Analytical
(d) Humorous
(e) Speculative

Q9. Which of the following is an advantage that self-employment has over


industry based employment?
(A) The work timings are highly flexible.
(B) Starting one’s own venture is an easy task and needs no investment as
financers are readily available.
(C) Self-employment makes one a master of other people and thus satisfies their
need to control others.

(a) Only (C)


(b) Only (A)
(c) Only (B)
(d) Only (A) and (B)
(e) Only (A), (B) and (C)

Q10. Which of the following may be inferred about self-employment?


(A) Self-employed slowly but steadily strengthens the economy of the country.
(B) Self-employed checks unemployment.
(C) Self-employment is slow and unprogressive, foreign investment is the only
effective tool to boost economy.

(a) Only (B)


(b) Only (B) and (C)
(c) Only (A)
(d) Only (A) and (B)
(e) Only (A) and (C)

Q11. What does the author indicate by the example of a bonsai tree?
(A) When provided the right kind of financial help, poor people can flourish.
(B) Poor people are as capable as the well-to-do class.
(C) Poor people are intolerant and incapable while adopting change.

(a) Only (A)


(b) Only (B)
(c) Only (A) and (B)
(d) Only (A) and (C)
(e) Only (B) and (C)

Q12. The author claims that self-employment is ‘tailor-made’ for people


with certain qualities. Which of the following are the qualities of such
people?
(A) They have an unconventional approach to all things.
(B) They are street smart.
(C) They possess many acquired and traditional skills.

(a) Only (C)


(b) Only (A)
(c) Only (B)
(d) Only (A) and (C)
(e) Only (A), (B) and (C)

Q13. Which of the following outweighs the employment benefits that


foreign industrialists bring?
(a) Huge industries set up by them cause environmental pollution.
(b) They employ more people belonging to their native nations, than the host
nations’ unemployed.
(c) They evade many taxes that could be a source of revenue for the host nation.
(d) They manufacture products that have no market in the host nation.
(e) They practice discrimination on grounds of gender when providing
employment to host nations’ residents.

Q14. Which of the following may be an appropriate title for the passage?
(a) Addressing conventional employment in developed nations.
(b) Varied strategies and approaches to eradicating poverty
(c) Limitations of industrial employment.
(d) How is poverty linked to conventional (industrial) employment?
(e) Role of self-employment in battling unemployment and eradication of
poverty.

Q15. Which of the following is TRUE as per the passage?


(a) Self-employment is beneficial only as per the passage?
(b) Self-employment is not as glamorous as conventional (industrial)
employment.
(c) Finance for poor is readily available in the developed nations of the world.
(d) Small-scale industries produce as much toxic waste as big industries.
(e) None is true

Solutions

S1. Ans.(b)
Sol. Rigid - not able to be changed or adapted.
Flexible- able to be easily modified to respond to altered circumstances.

S2. Ans.(e)
Sol. Step by step - so as to progress gradually and carefully from one stage to
the next.
Once and for all - now and for the last time; finally.

S3. Ans.(c)
Sol. Offering - present or proffer (something) for (someone)
Refusing - indicate that one is not willing to accept or grant

S4. Ans.(b)
Sol. Remain and Endure are synonyms referring to continuing to exist.

S5. Ans.(d)
Sol. Employed is the synonym of hired as per the usage of the word in the given
context.

S6. Ans.(a)
Sol. Maximum is the synonym of limit as per the usage of the word in the given
context which refers to the greatest amount, extent, or intensity possible,
permitted, or recorded.
S7. Ans.(b)
Sol. Refer Para-1 ‘…industrial employment may bring as substantial relief to
the dwindling economy of the host country and the profits of such foreign
investments are carried back to the parent company and foreign shareholders
aboard.’

S8. Ans.(c)
Sol. The writing style/tone of the given passage is analytical as here the author
is comparing and contrasting categories and factors related to self-employment
and eradicating unemployment. Analytical writing involves understanding
relationships between things and that is what the author is trying to do in this
passage.

S9. Ans.(b)
Sol. Refer Para-2, ‘The hours are flexible and can be adapted to fit any family
situation. It allows people to choose between running a business full time and
part time…’

S10. Ans.(d)
Sol. Refer last Para – ‘Obviously self-employment has limits, but in many cases
it is the only solution to help those whom economies refuse to hire and
taxpayers do not want to carry on their shoulders.’

S11. Ans.(c)
Sol. Refer Last Para – ‘…There is nothing wrong with the seed you planted;
only the soil base that is too inadequate. Poor people are bonsai people. There is
nothing wrong in their seeds. Simply society never gave them the base to grow.’

S12. Ans.(c)
Sol. Refer Para-2 , ‘Self-employment is tailor-made for anyone who is street-
smart and has many acquired and inherited traditional skills, rather than learning
acquired from books and technical schools.’

S13. Ans.(a)
Sol. Refer Para-1, ‘. Also, industrial plants often create toxic waste which
results in air and water pollution and environmental problems which can
outweigh whatever implement benefit industrial employment may bring as
substantial relief to the dwindling economy of the host country and the profits
of such foreign investments are carried back to the parent company and foreign
shareholders aboard.’

S14. Ans.(e)
Sol. This is the most appropriate title for the given passage.
S15. Ans.(b)
Sol. Among all the given options only option (b) is true with the given context.
Refer Para-2, ‘The problem is that self-employment is not as obviously
glamorous is not as a shiny new factory.’

PART-23

Direction (1-15): In each problem below, either part or the whole sentence is
underlined. Below the sentences are different ways of writing the underlined
part. Answer choice (A) repeats the original; the other answers vary. If you
think that the original phrasing is best, choose (A). otherwise choose other
given options.

Q1. During the summer of 2001, when it looked like parts of New Delhi and
Mumbai were going to run short of water, many businesses and homes were
affected by the stringent restrictions on the use of water.
(a) it looked like parts of New Delhi and Mumbai were going to run
(b) It looked as if parts of New Delhi and Mumbai would have run
(c) It appeared that parts of new Delhi and Mumbai would run
(d) appearances were that parts of New Delhi and Mumbai would run
(e) it was the appearance that parts of New Delhi and Mumbai would be running

Q2. Before the independence of India the books of Dr P.K. Yadav presented a
fundamental challenge to the accepted ideas of race relations that, two
generations later, will be true of the writings of the radical writers of the
1970s.
(a) that, two generations later, will be true of
(b) that, two generations later, would be true of
(c) as, two generation latter, would be true off
(d) as, two generations later, would be true of
(e) just in way that, two generations later, did

Q3. For the reason that diesel was relatively cheap and Rs. 3.00
per litre in 1960s, the average Indian came to view unfettered, inexpensive
driving as a right rather than a lucky privilege.
(a) For the reason that diesel was relatively cheap and
(b) Because diesel was relatively cheapest
(c) Due to the fact the gasoline was relatively inexpensive
(d) In that diesel was a relatively inexpensive
(e) Because diesel was relatively cheap

Q4. The political masters of the healthcare system have not listened to
professional health planners because it has not been profitable for them to do
that thing.
(a) has not been profitable for them to do that thing
(b) has not been profitable for them to do so
(c) has been unprofitable for them to do that thing
(d) has been profitable for them to do so
(e) doing so had not been profitable for them

Q5. Because of the efforts of Dr Rajendra Prasad and other advocates of the
“soft” path of solar energy, the economics of nuclear power are being more
closely examined now than ever before.
(a) being more closely examined now than ever before
(b) being attached more vigorously than ever before
(c) open to closer examination than they ever were before
(d) more closely examined than before
(e) more examined than they ever were before now

Q6. Most bacterial populations grown in controlled conditions will quickly


expand to limit the food supply, produce toxic waste products that inhibit
further growth, and reach an equilibrium state within a relatively short time.
(a) produce toxic waste products that inhibit further growth, and reached an
equilibrium state within a relatively short time
(b) will have produced toxic waste products that inhibit further growth and also
will reach an equilibrium state within a relatively short time
(c) will then produce a toxic waste product that inhibits further growth and thus
reached an equilibrium state in a very short time
(d) produce toxic waste products that inhibit further growth and reach equilibrium
(e) produce toxic waste products that inhibit further growth, and reach an
equilibrium state in a fairly prompt way

Q7. A little known danger of potent hallucinogens such as lysergic acid


diethylamide-25 is that not only is the user immediately disoriented, but also he
will experience significant ego suppression for a period of three weeks as
well.
(a) but also he will experience significant ego suppression for a period of three
weeks as well
(b) but also he will experience significant ego suppression for a period of three
weeks
(c) but also there will be a three-week period of ego suppression as well
(d) but the ego is suppressed for a period of three weeks as well
(e) but for user’s ego is suppressed for a period of three weeks in addition

Q8. Many people mistakenly believe that the body’s nutritional requirements
remain the same regardless of the quantity and form of other nutrients
ingested, physical activity and emotional state.
(a) regardless of the quantity and form of other nutrients ingested, physical
activity and emotional state
(b) regardless of the other nutrients, physical activity and emotional state.
(c) regardless of the quantity of nutrients or physical exercise or emotional
excitation
(d) regardless of the quantity or form of nutrients or physical exercise and
emotional statement
(e) regardless of the quantity or form of other nutrients ingested, physical activity
or emotional state

Q9. Measuring the brainwaves of human beings while they are engaged in
different types of thought hopefully will enable neuropsychologists to better
understand the relationship between the structures of the brain and thinking.
(a) hopefully will enable
(b) hopefully might enable
(c) will, it is hoped, enable
(d) would hopefully enable
(e) will, it is to be hoped at all, enable

Q10. It appears from a study of the detailed grammar of the Vedic Indians
that their system of assigning tenses is very different from that of English or
other European languages.
(a) It appears from a study of the detailed grammar of the Vedic Indians that their
system
(b) It seems that study of the Vedic Indians indicates that their system
(c) A detailed study of the grammar of the Vedic Indians language indicates that
its system
(d) Detailed study of Vedic Indians reveals that their system
(e) The Vedic Indians have a system

Q11. While everyone continues to hope for their survival, it is unlikely that the
astronauts could have made it back to the shelter before the power plant
exploded.
(a) have made it back to the shelter before the power plant exploded
(b) were making it back to the shelter before the power plant exploded
(c) were able to make it to the shelter before the power plant explodes
(d) have been able to make it back to the shelter before the power plant will
explode
(e) could have made it to the shelter before the power plant explosion would have
destroyed them

Q12. By the time peace and happiness will have come to the planet, many lives
will be wasted.
(a) will have come to the planet, many lives will be wasted
(b) come to the planet, many lives will have been wasted
(c) will have come to the planet, many lives will have been wasted
(d) shall have come to the planet, many lives shall be wasted
(e) would have come to the planet, many lives would have been wasted

Q13. It could be argued that the most significant virtue of a popular democracy
is not the right to participate in the selection of leaders, but rather that it
affirms our importance in the scheme of things.
(a) but rather that it affirms
(b) but rather that it is an affirmation of
(c) but rather it’s affirmation in terms of
(d) but instead of that, its affirming that
(e) affirming rather

Q14. Long popular among the connoisseurs of Indian music, Ravi Shankar first
impressed Western listeners with his phenomenal technical virtuosity, but they
soon came to appreciate his music as an artful expression of an older culture’s
musical insight.
(a) but they soon came to appreciate his music as an artful
(b) but it soon occurred that they appreciated his artful music as an
(c) but soon this was surpassed by an appreciation of it as an artful
(d) which was soon surpassed by an even deeper appreciation of it as an artful
(e) soon surpassed by an artful appreciation of an

Q15. Primarily accomplished through the use of the electron


microscope, researchers have recently vastly increased their knowledge of the
process of cell division.
(a) Primarily accomplished through the use of the electron microscope
(b) Through the competent use of advanced electron microscope
(c) Primarily through the use of electron microscopy
(d) In the large sense through the use of the electron microscope
(e) In the main, particularly through the use of electron microscope
Solutions

S1. Ans.(c)
Sol. It appeared that parts of new Delhi and Mumbai would run. Pronoun it will
be used, appeared is more appropriate word then looked.

S2. Ans.(d)
Sol. as, two generations later, would be true of. ‘as’ is correct.

S3. Ans.(e)
Sol. ‘Because diesel was relatively cheap’ this is correct word order given the
cause and effect nature of the given question statement.

S4. Ans.(b)
Sol. has not been profitable for them to do so. The end part of this phrase ‘to do
so’ completes the meaning and gives us the hint. Hence

S5. Ans.(a)
Sol. The underlined/bold part is correct itself, hence the option (A).

S6. Ans.(d)
Sol. produce toxic waste products that inhibit further growth and reach
equilibrium. Option D is more concise; the underlined/bold part is verbose. Hence
option D is correct choice.

S7. Ans.(b)
Sol. but also he will experience significant ego suppression for a period of three
weeks. The last part ‘as well” is not required to complete the conveyed meaning.
Hence option B is correct choice.

S8. Ans.(e)
Sol. regardless of the quantity or form of other nutrients ingested, physical
activity or emotional state. Connector ‘or’ will be used in place of ‘and’. Option
E is correct word order.

S9. Ans.(c)
Sol. ‘will, it is hoped, enable’ this is correct word choice.

S10. Ans.(c)
Sol. A detailed study of the grammar of the Vedic Indians language indicates that
its system. This is correct word order and grammatical correct sentence structure.

S11. Ans.(a)
Sol. have made it back to the shelter before the power plant exploded. The given
bold/underlined part is correct, hence option A is correct choice.

S12. Ans.(b)
Sol. come to the planet, many lives will have been wasted. In first part, simple
present tense will be used, hence option B is correct choice.

S13. Ans.(b)
Sol. but rather that it is an affirmation of. Option B is correct word order.

S14. Ans.(a)
Sol. but they soon came to appreciate his music as an artful. The given bold part
is correct.

S15. Ans.(c)
Sol. Primarily through the use of electron microscopy. This is better sentence
structure, concise and completes the idea. Hence option C is correct choice.

También podría gustarte